Acute - Module 10

Ace your homework & exams now with Quizwiz!

From what age may a child begin to feel a sense of maleness or femaleness? A) Birth. B) 6 months. C) 2 years. D) 5 years.

Answer: C. By the time a child is around 2 years old, he or she may begin to feel a sense of gender.Children are not born with a sense of gender identity. Some transgender individuals, however, may sense a mismatch from early childhood. This may lead to gender dysphoria or being uncomfortable with one's natal sex.

When discussing care and treatment of prostate cancer with clients, it is important for the nurse to remember that the most common issue among men who have been diagnosed with prostate cancer is the alteration of which factor? A) Comfort because of surgical pain B) Mobility after treatment C) Nutrition because of radiation side effects D) Sexual function after treatment

Answer: D. Altered sexual function is one of the biggest concerns of men after cancer treatment. Tell the client that function will depend on the type of treatment he has. Common surgical techniques used today do not involve cutting the perineal nerves needed for an erection.Comfort, mobility, and nutrition are important but are typically not the foremost concern in the minds of men with prostate cancer.

The client scheduled for chemotherapy is diagnosed with inflammatory breast cancer. Which intervention should the nurse implement based on the complete blood count (CBC) report? RBC 4.8, Hgb 12.4, Hct 40, WBC 7.8, Platelet 192, Granulocytes 75. 1. Administer the chemotherapy as prescribed. 2. Notify the oncologist immediately. 3. Request urine and blood cultures. 4. Prepare to administer one (1) unit of packed red blood cells.

ANSWER: 1. 1. The nurse should administer the chemotherapy as ordered because all listed laboratory values are WNL. 2. The laboratory data are WNL; the oncologist can see the laboratory report on rounds. 3. The laboratory data are WNL; there is no need for cultures because an infection is not indicated. 4. The laboratory data are WNL; no blood is needed. TEST-TAKING HINT: The test taker should be able to read and interpret reports so that appropriate nursing decisions can be made.

_____: The surgical removal of the prepuce or foreskin of the penis.

Circumcision

_____: Procedure for treating benign prostatic hyperplasia that uses laser energy to coagulate excess tissue. Also called interstitial laser coagulation (ILC).

Contact laser prostatectomy (CLP)

_____: Failure of the testes to descend into the scrotum.

Cryptorchidism

A 23-year-old female has a decreased level of follicle-stimulating hormone. What may this finding indicate? a. Midcycle of menses b. Pregnancy c. Premature menopause d. Infertility

D

What self-management strategy would the nurse recommend to a patient to prevent vulvovaginitis? a. Wear lightweight nylon underwear. b. Cleanse inner labial mucosa with antiseptic soap. c. Apply antiseptic cream daily to perineal area. d. Wear breathable fabrics, such as cotton.

D

Which woman is at greatest risk for pelvic organ prolapse? a. 16-year-old adolescent caring for her first child b. 25-year-old who became sexually active at age 15 c. 34-year-old who has a history of endome- triosis d. 48-year-old obese mother of four children

D

_____: Observation for cancer without immediate active treatment.

Active survellance (AS)

_____: Chemotherapy that is used along with surgery or radiation.

Adjuvant therapy

_____: Hair loss.

Alopecia

_____: The absence of menstrual periods in women.

Amenorrhea

_____: A nonspecific term to describe bleeding that is excessive or abnormal in amount or frequency without predisposing anatomic or systemic conditions. Such bleeding occurs most often at either end of the span of a woman's reproductive years, when ovulation is becoming established or when it is becoming irregular at menopause.

Dysfunctional uterine bleeding (DUB)

_____: Painful sexual intercourse.

Dyspareunia

_____: Painful urination.

Dysuria

_____: Procedure for treating benign prostatic hyperplasia with high-frequency electrical current to cut and vaporize excess tissue.

Electrovaporization of the prostate (EVAP)

_____: An infection of the endometrium.

Endometritis

_____: Inflammation of the epididymis.

Epididymitis

_____: A minimally invasive procedure in which the surgeon makes several small incisions near the umbilicus through which a small endoscope is placed to examine the abdomen; direct examination of the pelvic cavity through an endoscope.

Laparoscopy

_____: Benign, slow-growing solid tumors of the uterine myometrium (muscle layer). These are the most commonly occurring pelvic tumors.

Leiomyomas (fibroids; myomas)

_____: Sexual desire.

Libido

_____: A factor in disease development that can be altered or controlled by the patient. Examples include elevated serum cholesterol levels, cigarette smoking, hypertension, impaired glucose tolerance, obesity, physical inactivity, and stress.

Modifiable risk factor

_____: The surgical removal of leiomyomas with preservation of the uterus.

Myomectomy

_____: The need to urinate excessively at night.

Nocturia (nocturnal polyuria)

_____: Replacement of normal cells with connective tissue and collagen (scar tissue).

Fibrosis

_____: Easily crumbled or damaged.

Friable

_____: Discomfort with one's natal sex.

Gender dysphoria

_____: Abnormal enlargement of the kidney caused by a blockage of urine lower in the tract and filling of the kidney with urine.

Hydronephrosis

_____: n x-ray of the cervix, uterus, and fallopian tubes that is performed after injection of a contrast medium. This test is used in infertility workups to evaluate tubal anatomy and patency and uterine problems such as fibroids, tumors, and fistulas.

Hysterosalpingogram

_____: The most common type of breast cancer; it originates in the mammary ducts and grows in the epithelial cells lining these ducts.

Infiltrating ductal carcinoma

_____: A rare but highly aggressive form of invasive breast cancer. Symptoms include swelling, skin redness, and pain in the breasts.

Inflammatory breast cancer (IBC)

_____: Procedure for treating benign prostatic hyperplasia that uses laser energy to coagulate excess tissue. Also called contact laser prostatectomy (CLP).

Interstitial laser coagulation (ILC)

_____: Patients who self-identify as the opposite gender or a gender that does not match their natal sex.

Transgender

_____: A person who has modified his or her natal body to match the appropriate gender identity, either through cosmetic, hormonal, or surgical means.

Transsexual

_____: The condition of being swollen.

Tumescence

_____: Downward displacement of the uterus into the vagina.

Uterine prolapse

When is the best time for the nurse to begin discharge planning and a community-based plan of care for a client with prostate cancer? A) Before surgery. B) After surgery. C) 2 days before being discharged. D) The day of discharge.

Answer: A). Planning needs to begin as early as possible and would be started on admission and before surgery.After surgery is not the correct time to begin planning. Planning must begin earlier than 2 days before discharge.

The nurse is performing a history and physical on the male patient who suspects exposure to a sexually transmitted disease. Which symptom is the most common in the male with chlamydia? a. Painful intercourse b. Urethritis c. Dark yellow urine d. Thick, green discharge

B

Three years after the patient was diagnosed and treated for endometrial cancer, the patient and family are told that the cancer is recurring. Which intervention is the nurse most likely to use? a. Arrange for the patient to speak to other patients with recurring cancer. b. Assess for readiness to explore palliative care and hospice. c. Assist the patient to identify complemen- tary therapies for palliation. d. Teach about radical hysterectomy followed by brachytherapy.

B

What is the major advantage of tadalafil compared to other medications or treatments for erectile dysfunction? a. User is able to control erections. b. Erection occurs more naturally. c. There is no need to abstain from alcohol. d. Sexual stimulation is not required.

B

Which patient report indicates that the patient is experiencing the most serious complication of vaginoplasty? a. Reports burning sensation during urination b. Reports leakage of stool from the vagina c. Reports urinary incontinence when sneezing d. Reports tenderness and bruising on labia

B

Young women who have intercourse as teenagers and/or have multiple sex partners are at high risk for which disease/disorder? a. Endometriosis b. Cervical cancer c. Amenorrhea d. Ovarian cancer

B

A male patient with gender dysphoria confides in the nurse that he notices that his 4-year-old son shows a preference for playing with dolls and other traditional girls' toys. What is the nurse's best response? a. "Let him choose his own toys and he will be happier." b. "It's normal for children to explore different gender behaviors." c. "When you see him playing with dolls, how does this make you feel?" d. "Did you play with dolls and other girls' toys when you were a child?"

C

The nurse sees that a patient has been advised by the health care provider to apply lindane to the affected area. What is a self-care measure for this patient to ensure that the symptoms do not return after using the medication? a. Wash the area daily with hydrogen peroxide. b. Take a sitz bath for 30 minutes several times a day. c. Wash clothes and linens, and disinfect the home environment. d. Remove any irritants or allergens (e.g., change detergents).

C

The patient needs to be scheduled for an endometrial biopsy to assess unusually heavy menstrual bleeding. Which question is the most important to ask, in relation to scheduling the examination? a. "Have you ever had a spontaneous miscarriage or an elective abortion?" b. "How many pads per day are you using during the heaviest flow?" c. "What was the date of your last menstrual period and are you regular?" d. "Do any unexpected symptoms accompany the heavy menstrual flow?"

C

The patient reports fatigue and low libido. Based on the patient's report of symptoms, which labo- ratory result would the nurse seek out first? a. Pap smear results b. Rubella titer c. Red blood cell count d. Luteinizing hormone level

C

What is important information regarding breast cancer surveillance for a patient who had breast augmentation surgery? a. Prosthesis interferes with lump detection by breast self-examination. b. Mammograms are not useful because implant material is artificial. c. Implant displacement views allow more complete examination. d. Breast augmentation increases the risk for breast cancer.

C

What is the priority preoperative and postoperative nursing care for a patient with leiomyomas? a. Preventing infection b. Managing severe pain c. Monitoring for bleeding d. Assessing for and managing anxiety

C

Women with pelvic inflammatory disease are at an increased risk for which condition? a. Amenorrhea b. Appendicitis c. Infertility d. Cardiac disease

C

A young female patient requires hospitaliza- tion for a severe case of genital herpes. What information is given to the patient regarding the long-term consequences? a. There is an increased risk of central nervous system complications. B. There is a risk of neonatal transmission and an increased risk for acquiring HIV infection. C. There is an increased risk for scars and adhesions of the fallopian tubes. D. There is an increased risk for multiple types of reproductive cancers at a young age.

B

A young woman is suspected of having invasive breast cancer. Based on the types and frequencies of breast cancer, what is the most likely diagnosis? a. Fibrocystic breast condition b. Infiltrating ductal carcinoma c. Lobular carcinoma in situ d. Ductal carcinoma in situ

B

The patient has a continuous bladder irrigation via a three-way urinary catheter. At 7:00 am, the urine drainage bag was emptied and 1000 mL of irrigation fluid was hung. At 11:00 am, 350 mL of irrigation fluid had been administered through the catheter. The urinary drainage bag now contains 600 mL. How many mL of urine has the patient produced in the past 4 hours? ________ mL

250 Ml

According to the American Cancer Society, what are the recommendations for early detection by screening for breast masses? a. Women aged 45-54 should have an annual mammogram, then every 2 years at age 55 and older. b. High-risk women should have biannual mammograms and magnetic resonance imaging. c. High-risk women should be screened for breast cancer annually starting at age 21. d. Women aged 60 years or older should have a mammogram every 10 years.

A

During the first 24 hours after prostatectomy, what is the priority concern? a. Hemorrhage b. Infection c. Hydronephrosis d. Confusion

A

Females with an insufficient estrogen level should be assessed for which condition? a. Osteoporosis b. Diabetes mellitus c. Endometriosis d. Decreased immune function

A

For which patient would a test of cure be recommended after treatment is completed? a. Patient is treated for gonorrhea with cefixime. b. Patient is treated for chlamydia with azithromycin. c. Patient is treated for syphilis with benzathine penicillin. d. Patient is being treated for gonorrhea with ceftriaxone.

A

In caring for transgender patients, under which circumstance would the nurse make a clinical judgment and decide to forego exten- sive questioning about gender identity? a. Needs treatment for a sprained ankle sustained during a soccer game. b. Has recurrent urinary tract infections despite medication compliance. c. Appears to be a male but requests a pelvic examination by a female provider. d. Is dressed as a man and wants information about hormones that feminize the body.

A

In performing a genital exam on a teenage patient, the examiner sees multiple large cauliflower-like growths in the perineal area. The patient reports these appeared about 3 months after her first sexual experience. What does the examiner suspect? A. Condylomata acuminate b. Genital herpes c. Salpingitis d. Gonorrhea

A

The nurse is caring for a patient who had min- imally invasive surgery for testicular cancer. The nurse is also caring for a patient who had an open radical retroperitoneal lymph node dissection for testicular cancer. The nurse anticipates that the second patient has greater risk for which condition? a. Paralytic ileus b. Urinary incontinence c. Lower urinary tract symptoms d. Fluid overload

A

The nurse sees that the patient is taking tamsulosin. Which question would the nurse ask to determine if the medication is achieving the desired therapeutic effect? A. "Are you still having trouble passing urine?" b. "Does your urine have a strong odor or appear cloudy?" c. "Are you having any problems with achieving an erection?" d. "Have you had a green or yellow discharge from your penis?"

A

What does the nurse tell a patient with pelvic inflammatory disease (PID) about the practice of vaginal douching? A. Douching increases the risk for developing PID. B. Douche daily to reduce vaginal infections. C. Vinegar is a safe and natural alternative. D. Disposable equipment is recommended.

A

What postprocedure instructions would the nurse give to a patient who had a prostate biopsy? a. Light rectal bleeding and blood in the urine or stools is expected for a few days. b. Swelling of the biopsy area and difficulty urinating are expected for 1 week. c. Low-grade fever and bright-red penile discharge are normal for several days. d. Return to see the health care provider in 1 week for recheck of biopsy site.

A

_____: Cosmetic surgical procedure to enhance the size, shape, or symmetry of the breasts.

Breast augmentation

_____: Highly controversial practice of surgically removing the breast in order to reduce the risk of breast cancer.

Prophylactic mastectomy

A female patient is prescribed azithromycin 1 g orally in a single dose for the treatment of Chlamydia trachomatis. What additional information does the nurse give the patient about treatment issues? a. Abstain from sex for 7 days from the start date of treatment. b. Even with treatment, there is risk for meningitis and endocarditis. c. There is no need for rescreening unless there is new exposure. d. Watch for and report headache, malaise, or loss of appetite.

A

A patient had a pelvic examination and needs an additional diagnostic test for possible uterine leiomyomas. The nurse prepares the patient for which diagnostic test? a. Transvaginal ultrasound b. Laparoscopy c. Hysteroscopy d. Endometrial biopsy

A

A patient had a transrectal ultrasound with biopsy earlier in the day. What urine characteristics does the nurse expect to see? a. Light pink urine b. Bright red urine c. Dark urine with small clots d. Very pale yellow urine

A

A young female patient with a history of previous sexually transmitted disease has a hunched-over gait and has difficulty getting on the examination table. On observing this be- havior, what does the nurse first assess for? A. Lower abdominal pain b. Lower back pain c. Musculoskeletal weakness d. Vaginal bleeding

A

The nurse is reviewing the laboratory results from a postmenopausal woman being evalu- ated for a breast mass. What type of metastasis does the increased serum calcium and alkaline phosphatase levels suggest? A. Brain b. Bone c. Lung d. Liver

B

Which classic symptom is indicative of invasive gynecologic cancer? a. Swelling of the lymph nodes in groin area b. Dark and foul-smelling vaginal discharge c. Painless vaginal bleeding unrelated to menses d. Flank pain with dysuria and dark urine

C

Which patient statement most accurately de- scribes identifying oneself as transgender? a. "I enjoy wearing women's clothes. Women's fashions are pretty and interesting." b. "I think men are more powerful and influential, so I would rather be viewed as male." c. "Since childhood, I have always felt like I was born into the wrong body." d. "I have always been sexually attracted to other males and other men often reciprocate."

C

Which test detects cancerous and precancer- ous cells of the cervix? a. Serologic studies b. Vaginal culture c. Pap smear d. Human papillomavirus test

C

_____: Procedure for treating benign prostatic hyperplasia using low radiofrequency energy to shrink the prostate.

Transurethral needle ablation (TUNA)

_____: In health care, a small bladder or blister.

Vesicle

_____: The external female genitalia.

Vulva

What is the first symptom of primary syphilis? a. Small, painless, indurated, smooth, weeping lesion b. Urinary frequency, burning incontinence, and dribbling c. Malaise, low-grade fever, and general muscular aches and pains d. Rash that changes from papules to squamous papules to pustules

A

_____: A type of bone marrow transplant in which patients receive their own stem cells, which were collected before high-dose chemotherapy.

Autologous bone marrow transplantation

What does the nurse instruct the patient to do before a scheduled breast augmentation surgery? Select all that apply. A. Stop oral contraceptives. B. Stop smoking. C. Avoid taking NSAIDs. D. Avoid taking ginseng. E. Wear a supportive bra. F. Bathe with a mild soap.

B,C,D

The nurse is reviewing discharge instructions for a patient who had breast augmentation surgery. What does the nurse include in these instructions? Select all that apply. a. Expect soreness in chest and arms for several months. b. Breasts will feel tight and sensitive; the breast skin may feel warm or itchy. c. Anticipate having difficulty raising the arms over the head. d. Perform lifting, pushing, and pulling exercises several times a day. E. Walk every few hours to prevent deep vein thrombosis. F. Expect some swelling for 3-4 weeks after surgery.

B,C,E,F

_____: The surgical removal of both testes, typically performed as palliative surgery in patients with prostate cancer. It is not intended to cure the prostate cancer but to arrest its spread by removing testosterone.

Bilateral orchiectomy

_____: Surgical method for breast cancer that removes the bulk of the tumor rather than the entire breast.

Breast-conserving surgery

_____: The ulcer that is the first sign of syphilis. It develops at the site of entry (inoculation) of the organism, usually 3 weeks after exposure. The lesion may be found on any area of the skin or mucous membranes but occurs most often on the genitalia, lips, nipples, and hands and in the oral cavity, anus, and rectum.

Chancre

A male patient reports that a female sexual partner just told him that she was treated for gonorrhea. What symptoms does the nurse ask about, because they are the most likely to occur in a male with gonorrhea? a. Small, painless lump that occurred on the penis but spontaneously disappeared b. Numerous small, painless, papillary growths in the genital area c. Painful intercourse because of scrotal swelling and epididymitis d. Dysuria and a profuse yellowish green or scant clear penile discharge

D

A new nurse has been conscientious about asking all transgender patients about preferred use of pronouns and names, but inadvertently makes an error while caring for a patient. What should the new nurse do? a. Apologize and explain that working with transgender patients is a new experience. b. Assume that the patient is used to this type of error and continue care. c. Watch the patient's nonverbal behavior to gauge if the error was noticed. d. Self-correct and continue with care rather than making a prolonged apology.

D

_____: Examination of the cervix and vagina using a colposcope, which allows three-dimensional magnification and intense illumination of epithelium with suspected disease. This procedure can locate the exact site of precancerous and malignant lesions for biopsy.

Colposcopy

_____: The removal of a cone-shaped sample of tissue from the cervix for cytologic study.

Conization

A 40-year-old woman has heavy vaginal bleeding. Which question is the priority in evaluating the patient's condition? a. "Is the bleeding related to the menstrual cycle or intercourse?" b. "Are you having any sensations of pain or cramping?" c. "Are you sexually active and do you use oral contraceptives?" d. "Are you feeling weak, dizzy, or lightheaded?"

D

_____: (1) A way of decreasing muscle pain by "cooling down" the area with a local, short-acting gel or cream, such as after physical therapy; (2) in ophthalmologic surgery, use of a freezing probe to repair retinal detachment.

Cryotherapy

_____: An early, noninvasive form of breast cancer in which cancer cells are located within the duct and have not invaded the surrounding fatty breast tissue.

Ductal carcinoma in situ (DCIS)

_____: A benign breast disease caused by dilation and thickening of the collecting ducts in the subareolar area. The ducts become distended and filled with cellular debris, which activates an inflammatory response. It is usually seen in women approaching menopause.

Ductal ectasia

_____: Cancer of the inner uterine lining.

Endometrial cancer

_____: The inability to achieve or maintain a penile erection sufficient for sexual intercourse.

Erectile dysfunction (ED)

_____: Redness of the skin.

Erythema

_____: Therapy used to treat chlamydia in which patients are given a drug or prescription with specific instructions for administration to their partners without direct evaluation by a health care provider. Also called patient-delivered partner therapy.

Expedited partner therapy (EPT)

_____: A person's inner sense of maleness or femaleness not related to reproductive anatomy.

Gender identity

_____: Abnormal enlargement of the breasts in men.

Gynecomastia

_____: Blood in the urine.

Hematuria

_____: Abnormal distention of the ureter.

Hydroureter

_____: Growth that causes tissue to increase in size by increasing the number of cells; abnormal overgrowth of tissue.

Hyperplasia

_____: Examination of the interior of the uterus and cervical canal using an endoscope.

Hysteroscopy

_____: Acronym for "lesbian, gay, bisexual, transgender, and queer/questioning" culture.

LGBTQ

_____: A noninvasive form of breast cancer that does not show up as a calcified cluster on a mammogram and is therefore most often diagnosed incidentally during a biopsy for another problem.

Lobular carcinoma in situ (LCIS)

_____: Symptoms that occur as a result from prostatic hyperplasia, such as urinary retention and overflow incontinence, or urinary leaking.

Lower urinary tract symptoms (LUTS)

_____: Abnormal accumulation of protein fluid in the subcutaneous tissue of the affected limb after a mastectomy.

Lymphedema

_____: An x-ray of the soft tissue of the breast.

Mammography

_____: Low sperm count.

Oligospermia

_____: An acute testicular inflammation resulting from trauma or infection.

Orchitis

_____: Loss of urine during activities that increase intra-abdominal pressure, such as laughing, coughing, sneezing, or lifting heavy objects.

Stress urinary incontinence (SUI)

_____: A procedure in which the interventional radiologist threads a small vascular catheter into the prostate's arteries and injects particles blocking some of the blood flow to shrink the prostate gland.

Prostate artery embolization

_____: Inflammation of the prostate.

Prostatitis

_____: Rectal mucosa inflammation that results from external beam radiation therapy.

Radiation proctitis

_____: Surgery, particularly procedures that affect the external or internal genitalia, that transitions an individual from one's natal sex to one's inner gender identity.

Sex (gender) reassignment surgery

_____: A complex sexually transmitted disease that can become systemic and cause serious complications and even death. It is caused by the spirochete Treponema pallidum, which is found in the mouth, intestinal tract, and genital areas of people and animals. The infection is usually transmitted by sexual contact, but transmission can occur through close body contact and kissing.

Syphilis

Which sexually transmitted diseases are vaginal infections? (Select all that apply). A) Chlamydia. B) Endometritis. C) Epididymitis. D) Gonorrhea. E) Proctitis. F) Syphilis.

"Answer: A, D, F. Chlamydia can be transmitted during vaginal, anal, or oral sex. Gonorrhea is spread through contact with the penis, vagina, mouth, or anus. Both gonorrhea and chlamydia can also be spread from an infected mother to her baby during vaginal childbirth. Syphilis is a bacterial infection usually spread by sexual contact which starts as a painless sore, typically on the genitals, rectum, or mouth. Syphilis spreads from person to person via skin or mucous membrane contact, such as vaginally.Endometritis is the infection of the innermost lining of the uterus (the endometrium). Epididymitis is an inflammation of the coiled tube (epididymis) at the back of the testicle that stores and carries sperm. Pain and swelling are the most common signs and symptoms of epididymis. Proctitis is an inflammation of the rectum that causes discomfort, bleeding, and, occasionally, a discharge of mucus or pus. Sexually transmitted infections, spread particularly by people who engage in anal intercourse, can result in proctitis."

A 29-year-old patient has strictures and adhesions in her fallopian tubes. This can be the result of which condition? a. Pelvic inflammatory disease b. Frequent episodes of colitis c. Gallbladder disease d. Frequent urinary tract infections

A

A patient has just been informed that she has an abnormal Pap smear and a positive human papillomavirus test. The nurse should be prepared to provide information about which topic? a. Increased risk for cervical cancer b. Increased risk for endometrial cancer c. Increased risk for herpes simplex virus type 2 d. Increased risk for human immunodefi- ciency virus

A

A patient is diagnosed with primary syphilis. The nurse prepares to administer and educate the patient about which treatment regimen? a. Benzathine penicillin G given intramuscu- larly as a single 2.4-million-unit dose, and follow-up evaluation at 6, 12, and 24 months b. Benzathine penicillin G given intramuscu- larly for 7 days and follow-up evaluation at 6, 12, and 24 months c. Ceftriaxone 125 mg intramuscularly in a single dose plus azithromycin 1 g orally in a single dose, and follow-up evaluation at 6, 12, and 24 months d. Metronidazole 500 mg orally twice daily for 14 days and follow-up evaluation at 6, 12, and 24 months

A

A patient reports the sensation of feeling as if "something is falling out" along with painful intercourse, backache, and a feeling of heavi- ness or pressure in the pelvis. Which question does the nurse ask to assess for a cystocele? a. "Are you having urinary frequency or urgency?" b. "Do you feel constipated?" c. "Have you had problems with hemor- rhoids?" d. "Have you had any heavy vaginal bleeding?"

A

A patient requires treatment for genital warts. Which treatment can be done by the patient at home if given the proper instructions? a. Imiquimod 5% cream b. Cryotherapy with liquid nitrogen c. Podophyllin resin 10% in a compound of tincture of benzoin d. Trichloroacetic acid or bichloracetic acid

A

A young woman had minimally invasive surgery for the removal of uterine fibroids. The nurse em- phasizes that this information should be included when giving health history; however for which future scenario is the history most essential? a. Becomes pregnant and is looking forward to a home delivery with midwife assistance b. Potentially needs a hysterosalpingogram for evaluation of fallopian tube patency c. Develops recurrent and frequent episodes of vulvovaginitis related to Candida albicans d. Plans to take oral contraceptives for sev- eral years to delay pregnancy

A

The nurse notes bright-red blood with numerous clots in the urinary drainage bag for a patient who had a transurethral resection of the prostate. Besides notifying the surgeon, what is the nurse's best action? a. Irrigate the catheter with normal saline per protocol. b. Remove the urinary catheter and save the tip to culture. c. Start an IV infusion and draw blood for type and cross. d. Empty the drainage bag and record the appearance of output.

A

Which sexually transmitted disease is detectable by urine self-collection? a. Chlamydia b. Herpes simplex type 2 c. Syphilis d. Gonorrhea

A

Which woman has the highest risk for developing breast cancer? a. 68-year-old who takes hormone replacement therapy b. 35-year-old who has three children and had one miscarriage c. 23-year-old who started menstruating at age 12 d. 40-year-old who has two cousins who had breast cancer

A

The acronym LGBTQ (lesbian, gay, bisexual, transgender, and queer/questioning) is used to describe a group of people of minority sexual and gender identities under one population category. Which of these terms refer specifically to sexual orientation? Select all that apply. A. Lesbian b. Gay c. Bisexual d. Transgender e. Queer f. Questioning

A,B,C

A young patient has been diagnosed with gonorrhea. What additional testing should be offered to this patient because of possible concurrent infections? Select all that apply. a. Syphilis b. Chlamydia c. Hepatitis B d. Hepatitis C e. HIV infection f. Candida albicans

A,B,C,D,E

For a patient with a vaginoplasty what would be included in the self-care measures? Select all that apply. a. Routine douching and douching after intercourse to prevent infection. b. Regular sexual intercourse to keep the vagina dilated. c. Inserting the vaginal dilator several times a day for months after surgery. d. Using a water-based lubrication when inserting the vaginal stent. e. Watching for and immediately reporting feces leaking from vagina. f. Applying an ice pack to the perineum twice daily for two months after surgery.

A,B,C,D,E

The nurse is caring for a patient who had a laparoscopy. What is included in the postopera- tive care for this patient? Select all that apply. a. Administer oral analgesics for incisional pain. b. Notify the health care provider of postop- erative shoulder pain. c. Reassure the patient that most painful sensations disappear within 4-6 weeks. d. Instruct the patient to change the small adhesive bandage as needed. e. Teach the patient to observe the incision for signs of infection or hematoma. f. Remind the patient to avoid strenuous activity for 4-6 weeks after the procedure.

A,D,E

What are common serum tumor markers that confirm a diagnosis of testicular cancer? Select all that apply. a. Lactate dehydrogenase b. Early prostate cancer antigen c. Glutathione S-transferase d. Alpha-fetoprotein e. Beta human chorionic gonadotropin f. BRCA1 or BRCA2 mutations

A,D,E

The client is diagnosed with primary syphilis. Which symptoms should the nurse observe? 1. A chancre sore in the perineal area. 2. A rash on the trunk and extremities. 3. Blistering of the palms of the hands. 4. Confusion and disorientation.

ANSWER: 1. 1. A chancre sore on the perineal area is a symptom of primary syphilis. 2. A rash on the trunk and extremities occurs in secondary syphilis. 3. Blistering of the palms of the hand occurs in secondary syphilis. 4. Tertiary syphilis occurs over a prolonged period and includes symptoms of dementia, psychosis, paresis, stroke, and meningitis.

The nurse is caring for client A, who is postoperative after a breast biopsy. The pathology report is posted on the chart. Which statement is the interpretation of the DNA ploidy pathology report? REPORT CLIENT A: A 0.5- 1.2-cm specimen of tissue from the right breast was evaluated with microscope examination. Margins were not clear. Tissue sample indicates high-grade invasive ductal cell carcinoma. DNA ploidy by flow cytometry reveals aneuploid characteristics. Findings: Invasive ductal cell carcinoma of the right breast with aneuploid findings. 1. This is Stage IV breast cancer with a poor prognosis. 2. The cancer will respond to hormonal therapy. 3. The chromosomes do not resemble normal human DNA. 4. The client should have a mastectomy as soon as possible.

ANSWER: 3. 1. Staging for breast cancer is completed using a variety of measurements, including size of the tumor and metastasis sites, but staging does not involve the DNA characteristics. 2. The DNA ploidy tests are performed to determine the response to all types of treatments. The more the cell represents normal human DNA pairings (euploid), the better the prognosis for the client. 3. Aneuploid means the cells do not have human pairing characteristics. This finding indicates the cells cannot be expected to respond as normal human cells respond and the prognosis is not good for the client. 4. The choice of surgical approach is determined between the client and HCP. The tumor should be removed with enough tissue to have clear margins. Whether the client chooses a modified radical mastectomy, lumpectomy, or wedge resection depends on the choice of follow-up treatment.

The charge nurse is making rounds on the genitourinary surgery floor. Which action by the primary nurse warrants immediate intervention? 1. The nurse elevates the scrotum of a client who has had an orchiectomy. 2. The nurse encourages the client to cough, although he complains of pain. 3. The nurse empties the client's JP drain and leaves it rounded. 4. The nurse asks the UAP to empty a catheter drainage bag.

ANSWER: 3. 1. This should be done for clients who have had scrotal surgery. 2. Postoperative clients do not want to perform deep-breathing and coughing exercises because it hurts, but they should be encouraged to do so to prevent complications. 3. The Jackson Pratt (JP) drain is a drain attached to a bulb, and the bulb should remain compressed to apply gentle suction to the surgical site. 4. This is appropriate delegation. TEST-TAKING HINT: The test taker could eliminate option "4" by knowing the rules of delegation. The nurse did not ask the UAP to perform anything requiring nursing judgment or assessment. Option "2" is just good nursing practice and could be eliminated. The charge nurse does not need to stop this action

Which specific complication should the nurse assess for in the client with a uterine prolapse recovering from an anterior and posterior repair? 1. Orthostatic hypotension. 2. Atelectasis. 3. Allen sign. 4. Deep vein thrombosis

ANSWER: 4. 1. Orthostatic hypotension is not a surgeryspecific complication. 2. Atelectasis is a complication of general anesthesia, not an A & P repair. 3. An Allen test is a physical examination evaluating arterial blood supply to the radial and ulnar arteries. 4. Assessing for deep vein thrombosis (DVT) is performed on all clients having a vaginal hysterectomy. After any surgery requiring the client to be placed in the lithotomy position, the client should be assessed for DVT. These clients are at a higher risk for this complication. TEST-TAKING HINT: The test taker must have knowledge of this surgical procedure to be able to answer this question. A good choice is to choose an option assessed for most clients who are immobile.

The client diagnosed with testicular cancer is scheduled for a unilateral orchiectomy. Which information is important to teach regarding sexual functioning? 1. The client will have ejaculation difficulties after the surgery. 2. The client will be prescribed male hormones following the surgery. 3. The client may need to have a penile implant to be able to have intercourse. 4. Libido and orgasm usually are unimpaired after this surgery.

ANSWER: 4. 1. The client usually will be able to maintain normal sexual functioning with the remaining testicle. If not, testosterone may be prescribed to improve functioning. Ejaculation is not the problem; it is impotence (erectile dysfunction). 2. This will not be done until the HCP determines the remaining testicle is not able to maintain adequate hormone production. 3. The client will be able to function sexually with the remaining testicle alone or with prescribed male hormones. 4. Sex drive (libido) and orgasms usually are unimpaired because the client still has one testicle. TEST-TAKING HINT: The key to this question is "unilateral." The client will still have one functioning testicle.

The nurse in the gynecology clinic is assessing the 50-year-old client who has had four (4) children and is complaining of having lower abdominal pressure and fatigue along with some urinary incontinence. Which instruction should the nurse teach the client? 1. Wear a peri-pad to keep from having an accident. 2. Try not to laugh or sneeze unless at home. 3. Discuss the pros and cons of a vaginal hysterectomy. 4. Instruct to perform Kegel exercises.

ANSWER: 4. 1. The client would have determined the need for protection without the nurse having to tell her to do so. 2. It is unrealistic to tell a client not to laugh or sneeze. 3. The client probably has a cystocele resulting from childbirth. The corrective surgical repair is a bladder suspension. 4. Kegel exercises help to strengthen the pelvic muscles. They are recommended for all women and should be performed 30 to 80 times per day

Which steps should the nurse provide clients who choose to perform breast self-examination (BSE) according to the American Cancer Society (ACS) guidelines? Rank in order of performance. 1. Lie flat on the bed with a rolled towel placed under the scapula; perform palpation of each breast. 2. Pinch each nipple to see if fluid can be expressed. 3. With the breasts exposed, stand in front of a mirror and examine the breasts from front and each side. 4. In the shower, soap the breasts, and perform palpation in a systematic manner on each breast. 5. Find a private place where the selfexamination can be performed.

ANSWER: In order of performance: 5, 3, 4, 1, 2. 5. Breast self-examination should be performed in a private area with good lighting and a comfortable temperature so the woman is warm and can be consistent in the steps. 3. The first part of BSE is performed in front of a mirror. The breasts are examined from the front and each side, then with arms down by the side, then raised, next with the arms on the hips and bending over. The woman is looking for dimpling and irregularity in size or shape. 4. The woman then gets into a warm shower and palpates each breast when the breasts are soapy and slippery. 1. The last position the woman assumes is lying flat on the bed with a towel behind each scapula and palpation is performed. 2. After each breast is palpated the last thing done is to pinch the nipples to see if there is a discharge. TEST-TAKING HINT: The test taker must remember basic guidelines for procedures and which steps are involved.

The nurse is teaching a class on Breast Health Awareness. Which are the American Cancer Society's recommended guidelines for the performance of breast self-examination (BSE)? List in order of recommended performance. 1. Visualize the breast from the front while standing before a mirror. 2. Gently squeeze the nipple to express any fluid. 3. Turn to each side and view each breast in the mirror. 4. Palpate each breast in a circular motion while lying on the back. 5. Palpate each breast in a circular motion while in the shower

ANSWER: The order should be: 1, 3, 5, 4, 2. 1. The first step in BSE is to visualize the breasts for symmetry while looking at a frontal view before the mirror. 3. The next step is to turn from side to side, looking for any dimpling, puckering, or asymmetry, in front of a mirror. 5. The client should palpate the breasts in a warm shower with the breasts soaped to allow for the fingers to glide over the breast tissue. 4. After the shower, the client should lie on the bed with a towel rolled up and placed under the shoulder to flatten the breast tissue and palpate the breast. 2. The last step in BSE is to gently squeeze the nipple to determine if there is expressed fluid.

The nurse is reviewing laboratory results on a 34-year-old client who is suspected of having endometrial (uterine) cancer. Which laboratory tests does the nurse expect to see? (Select all that apply). A) Alpha-fetoprotein (AFP) test B) Cancer antigen (CA)-125 test C) Human chorionic gonadotropin (hCG) level D) Complete Blood Count (CBC) E) Serum electrolytes F) Hereditary nonpolyposis colon cancer (HNPCC) test

Answer: A, B, C, D, F. Serum tumor markers to assess for metastasis include alpha-fetoprotein (AFP) and CA-125 (cancer antigen-125), both of which may be elevated when ovarian cancer is present (Pagana and Pagana, 2014). A human chorionic gonadotropin (hCG) level may be taken to rule out pregnancy before treatment for cancer begins. A complete blood count (CBC) typically shows anemia because the client has heavy bleeding. Testing for HNPCC is done if a family history is reported because a connection has been noted between HNPCC and endometrial cancer.Serum electrolytes are not routinely checked in the diagnostic process for endometrial cancer but might be performed later.

Which routes are used for testosterone administration? (Select all that apply). A) Buccal. B) Intramuscular. C) Intravenous. D) Oral. E) Transdermal.

Answer: A, B, D, E. Testosterone can be given via buccal, intramuscular, oral, and transdermal routes.Testosterone cannot be given intravenously.

A male-to-female client has a body mass index of 29 and is planning to have transvaginal surgery. The nurse is aware that this client is at higher risk for which complications? (Select all that apply). A) Atelectasis. B) Difficulty ambulating. C) Ileus formation. D) Nausea/vomiting. E) WOund infection.

Answer: A, B, E. Individuals are considered overweight if they have a body mass index over 24.9. Overweight clients who have had transvaginal surgery are at higher risk for difficulty with ventilation leading to atelectasis, difficulty with ambulating, and wound infection.Nausea/vomiting is a risk after any surgery with general anesthesia. Ileus is a risk with abdominal surgery.

Which gynecologic clients does the charge nurse assign to an LPN/LVN? (Select all that apply). A) A 23-year-old who is nauseated after her laparotomy and needs to receive antiemetic drugs B) A 34-year-old who had a total hysterectomy for invasive cervical cancer and has a blood pressure (BP) of 88/54 mm Hg C) A 42-year-old who had an abdominal hysterectomy whose primary health care provider wants to remove sutures at her bedside D) A 48-year-old who is receiving IV chemotherapy to treat stage II cervical cancer E) A 52-year-old who just returned to the unit following a total abdominal hysterectomy

Answer: A, C. Both the client who is nauseated after laparotomy and needs to receive antiemetic drugs and the client who had an abdominal hysterectomy and whose primary health care provider wants to remove sutures can be cared for by an LPN/LVN.The client with a total hysterectomy and low BP has a deteriorating status (dropping BP) and requires treatment that only an RN can administer. Also, the client who is receiving IV chemotherapy requires treatment that only an RN can administer. The client who has just returned from surgery must first be assessed by the RN to determine if she is stable.

The nurse is teaching a group of young men about sexually transmitted diseases. What does the nurse tell them to look for in the primary stage of syphilis? A) A painless chancre. B) A rash in the genital area. C) Scrotal swelling. D) Weeping discharge from the urethra.

Answer: A. The appearance of an ulcer called a chancre is the first sign of primary syphilis.Genital rash is a common manifestation of the secondary stage of syphilis. Scrotal swelling is not a symptom of syphilis. Weeping discharge from the urethra is a symptom of gonorrhea.

The nurse is establishing a plan of care for a client with newly diagnosed pelvic inflammatory disease (PID). Which problem does the nurse place as the client's highest priority? A) Infection. B) Infertility. C) Reduced sexual drive. D) Reduced self-esteem.

Answer: A. The highest priority problem for clients with pelvic inflammatory disease (PID) is infection related to invasion of pelvic organs by sexually transmitted pathogens.Although important, infertility (or the risk for the development of it), reduced sexual drive, and reduced self-esteem are not the client's most important problems in this scenario.

Periodic laboratory tests will be monitored for a female-to-male client who is taking testosterone. Which laboratory tests does the nurse tell the client to anticipate will be monitored? (Select all that apply). A) Arterial blood gases (ABGs) B) Blood glucose. C) Blood urea nitrogen (BUN)/creatinine D) Lipid profile. E) Liver profile

Answer: B, D, E. Testosterone therapy may cause an increase in blood glucose and liver enzymes as well as a decrease in high-density lipids and an increase in low-density lipids.ABGs will not be affected and will not routinely be monitored. BUN and creatinine are not generally monitored as part of routine blood chemistries with testosterone therapy.

The nurse is teaching a client who is diagnosed with gonorrhea. What does the nurse tell the client about the disease? A) "Close follow-up is required because treatment failure is common." B) "Do not engage in sexual activity until your blood tests are negative." C) "You are contagious even if you have no outward symptoms." D) "You are only infectious while the lesions are draining."

Answer: C. Gonorrhea can be asymptomatic in both men and women, but women have asymptomatic, or "silent," infections more often than men.Treatment failure is rare when gonorrhea is treated according to Centers for Disease Control and Prevention recommendations. Clients need to avoid sexual activity until antibiotic therapy is completed and they no longer have physical symptoms. Lesions are not associated with gonorrhea.

Which method is a common integrative health therapy for benign prostatic hyperplasia (BPH)? A) Acupuncture. B) Calcium supplements. C) Serenoa repens. D) Yoga.

Answer: C. Serenoa repens (saw palmetto), a plant extract, is often used by men with early to moderate BPH. They believe that this agent relieves their symptoms and prefer this treatment over prescription drugs or surgery. However, studies on the effectiveness of Serenoa repens have not shown that it is effective. The nurse must remind clients wanting to use complementary and integrative health therapies to check with their primary health care provider before using them.Acupuncture, calcium supplements, and yoga are not common alternative therapies for BPH.

Four women phone the gynecology clinic about having new-onset vaginal bleeding. Which call does the RN decide to return first? A) A 23-year-old using medroxyprogesterone acetate (Depo-Provera) B) A 34-year-old with a history of multiple leiomyomas C) A 48-year-old who had an endocervical curettage yesterday D) A 62-year-old with no previous gynecologic problems

Answer: D. Vaginal bleeding in a postmenopausal woman is abnormal and may be an indication of serious problems such as endometrial cancer.Bleeding in the 23-year-old using medroxyprogesterone acetate (Depo-Provera), the 34-year-old with a history of multiple leiomyomas, and the 48-year-old who had endocervical curettage yesterday is not unusual. The nurse will need to follow up with these clients.

A patient has undergone external beam radiation therapy (EBRT) for palliative treatment of prostate cancer. What suggestions does the nurse make to help the patient manage acute radiation cystitis secondary to EBRT? a. Limit intake of water and other fluids. b. Avoid consumption of caffeinated drinks. c. Increase consumption of dairy products. d. Wash genitals with mild soap and water.

B

If the function of normal vaginal flora is disrupted, the female patient is most likely to experience which condition? a. Vaginal dryness b. Vaginal infection c. "Hot flashes" d. Irregular menses

B

Which comment from a family member would be most strongly associated with the term "natal sex"? a. "My little girl likes to play with her brother's toys and she admires her brother." B. "My brother just called me. His wife delivered a healthy baby girl this morning." C. "My brother seems a bit effeminate, but I wouldn't call him transgender." D. "My sister is unsure if she is sexually attracted to men or to women."

B

Which factor is the incidence of breast cancer most closely related to? a. Lifestyle choices b. Aging c. Ethnic background d. Socioeconomic status

B

Which laboratory result indicates that the pri- mary goal of treatment of the patient's uterine fibroids has been successful? a. Vaginal smears show no bacterial growth. b. Red blood cell count is within normal limits. c. Human chorionic gonadotropin is negative. d. White blood cell count is within normal limits.

B

Which man has the greatest risk for developing prostate cancer? a. Patient's grandfather, age 82 years, has benign prostatic hyperplasia. b. Patient's father was diagnosed and treated for prostate cancer at age 50. c. Patient's mother took diethylstilbestrol to control bleeding during pregnancy. d. Patient's brother had delayed development of sexual characteristics.

B

Which patient is most likely to report problems with impotence? a. Patient has prostate specific antigen of 2 ng/mL. b. Patient has diabetes mellitus. c. Patient has a history of orchitis. d. Patient has testosterone of 500 ng/dL.

B

Which patient is most likely to require an iron supplement? a. 53-year-old woman who is entering menopause and has a breast mass b. 32-year-old female with heavy menstrual bleeding and an intrauterine device c. 70-year-old male who is diagnosed with benign prostatic hyperplasia d. 23-year-old woman who has pelvic inflammatory disease

B

Which sexually transmitted disease is associated with an increased risk for cervical cancer? a. Syphilis caused by Treponema pallidum b. Condylomata acuminata caused by HPV type 16 c. Condylomata lata caused by Treponema pallidum d. Chlamydia caused by Chlamydia trachomatis

B

Which sign/symptom is associated with ad- vanced prostate cancer? a. Difficulty starting urination b. Swollen lymph nodes c. Frequent bladder infections d. Erectile dysfunction

B

A patient who had a total abdominal hysterec- tomy is anxious to resume her activities because she has young children at home. What postprocedure information does the nurse provide to the patient? Select all that apply. a. Climb stairs to build strength and endurance. b. Avoid sitting for prolonged periods. c. Do not lift anything heavier than 5-10 lbs. d. Walk or jog at least 1-2 miles every day. e. When sitting, do not cross the legs. f. Resume regular household chores and activities.

B,C,E

A patient with a fibrocystic breast condition has just undergone fine needle aspiration to drain the cyst fluid and reduce pressure and pain. When would the nurse prepare patient education material about breast biopsy? Select all that apply. A. If hormonal replacement therapy is prescribed b. If fluid is not aspirated c. If the mammogram shows suspicious findings d. If fluid buildup recurs e.If the mass remains palpable after aspiration f. If aspirated fluid reveals cancer cells

B,C,E,F

_____: Surgical removal of both fallopian tubes and both ovaries.

Bilateral salpingo-oophorectomy (BSO)

A 22-year-old woman is being seen for a self- detected mass in her right breast. Clinical examination reveals an oval-shaped, freely mobile, and rubbery lesion. What type of tumor is this most likely to be? A. Ductal ectasia b. Papilloma c. Fibroadenoma d. Macrycyst

C

A 54-year-old woman has identified a hard breast mass with irregular borders, redness, and edema. She reports a nipple discharge and enlarged axillary nodes. Based on the patient's age and description of the symptoms, what does the nurse suspect? A. Fibroadenoma b. Fibrocystic breast condition c. Ductal ectasia d. Intraductal papilloma

C

A patient had podophyllin treatment for Con- dylomata acuminata. For which sign/symptom does the nurse tell the patient to return for further treatment? a. Discomfort at the site b. Bleeding from the site c. Infection at the site d. Sloughing of parts of warts

C

The nurse is teaching a patient who had an open radical prostatectomy about how to manage the common potential long-term complications. What does the nurse teach the patient? a. How to perform testicular self- examination b. How to manage a permanent suprapubic catheter c. How to perform Kegel perineal exercises d. How to use dietary modifications to acidify the urine

C

The nurse is teaching a patient who received treatment for genital herpes. Which patient statement indicates a need for further teaching? A. "I can be contagious even when I do not have any lesions." B. "If I get pregnant, I need to tell my nurse midwife that I have genital herpes." C. "After taking acyclovir, I won't have to worry about exposing my partner." D. "I need an annual Pap smear because of my increased risk for cervical cancer."

C

The nurse is preparing an information packet about women's health considerations for sexually transmitted diseases (STDs). What information does the nurse include? Select all that apply. A. Young women generally have excellent knowledge about the risk of STD's. b. Young women frequently believe that they are vulnerable to STDs. C. Young women mistakenly believe that contraceptives protect the from STDs. D. Mucosal tears in postmenopausal women may also place them at greater risk for STDs. E. Women have more asymptomatic infections that may delay diagnosis and treatment. F. Young women who alcohol binge are more prone to risky behavior.

C,D,E,F

The nurse teaches a patient with benign pros- tatic hyperplasia to follow which instructions? Select all that apply. a. Take diuretics to increase output b. Avoid sexual intercourse. c. Avoid antihistamines. d. Avoid caffeine. e. Avoid drinking large amounts of fluid in a short time. f. Void when the urge occurs.

C,D,E,F

The nurse is teaching a group of high school students about the use of condoms. What information does the nurse include? Select all that apply. a. Use spermicide (nonoxynol-9) with condoms. b. Do not used female condoms; they are too difficult for inexperienced partners. c. Keep condoms (especially latex) in a cool, dry place out of direct sunlight. d. Do not use condoms if package is damaged or if they are brittle or discolored. e. Put condoms on the penis before foreplay or arousal. f. Make sure that lubricants are water-based.

C,D,F

A patient is prescribed leuprolide, a luteinizing hormone-releasing hormone agonist, for treatment of a prostate tumor. What possible side effect of this medication does the nurse advise the patient about? a. Nipple discharge b. Scrotal enlargement c. Fragility of the skin d. Erectile dysfunction

D

_____: A solid, slowly enlarging benign mass of connective tissue that is unattached to the surrounding breast tissue and is typically discovered by the patient herself. The mass is usually round, firm, easily movable, nontender, and clearly delineated from the surrounding tissue.

Fibroadenoma

_____: A test that can identify many high-risk types of HPV associated with the development of cervical cancer.

Human papilloma virus (HPV) test

_____: Diagnostic procedure/treatment in which a thin loop-wire electrode that transmits a painless electrical current is used to cut away affected cervical cancer tissue.

Loop electrosurgical excision procedure (LEEP)

_____: A cytologic study that is effective in detecting precancerous and cancerous cells obtained from the cervix.

Papanicolaou test (Pap test; Pap smear)

_____: Removal of the uterus and cervix; the procedure may be vaginal or abdominal.

Total hysterectomy

_____: Treatment for leiomyomas in which a radiologist uses a percutaneous catheter inserted through the femoral artery to inject polyvinyl alcohol pellets into the uterine artery. The resulting blockage starves the tumor of circulation, allowing it (or them) to shrink.

Uterine artery embolization

_____: The construction of a new vagina in a male-to-female patient, usually with inverted penile tissue or a colon graft, and the creation of a clitoris and labia using scrotal or penile tissue and skin grafts.

Vaginoplasty

_____: A longitudinal area between the labia minora, the clitoris, and the vagina that contains Bartholin glands and the openings of the urethra, Skene's glands (paraurethral glands), and vagina.

Vestibule

_____: Inflammation of the lower genital tract resulting from a disturbance of the balance of hormones and flora in the vagina and vulva.

Vulvovaginitis

A patient is lying in bed after a mastectomy. How does the nurse position the patient? a. Head of the bed up at least 30 degrees with the affected arm elevated on a pillow b. Supine body position with the affected arm positioned straight by the side c. Any position that is the most comfortable to the patient d. Side-lying position with the unaffected side down toward mattress

A

A patient is prescribed trastuzumab for breast cancer. What is the priority nursing interven- tion for this patient? a. Assess for signs of cardiotoxicity b. Assess for signs of bleeding. c. Premedicate with an antiemetic. d. Rotate injection sites

A

A patient is undergoing large-volume bladder irrigation. During and after the procedure, the nurse observes the patient for confusion, mus- cle weakness, and increased gastrointestinal motility related to which potentially adverse effect of large-volume irrigation? a. Hyponatremia b. Hypovolemia c. Hypokalemia d. Hypotension

A

A patient needs surgical intervention for an enlarged prostate but also needs to maintain his anticoagulant therapy. Which brochure would be the most appropriate to prepare for the patient? a. "Talking to Your Doctor About Holmium Laser Enucleation of the Prostate" b. "Transurethral Resection of the Prostate: The Traditional Treatment." c. "Is Laparoscopic Radical Prostatectomy with Robotic Assistance Right for You?" d. "Common Questions About the Open Surgical Technique for Radical Prostatectomy."

A

A patient who had surgery for breast cancer appears in need of continued community support. The nurse refers the patient to which organization? a. Reach to Recovery b. Empty Arms c. Resolve to Reach d. Evolve

A

A patient with a fever, myalgia, sore throat, and sunburn-like rash is admitted with the diagnosis of toxic shock syndrome. What additional clinical manifestation should the nurse assess for? a. Hypotension b. Vaginal bleeding c. Bradycardia d. Polyuria

A

A patient with a fibrocystic breast condition is prescribed hormonal therapy to manage symptoms. Which teaching materials will the nurse prepare for the patient related to the therapy? a. Signs/symptoms of stroke, liver disease, and increased intracranial pressure b. Signs/symptoms of acute renal failure, fluid retention, or electrolyte imbalances c. Signs/symptoms of problems with coagulation or other hematologic disorders d. Signs/symptoms of cardiotoxicity, cardiac failure, or fluid overload

A

A young woman reports that she has a genital discharge causing irritation and odor. She feels embarrassed, but insists that she has not had recent sexual relations. Which question is the nurse most likely to ask? a. "Have you recently taken any antibiotic medications?" b. "Do you have a family history for cervical cancer?" c. "How old were you when you first had sexual intercourse?" d. "Have you had a change of diet or noticed weight loss?"

A

An older patient is scheduled for an annual physical including a prostate-specific antigen (PSA) and a digital rectal examination (DRE). How are these two tests scheduled for the patient? a. PSA is drawn before the DRE is performed. b. DRE is done several weeks before the PSA. c. PSA is reviewed first because DRE may be unnecessary. d. Both tests can be done at the convenience of the patient.

A

An older patient's wife is very upset because "my husband was just told that he had prostate cancer. He feels fine now, but the doctor told him to watch and wait. Why are we just watching? What are we supposed to do?" What is the nurse's best response? a. "Prostate cancer is slow growing. Your husband needs regular prostate specific antigen testing; I'll give you a list of symptoms to watch for." b. "This is very upsetting news. Let's sit down and talk about how you feel and then I will have the doctor talk to you again." c. "It's okay, don't be upset. This is a very common way to handle prostate cancer for men who are your husband's age." d. "I can get you some information about prostate cancer. This will help you understand why the doctor said this to your husband."

A

Based on risk factors and personal history, which woman has the greatest risk of develop- ing breast cancer? a. Physician, age 56, who had her first child at age 38 b. Ballet dancer, age 25, who has a 5-year-old son c. Radiation technician, age 24, who had her menarche at age 13 d. Housewife, age 42, who had breast reduction surgery at age 26

A

During clinical breast exam, the examiner observes a small mass in the breast. What is the most important item to include in the documentation of this finding? a. "Face of the clock" location of the mass b. Amount of pressure required to detect the mass c. Patient's self-awareness of the location d. Method used to examine the breast

A

For women with genetic risk factors for breast cancer, which intervention would address one of the modifiable risk factors? a. Discuss strategies to avoid weight gain and obesity. b. Encourage frequent genetic testing for tumors. c. Have testing for BRCA1 and BRCA2 gene mutations. d. Consider hormone replacement therapy.

A

The advanced practice nurse is performing a tes- ticular exam on a young Caucasian male patient. The practitioner finds a hard, painless lump. This finding is considered the most common mani- festation of which disease or disorder? a. Testicular cancer b. Erectile dysfunction c. Prostate cancer d. Epididymitis

A

The health care provider tells the nurse that the patient is being evaluated for galactorrhea and to please call with the relevant laboratory results. Which laboratory result will the nurse look for? a. Prolactin level b. Endometrial biopsy results c. Follicle-stimulating hormone level d. Progesterone level

A

The home health nurse is reviewing the patient's medication list and sees that the patient was given doxorubicin at the hospi- tal. What gynecologic diagnosis would the nurse expect to see as part of the patient's history? a. Endometrial cancer b. Cervical polyps c. Endometriosis d. Dysfunctional uterine bleeding

A

The mother of a 17-year-old adolescent girl tells the nurse that her daughter has been purging, showing anorexic behavior, and continuously exercising. Based on the mother's report, which question related to the reproductive system would the nurse ask the girl? a. "When was your last normal menstrual period?" b. "Are you sexually active?" c. "Are you having any unusual vaginal discharge?" d. "Do you have any questions about contraception?"

A

The nurse is caring for a patient who had a posterior colporrhaphy. Which task is most appropriate to delegate to unlicensed assistive personnel? a. Assist the patient with a sitz bath to relieve discomfort b. Assist the patient to select menu items that are low residue. c. Change the bed linens every 4 hours or more as needed. d. Supervise and teach the patient to turn every 2 hours.

A

The nurse is caring for a patient who had hysteroscopic surgery. The patient reports severe lower abdominal pain, appears pale, and has trouble focusing on the nurse's questions about the pain. Vital signs show T 98.6°F, P 120/min, R 24/min, BP 103/60. Which complication does the nurse suspect? a. Hemorrhage b. Embolism c. Fluid overload d. Incomplete suppression of menstruation

A

The nurse is caring for a patient who is diagnosed with ductal ectasia. What is the primary goal of the nursing care? a. Reduce the anxiety associated with the threat of breast cancer. b. Review side effects of radiation and chemotherapy treatments. c. Give support for decision-making about prophylactic mastectomy. d. Assess for readiness and willingness to seek a support group.

A

The nurse is designing a teaching plan for a patient with an enlarged prostate with obstructive symptoms. What action could the patient perform that might help to relieve the obstruction? a. Increase frequency of sexual intercourse. b. Void before going to bed and upon waking. c. Urinate forcefully after drinking fluids. d. Spread fluid intake throughout the day.

A

The nurse is giving discharge instructions to a patient who had a transurethral resection of the prostate. What does the nurse include in the instructions? a. Reassurance that loss of control of urina- tion or dribbling of urine is temporary b. Instructions about how to apply a condom catheter and monitor for skin breakdown c. Advice about how to control bleeding and passage of blood clots d. Information about the side effects related to aminocaproic acid

A

The nurse is preparing patient teaching for several young women who will undergo surgical procedures for gynecologic problems. Which surgical procedure is most likely to induce menopausal symptoms? a. Bilateral salpingo-oophorectomy b. Radioablation c. Uterine artery embolization d. Hysteroscopic myomectomy

A

The nurse is teaching a class about laparoscopic or hysteroscopic myomectomy. Which patient should be invited to attend the teaching session? a. 23-year-old woman with uterine fibroids who would like to have children in the future b. 65-year-old woman who was diagnosed with uterine cancer but also has bleeding disorder. C. 43-year-old African American woman with early diagnosed endometrial cancer. D. 73-year-old woman with urinary incontinence secondary to a cystocele

A

The nurse is teaching a patient about the contraindications for hysteroscopy. What does the nurse tell the patient? a. "During the procedure, normal or abnormal cells can be pushed through the fallopian tubes and into the pelvic cavity; therefore pregnancy is a contraindication." b. "The procedure causes irritation and can be very painful if your vaginal tissue is dry or fragile; therefore, the procedure is not recommended for postmenopausal women." c. "The procedure can cause a lot of bleeding so a recent prescription of an anticoagu- lant is a contraindication." d. "During the procedure, an iodine-based dye is used, so allergies to shellfish or iodine are contraindications."

A

The nurse sees that John Smith, natal sex male, has an order for insertion of an indwelling urinary catheter for hourly measurements of urine output. However, when the nurse enters the room, the patient appears to be female. What should the nurse do first? a. Introduce self, verify patient's identity by checking name band, and explain the procedure for catheterization. b. Inspect the genitalia and adapt catheter insertion as appropriate while avoiding use of gender-specific language. c. Respectfully address the patient as Mr. Smith, and perform catheter insertion for a male patient. d. Politely excuse self and obtain advice from the charge nurse about whether to treat the patient as a male or a female.

A

The patient had a transurethral resection of the prostate (TURP) several days ago, and the urinary catheter was removed 6 hours ago. Which sign/symptom must be resolved before the patient is discharged? a. Patient has not voided since the catheter was removed. b. Patient reports a burning sensation with urination. c. Patient reports dribbling and leakage since catheter was removed. d. Patient reports anxiety related to sexual function because of TURP.

A

The patient is prescribed a broad-spectrum antibiotic for prostatitis. Which laboratory re- sult indicates that the medication is having the desired therapeutic effect? a. Normalization of white cell count b. Decreased blood urea nitrogen level c. Increased red blood cell count d. Prostate-specific antigen within normal limits

A

The patient needs diagnostic testing to deter- mine the presence of endometrial thickening and possible cancer. Which brochure will the nurse prepare for the patient? a. "How Transvaginal Ultrasound and Endo- metrial Biopsy Are Used in Cancer Diag- nosis." b. "The Role of Abdominal Ultrasound and Magnetic Resonance Imaging in Cancer Diagnosis." c. "Advances in the Diagnosis of Cancer Using Computed Tomography and Cystography." d. "What the Presence of BRCA1 or BRCA2 Gene Mutations Means in the Diagnosis of Cancer."

A

Which combination of screening techniques is best for early detection of breast cancer? a. Mammogram, clinical breast exam, and breast self-awareness b. Magnetic resonance imaging, breast self- examination, and mammogram c. Breast biopsy, clinical breast exam, and mammogram. D. Mammogram, breast self-examination, and breast tomosynthesis

A

Which patient circumstance is most appropriate for expedited partner treatment? a. Partner is afraid to come to health care facility. b. Partners are clients of a commercial sex worker. c. Patient and partner are both HIV positive. d. Patient and partner have multiple other partners.

A

Which patient has the greatest need for evaluation of possible endometrial cancer? a. 63-year-old woman who is having painless vaginal bleeding b. 33-year-old woman who reports a past history of multiple sex partners. c. 23-year-old woman who has not had a menstrual period for 3 months d. 52-year-old woman who is having irregular menses for 3 months

A

Which patient is describing an ominous sign associated with inflammatory breast cancer, which is a highly aggressive invasive breast cancer? a. Patient reports breast pain and a rapidly growing breast lump. b. Patient notices numerous small tender lumps before menses. c. Patient says her breasts feel similar to when she breastfed. d. Patient reports noticing a change in the color of her nipple.

A

Which patient should be advised to have an annual clinical breast examination? a. 42-year-old woman who is not having any symptoms b. 25-year-old woman who is nursing a newborn c. 35-year-old woman with no known health problems d. 28-year-old woman who recently had breast reduction.

A

Which therapies would the nurse expect to use for a patient who is being treated for a rectocele? a. High-fiber diet, stool softeners, and laxatives b. Intravaginal estrogen and a pessary c. Oral contraceptives and antibiotics d. Doxorubicin and cisplatin

A

Which woman has the greatest risk for pelvic inflammatory disease? A. 24-year-old who smokes and has multiple sexual partners b. 62-year-old who uses topical estrogen therapy for dyspareunia c. 35-year-old lesbian who recently entered a new relationship d. 45-year-old whose husband was treated for sexually transmitted disease

A

Which factors increase the risk for pelvic in- flammatory disease? Select all that apply. a. Age younger than 26 years b. Multiple sexual partners c. Smoking d. Caffeine use e. History of chlamydia or gonococcal infection f. Intrauterine device placed within the previous 3 weeks

A,B,C,EF

The nurse is interviewing a patient who reports a discharge from his penis that started 3 days ago. What does the nurse ask the patient regarding this problem? Select all that apply. a. "Has your sexual partner(s) noticed a discharge?" b. "Does the discharge have an odor?" c. "Have you noticed a mass or lump in the scrotum?" d. "Are you having any pain or burning during urination?" e. "What is the consistency of the discharge?" f. "Can you describe the color of the discharge?"

A,B,D,E,F

Regarding the diagnosis and treatment of breast cancer, what are important consider- ations for young women? Select all that apply. a. Genetic predisposition is a stronger risk factor for young women compared to older women b. Young women frequently have more aggressive forms of the disease. c. Prognosis is usually better for young women because they have fewer chronic conditions. d. Screening tools can be less effective because young women have denser breast tissue. e. Young women are less likely to be concerned about cancer, so they delay seeking treatment. F. Early menopause, infertility, and sexual dysfunction are concerns for young women

A,B,D,F

The nurse is designing a teaching plan for a patient who had surgery for breast cancer. What information does the nurse include in the plan? Select all that apply. a. Do not use lotions or ointments on the area. b. Delay using deodorant under the affected arm until healing is complete. c. Swelling and redness of the scar itself are considered normal and permanent. d. Report any increased heat and tenderness of the area to the surgeon. e. Wear loose pajamas at home for 6-8 weeks. f. Begin active range-of-motion exercises 1 week after surgery.

A,B,D,F

Which conditions meet the criteria for a having a surgical intervention for benign prostatic hyperplasia? Select all that apply. A. Acute urinary retention b. Hydronephrosis c. Acute urinary tract infection that does not respond to first-line antibiotics d. Recurrent kidney stones e. Hematuria f. Chronic urinary tract infections secondary to residual urine in the bladder

A,B,E,F

The nurse is doing the preoperative care for an male-to-female patient who will undergo a vaginoplasty. What is included in the care for this patient? Select all that apply. a. Administer an enema and laxatives as ordered. b. Give nothing by mouth for 24 hours prior to surgery. c. Monitor and report hemoglobin and hematocrit results. d. Administer preoperative antimicrobials to minimize infection. e. Monitor and record the drainage from Jackson-Pratt drain. f. Instruct to ambulate, because positioning for surgery will be prolonged

A,C,D

The nurse is teaching the patient about breast self-examination. Which correct actions are included in the procedure? Select all that apply. a. "Lie down on your back and place your right arm behind your head." b. "Use the palm of your left hand to feel for lumps in the right breast." c. "Use three different levels of pressure to feel all the breast tissue." d. "Move around the breast in an up and down pattern." e. "Repeat the exam on your left breast." f. "Stand in front of a mirror, press your hands firmly down on your hips, and ob- serve breasts."

A,C,D,E,F

The patient reports itching, change in vaginal discharge, and an odor. The nurse suspects that the patient has vulvovaginitis. Based on knowledge about the common causes of vulvovaginitis, which questions would the nurse ask? Select all that apply. a. "Have you recently been taking antibiotics?" b. "Have you been swimming in a lake or pond?" c. "Do you consistently wipe from front to back?" d. "Do you use tampons?" e. "Do you douche or use vaginal sprays?" f. "Have you had problems with vaginal yeast infections?"

A,C,D,E,F

Which sexually transmitted diseases are reportable to the local health authorities in every state? Select all that apply. a. Chlamydia b. Genital herpes c. Gonorrhea d. Syphilis e. Chancroid f. Human immunodeficiency virus

A,C,D,E,F

The nurse is taking a health history on a patient with organic erectile dysfunction. What are possi- ble causes of this condition? Select all that apply. a. Medications for hypertension b. Obesity c. Thyroid disorders d. Diabetes mellitus e. Diverticulitis f. Smoking and alcohol

A,C,D,F

The nurse is teaching a patient who is being discharged after having a total abdominal hysterectomy. Which conditions does the nurse tell the patient to immediately report to the surgeon? Select all that apply. a. Vaginal drainage that becomes thicker or foul-smelling b. Hot flashes and night sweats c. Temperature over 100°F (38°C) d. Burning during urination e. Feeling more tired and sleeping longer f. Pain, tenderness, redness, or swelling in calves

A,C,D,F

A male patient reports that he has a decreased libido. The nurse assesses for which factors related to this condition? Select all that apply. A. Tobacco use b. Type of exercise c. Alcohol consumption d. Occupation e. Illicit drug use f. Weight gain

A,C,E

The nurse is talking to a group of college stu- dents about safer sex practices. What recom- mendations are included as safer sex practices? Select all that apply. a. Use a latex or polyurethane condom for genital and anal intercourse. b. Wash hands immediately after hand con- tact with vagina or rectum. c. Practice abstinence. d. Practice serial monogamy. e. Decrease the number of sexual partners. f. Men should not have sex with men.

A,C,E

In a patient with a reproductive health problem, what health and lifestyle habits would the nurse assess? Select all that apply. a. Diet b. Socioeconomic status c. Exercise pattern d. Occupation e. Sleep pattern f. Sexual practices

A,C,E,F

The surgical procedure for stage I disease of endometrial cancer involves removal of which components? Select all that apply. a. Uterus b. Vagina c. Fallopian tubes d. Rectum e. Ovaries f. Peritoneum fluid for cytologic examination

A,C,E,F

Following a uterine embolization using a vas- cular closure device, what patient care would the nurse provide? Select all that apply. a. Assist the patient to ambulate 2 hours after the procedure. b. Keep the patient on bedrest with the leg immobilized for 4 hours before ambulating. c. Encourage the patient to drink a lot of fluids. d. Assess for constipation and administer laxatives as needed. e. Raise the head of the bed. f. Assess pain level and provide analgesics as needed.

A,E,F

Which assessment findings will the nurse expect for a client who is suspected of having pelvic inflammatory disease? (Select all that apply.) A) Vaginal bleeding B) Fever C) Lower abdominal pain D) Constipation E) Rigid abdomen

ANS: A, B, C, E. Assessment findings B, C, and E are consistent with any abdominal infection and are common for the woman with PID. In addition, invasion of the pathogen into the reproductive organs can cause vaginal bleeding (A). Constipation is not a common assessment finding for the woman with PID (C).

The nurse provides health teaching for a client receiving estrogen therapy. Which statement by the client indicates that the teaching was effective? (Select all that apply.) A) "I need to check my blood pressure frequently when taking this drug." B) "I will call my doctor if I have any redness or swelling in my legs." C) "I will drink extra fluids because this drug will cause me to urinate a lot." D) "I know that the drug will cause my breast to feel tender." E) "I can get frequent headaches from taking this drug."

ANS: A, B, D, E. Estrogen supplementation can cause side effects such as headache, breast tenderness, nausea/vomiting, and weight gain (often due to fluid retention) or loss (D and E), and can lead to complications, such as deep vein thrombosis (DVT) and estrogen-dependent cancers. Redness and swelling are classic signs of DVT (B); fluid retention can cause increased blood pressure (A). Therefore, the answer is A, B, D, and E.

A client is admitted to the postanesthesia care unit (PACU) following a vaginoplasty. For which surgical complications will the nurse monitor in the PACU? (Select all that apply.) A) Bleeding B) Rectal perforation C) Urinary incontinence D) Surgical site infection E) Urinary retention

ANS: A, B. The client has just been admitted to the PACU from the OR and will have a urinary catheter. Therefore, choices C and D are not correct answers. Wound infections typically occur several days after surgery, so D is also not correct. However, immediately after surgery in the PACU, the nurse will monitor for bleeding and signs of rectal perforation (A and B). The client with rectal perforation will likely have severe rectal pain and bleeding or drainage.

The nurse is assigned to care for a client who has undergone a modified radical right mastectomy for breast cancer. When delegating care, which statements by the nursing assistant would require further teaching by the nurse? (Select all that apply.) A) "I will irrigate the drainage tube after I empty it." B) "If the client says she is in pain, I will tell you right away." C) "It is important for me to take blood pressure on the client's right arm." D) "When helping the client walk, I'll remind her to stand straight." E) "I'll let you know if her surgical dressing is intact and dry."

ANS: A, C, E. Nursing assistants are unlicensed assistive personnel, and therefore do not perform nursing assessments, as required in option E. Drainage tubes should not be irrigated without a prescription from the surgeon; only licensed nurses may irrigate any type of drain. In addition, surgical drains are not typically irrigated. Therefore, option A is beyond the scope of UAP. The blood pressure can be taken by the nursing assistant, but not in the operative arm (right) (option C). Therefore, the assistant requires further teaching about A, C, and E. The other two options are within the scope of the role of UAP and are appropriate for a post-operative mastectomy patient.

A client had an orchiectomy and laparoscopic radical retroperitoneal lymph node dissection this morning. What assessment finding will the nurse report to the primary health care provider? A) The client's pain level is a 10 on a 0 to 10 pain scale after pain medication. B) The client's urinary catheter is draining clear yellow urine. C) The client is concerned about his ability to have an erection. D) The client's blood pressure is 132/82 in a sitting position.

ANS: A. Choices B, C, and D are expected postoperative assessments for a client who had these surgeries. Therefore, the appropriate finding to be reported to the primary health care provider is his pain level of 10 that continues after receiving pain medication.

A client returns from surgery after a laparoscopic total abdominal hysterectomy. Upon initial assessment, which finding by the nurse requires immediate intervention? A) Decreased bowel sounds in all quadrants B) Heavy vaginal bleeding with clots C) Temperature of 99°F (37.2°C) D) Client statement that pain is 4 on a scale of 0 to 10

ANS: B. In the immediate postoperative period, the client typically does not have severe pain, but may have a slight temperature elevation due to inflammation from the surgical procedure (C and D). Bowel sounds may be decreased from the surgery due to anesthesia and manipulation during the procedure (A). Vaginal bleeding should be scant or minimal because this procedure is abdominal, not vaginal. Therefore, the nurse will report heaving bleeding with clots (choice B).

A client had an anterior and posterior colporrhaphy procedure this morning. What is the nurse's current priority assessment when caring for this client? A) Monitoring for urinary incontinence B) Determining pain level C) Checking for bowel sounds D) Inspecting the sternal incision

ANS: B. The client who had an A and P repair has an indwelling urinary catheter in the immediate postoperative period. Therefore, choice A is not the correct response. There is no sternal incision for this procedure, so choice D is also not correct. Nursing assessments in the immediate postoperative period include A and C. However, the priority assessment is to determine the client's pain level. This surgery is very painful due to its perineal location where multiple nerves are located.

A client has undergone a breast biopsy. Which post-procedure symptoms will the nurse teach the client to report immediately to the primary health care provider? A) Tenderness around the biopsy site B) Numbness around the biopsy site C) Heavy bleeding from the biopsy site D) Slight edema near the biopsy site

ANS: C. After a breast biopsy, it is common for the client to experience tenderness, slight swelling (edema), and numbness around the biopsy site (A, B, D). Therefore, there is no need to report these findings to the primary health care provider. Heavy bleeding is not typical after a breast biopsy and it should therefore be reported (C).

A client had a transurethral resection of the prostate (TURP) with continuous bladder irrigation yesterday. The staff nurse notes that the urinary drainage is pink-tinged and clear. What is the nurse's best action? A) Notify the charge nurse as soon as possible. B) Increase the rate of the bladder irrigation. C) Document the assessment in the medical record. D) Prepare the patient for a blood transfusion.

ANS: C. The CBI after a TURP should run at a rate to ensure that the urinary drainage is pink-tinged and clear, rather than red or darker. Therefore, this finding for the client is expected and requires documentation (C). No other action is needed.

The nurse gives a client an IM dose of penicillin G for primary syphilis. Which client statement indicates a need for further teaching? A) "I will wait in the clinic for 30 minutes to be sure I do not have a reaction." B) "When I get home, I will call my partner to tell him/her about my diagnosis." C) "If I have sex with someone, I do not have to worry about spreading the disease." D) "I plan to return to see my primary care provider for follow-up in 6, 12, and 24 months."

ANS: C. The client with syphilis needs to inform any partner(s) about having the disease so that the partner can be evaluated. Follow-up is very important to assure that the disease has been adequately treated. After receiving a penicillin dose, the client should be carefully monitored for an allergic reaction. Therefore, client statements A, B, and D are accurate and would not require additional teaching. The correct answer to this question is C because the client believes that the disease will not be communicable during sex, which is NOT an accurate statement.

The nurse is teaching a client about taking sildenafil for erectile dysfunction. Which statement by the client indicates a need for further teaching? A) "I should have sex within an hour after taking the drug." B) "I should avoid alcohol when on the drug or it might not work well." C) "I can expect to maybe feel flushed or get a headache when I take the drug." D) "If I have chest pain during sex, I should take a nitroglycerin tablet."

ANS: D. All of the choices except D are correct information about sildenafil. However, nitroglycerin should be avoided when taking this drug because severe hypotension can occur. Teach the patient to call 911 if chest pain occurs. Therefore, choice D is incorrect information, which requires further teaching

A 70-year-old client asks the nurse if she needs a Pap smear. Her last Pap smear was 3 years ago and it was normal. What is the correct nursing response? A) "Yes, you need a Pap test this year." B) "You are not due for another Pap test until next year." C) "A Pap smear is not needed unless you are sexually active." D) "You may not need an annual Pap smear after 65 years of age."

ANS: D. The current recommendation from the American Cancer Society is to stop Pap testing at 65 years of age. In Canada, the recommendation is to stop this test at 70. Therefore, the correct answer is D. If the woman has had cervical cancer, these recommendations may be altered.

The nurse is caring for four clients. Which client does the nurse recognize as having the highest risk for development of breast cancer? A) 55-year-old male with gynecomastia and obesity B) 60-year-old female whose father died from colon cancer C) 65-year-old male whose mother had ovarian cancer D) 75-year-old female who was treated for breast cancer 5 years ago

ANS: D. Women are far more at risk for breast cancer than men, so A and C are incorrect. Depending on the type of breast cancer, a woman with a prior history who was treated is at a high risk for recurrence as either breast cancer or metastasis. Therefore, D is the correct response.

The nurse is assessing the client diagnosed with a rectocele. Which signs and symptoms should the nurse expect? Select all that apply. 1. Rectal pressure. 2. Flatus. 3. Fecal incontinence. 4. Constipation. 5. Urinary frequency.

ANSWER: 1, 2, 3, 4. 1. A rectocele causes the rectum to be pouched upward, causing rectal pressure. 2. When the rectum pushes against the posterior wall of the vagina, the result is flatus. 3. Clients with a rectocele experience fecal incontinence. 4. Clients with a rectocele frequently are constipated. 5. A client who has a rectocele does not experience urinary frequency. TEST-TAKING HINT: The test taker should learn the terms. If the test taker does not know the answer, the test taker can eliminate urinary frequency because it is different from the other options. It refers to the urinary tract, whereas the others refer to bowel elimination

The client diagnosed with cancer of the uterus is scheduled to have radiation brachytherapy. Which precautions should the nurse implement? Select all that apply. 1. Place the client in a private room. 2. Wear a dosimeter when entering the room. 3. Encourage visitors to come and stay with the client. 4. Plan to spend extended time with the client. 5. Notify the nuclear medicine technician.

ANSWER: 1, 2, 5. 1. Brachytherapy is the direct implantation of radioactive seeds through the vagina into the uterus. The client should be in a private room at the end of the hall to prevent radiation exposure to the rest of the unit. 2. Nurses wear a dosimeter registering the amount of radiation they have been exposed to. When a certain level is reached, the nurse is no longer allowed to care for clients undergoing internal radiation therapy. 3. Visitors are limited while the radiation is in place. 4. In this case, spending extra time with a client is not done. The nurse does only what must be done and leaves the room. 5. The nuclear medicine technician will assist with the placement of the implants and will deliver the implants in a leadlined container. The technician will also scan any items (linens and wastes) leaving the room for radiation contamination. TEST-TAKING HINT: This is an alternate-type question, which requires the test taker to select more than one correct answer. The test taker must select all the correct answers to receive credit for the question.

The client is being discharged after a left wedge resection. Which discharge instructions should the nurse include? Select all that apply. 1. Notify the HCP of a temperature of 100°F. 2. Carry large purses and bundles with the right hand. 3. Do not go to church or anywhere with crowds. 4. Try to keep the arm as still as possible until seen by the HCP. 5. Have a mammogram of the right and left breasts yearly

ANSWER: 1, 2, 5. 1. It is a common instruction for any client who has had surgery to notify the HCP if a fever develops. This could indicate a postoperative infection. 2. The client who has had a mastectomy is at risk for lymphedema in the affected arm because the lymph nodes are removed during the surgery. The client should protect the arm from injury and carry heavy objects with the opposite arm. 3. The client can attend church services and large gatherings. This client had surgery, not chemotherapy, which would make the client immunosuppressed. 4. The client should be taught arm-climbing exercises before leaving the hospital to facilitate maintaining range of motion. 5. The client has developed a malignancy in one breast and is at a higher risk for developing another tumor in the remaining breast area. TEST-TAKING HINT: The test taker must determine if the option of keeping the arm still is recommended. Most postoperative recommendations require the client to move as much as possible.

Which should the nurse teach the client regarding Breast Health Awareness (BHA) according to the American Cancer Society (ACS). Select all that apply. 1. Women at high risk should talk to the HCP about when to have a mammogram. 2. Beginning at age 45, to have a yearly mammogram. 3. To perform a breast self-examination (BSE) bimonthly. 4. To get a sonogram of the breasts semiannually. 5. To have Magnetic Rresonance Imaging of the breasts every five (5) years

ANSWER: 1, 2. 1. Women at high risk for breast cancer should have the opportunity to have mammograms beginning earlier than recommended for routine screening. 2. According to 2015 guidelines, routine screening mammograms should begin at age 45 and be performed yearly until age 55. At age 55 mammograms should be performed biennially. 3. Beginning in their 20s, women should be told about the benefits and limitations of breast selfexamination (BSE). Even those who choose not to do BSE should be aware of how their breasts normally look and feel and report any new breast changes to a health professional as soon as they are found. Finding a breast change does not necessarily mean there is a cancer. BSE is performed monthly, not bimonthly. 4. MRIs are performed on women with special needs (breast implants, etc.) but are not performed on a routine basis. TEST-TAKING HINT: The test taker could eliminate option "3" based on the timing "bimonthly."

The nurse is caring for a client who is one (1) day postoperative hysterectomy for cancer of the ovary. Which nursing interventions should the nurse implement? Select all that apply. 1. Assess for calf enlargement and tenderness. 2. Turn, cough, and deep breathe every six (6) hours. 3. Assess pain on a 1-to-10 pain scale. 4. Apply sequential compression devices to legs. 5. Assess bowel sounds every four (4) hours.

ANSWER: 1, 3, 4, 5. 1. All clients who have had surgery are at risk for developing deep vein thrombosis (DVT), and an enlarged, tender calf is a sign of DVT. 2. The client should be turned and encouraged to cough and deep breathe at least every two (2) hours. 3. Clients who have had surgery should be assessed for pain on a pain scale and by observing for physiological markers indicating pain. 4. Sequential compression hose are used prophylactically to prevent deep vein thrombosis. 5. The client should be assessed for the return of bowel sounds. TEST-TAKING HINT: Option "2" has a time frame in it, and the test taker should ask if the time frame is correct for the intervention.

The Gravida 7 Para 6 client delivered a 9-pound 4-ounce infant two (2) hours ago. Which intervention is priority for the nurse to implement? 1. Assess the client's fundus every hour. 2. Assess the client's voiding pattern every shift. 3. Discuss birth control options with the client. 4. Discuss breastfeeding methods with the client.

ANSWER: 1. 1. A Gravida 7 Para 6 client has had seven (7) pregnancies and carried six (6) of those pregnancies to 20 weeks or longer. This woman is at risk of having the fundus remain boggy and not diminishing in size, resulting in excessive bleeding. The nurse should assess the fundus hourly. 2. Voiding should be assessed related to delivery time and position of the fundus, not just every 12 hours (a normal shift). 3. It is not priority immediately postdelivery to discuss birth control options. 4. If the client wishes to breastfeed her infant at this time, with her history she can perform this without teaching. TEST-TAKING HINT: The test taker could eliminate option "2" because of the words "every shift." Timing words such as "hourly," "every day," "every 2 hours" can make an option correct or incorrect. The stem of the question said the delivery was two (2) hours ago; this requires the nurse to determine which intervention should be performed immediately postdelivery

The client who is scheduled to have a breast biopsy with sentinel node dissection states, "I don't understand. What does a sentinel node biopsy do?" Which scientific rationale should the nurse use to base the response? 1. A dye is injected into the tumor and traced to determine spread of cells. 2. The surgeon removes the nodes that drain the diseased portion of the breast. 3. The nodes felt manually will be removed and sent to pathology. 4. A visual inspection of the lymph nodes will be made while the client is sleeping.

ANSWER: 1. 1. A sentinel node biopsy is a procedure in which a radioactive dye is injected into the tumor and then traced by instrumentation and color to try to identify the exact lymph nodes the tumor could have shed into. 2. This is the older procedure in which the surgeon removed the nodes thought to drain the tumor. There was no way of knowing if the surgeon was actually removing the affected nodes. 3. The purpose of the procedure is not to rely on guesswork in determining the extent of tumor involvement. 4. Microscopic disease cannot be seen by the naked eye. TEST-TAKING HINT: The test taker could eliminate options "3" and "4" if he or she were aware of the definition of sentinel, which means "to watch over as a sentry." This might lead the test taker to determine that specific areas would have to be identified.

The HCP orders cultures of the urethral urine, bladder urine, and prostatic fluid. Which instructions should the nurse teach to achieve the first two (2) specimens? 1. Collect the first 15 mL in one jar and then the next 50 mL in another. 2. Collect three (3) early-morning, clean voided urine specimens. 3. Collect the specimens after the HCP massages the prostate. 4. Collect a routine urine specimen for analysis

ANSWER: 1. 1. After cleansing the penis and retracting the foreskin (if present), the client voids the first 15 mL in a sterile specimen cup; this is the urethral urine specimen. Then, the client voids the next 50 to 75 mL into another sterile specimen cup; this is the bladder urine. If the client does not have acute prostatitis, the HCP can massage the prostate and collect prostate fluid for culture, but if fluid is not expressed with massage, the next urine is sent for analysis. 2. Early morning has no bearing on collecting prostatitis specimens. 3. This is the third specimen and is obtained by the HCP. 4. A routine specimen will not provide the information needed.

The client has a diagnosis of rule-out Paget's disease. Which test provides a definitive diagnosis of the disease? 1. A breast biopsy. 2. A diagnostic mammogram. 3. Ultrasound of the breast. 4. Magnetic resonance imaging.

ANSWER: 1. 1. Biopsy of the lesion is the only definitive test for Paget's disease, a form of breast cancer accounting for about 1% of all breast cancers. 2. Mammography is the only test that routinely screens for breast cancer, but a definitive diagnosis is made by tissue identification. 3. Ultrasound of the breasts can diagnose fluidfilled cysts. 4. Magnetic resonance imaging can be done to determine the extent of tumor involvement, but tissue identification is the definitive test for tumor diagnosis

The nurse is caring for a client with epididymitis secondary to a chlamydia infection. Which discharge instruction should the nurse discuss? 1. The sexual partner must be prescribed antibiotics. 2. Delay sexual intercourse for a minimum of three (3) months. 3. Expect the urine to have white clumps for one (1) to two (2) months. 4. Drainage from the scrotum is fine as long as there is no fever.

ANSWER: 1. 1. Chlamydia is a sexually transmitted disease usually silent in the male partner, but it can cause epididymitis. If both sexual partners are not treated, then the partner can reinfect the client. 2. Sexual intercourse can be resumed within a couple of weeks as long as both partners have been treated for the infection. 3. This indicates a urinary tract infection. A UTI is another cause of epididymitis. 4. The scrotum is a closed cavity. There should not be any drainage from the scrotum. If there is drainage, this could indicate a fistula and requires immediate notification of the HCP. TEST-TAKING HINT: The test taker should always look at the time frame provided in either the stem or the answer options as a clue to answering the question correctly. The test taker should look closely at any option that has a "month" time frame.

Which could be a complication of cryotherapy surgery for cancer of the prostate? 1. The urethra could become scarred and cause retention. 2. The client could have ejaculation difficulties and be impotent. 3. Bone marrow suppression could occur from the chemotherapy. 4. Chronic vomiting and diarrhea causing electrolyte imbalance could occur.

ANSWER: 1. 1. Cryotherapy involves placing freezing probes into the prostate to freeze the cancer cells. An indwelling catheter is placed into the urethra, and warm water is circulated through the catheter to try to prevent the urethra from freezing. If the urethra scars, then the lumen will constrict, causing retention of urine. 2. Ejaculation difficulties are caused by obstruction from tumor growth. 3. Bone marrow suppression is caused by radiation or chemotherapy. 4. Cryotherapy does not cause chronic vomiting or diarrhea. TEST-TAKING HINT: The test taker should dissect the word "cryotherapy." Cryo- means "to freeze," so cryotherapy indicates some form of cold therapy. Then the test taker should decide what cold therapy could cause to happen in the area being treated.

The nurse is planning the care of a client diagnosed with pelvic inflammatory disease secondary to an STD. Which collaborative diagnosis is appropriate for this client? 1. Risk for infertility. 2. Knowledge deficit. 3. Fluid volume deficit. 4. Noncompliance.

ANSWER: 1. 1. Determining and diagnosing the risk for infertility problems requires collaboration between the nurse and the HCP. 2. The nurse is required to teach a client. This is an independent action. 3. Fluid volume deficit is not an appropriate nursing diagnosis for this client. 4. Noncompliance is an independent nursing problem. TEST-TAKING HINT: The question requires the test taker to determine which are autonomous functions of the nurse. The nurse does not have the capability to prescribe fertility medications or treatments.

The client has undergone a bilateral orchiectomy for cancer of the prostate. Which intervention should the nurse implement? 1. Support the scrotal sac with a towel and apply ice. 2. Administer testosterone replacement hormone orally. 3. Encourage the client to place sperm in a sperm bank. 4. Have the client talk to another man with ejaculation dysfunction.

ANSWER: 1. 1. Elevating a surgical site and applying ice will reduce edema to the area. 2. The testes have been excised to remove the majority of male hormones. Replacing the hormones negates the purpose of the surgery. 3. Sperm banking is encouraged for younger men who want to have children in the future. Prostate cancer is a cancer diagnosed in older men, and sperm banking is not normally recommended. 4. Bilateral orchiectomy will render the client impotent but not with ejaculation dysfunction. TEST-TAKING HINT: The test taker should try to match the procedure to the answer option. The procedure removes hormone-producing ability, so option "2" could be eliminated because it reverses the effects of the procedure.

An elderly woman is diagnosed with pelvic relaxation disorder secondary to age-related changes. Which medication should the nurse expect to administer? 1. Estrogen, a hormone. 2. Cervidil, a cervical ripening agent. 3. Progesterone, a hormone. 4. Pitocin, an oxytocic agent.

ANSWER: 1. 1. Estrogen changes the pelvic floor muscles and lining of the uterus and may help improve a pelvic relaxation disorder. 2. Cervidil is used to prepare the cervix for delivery of a baby. It causes the cervix to shorten, soften, and dilate. 3. Progesterone is given for implantation of a fertilized ovum. 4. Pitocin causes the uterus to contract. It is used during the labor and delivery process. TEST-TAKING HINT: Options "2," "3," and "4" are similar because they are all used during a pregnancy or delivery. If the test taker doesn't have a clue as to the correct answer, the test taker should attempt to determine what answer is different from all the others.

The nurse is instructing a two (2)-week postpartum client who has red, tender breasts after trying to breastfeed the infant. Which should the nurse teach the client? 1. Be sure the baby empties each breast when feeding. 2. Apply warm, moist pack to the breasts for comfort. 3. Apply rubbing alcohol to the breast to treat the infection. 4. The baby must be given formula because the mother cannot breastfeed.

ANSWER: 1. 1. Failure to have the baby to express all of the milk produced will result in less milk produced at future feedings and for the breasts to become sore and inflamed. 2. Ice packs are usually applied to decrease the discomfort. 3. Rubbing alcohol will dry out the breast tissues and the infant should not feed after alcohol has been applied to the breast because of absorption into the infant's system. 4. The mother should be taught correct breastfeeding techniques, not to avoid breastfeeding. TEST-TAKING HINT: The test taker could eliminate option "3" because the infant could ingest the alcohol during feeding.

The nurse is assessing a 34-year-old female client with fibrocystic breasts. Which question should the nurse ask the client during the assessment? 1. "Are your breasts more tender during your period?" 2. "Have you ever developed an allergy to chocolate?" 3. "Can you tell me more about your feelings of having fibrocystic breast changes?" 4. "Have you considered having a prophylactic mastectomy?"

ANSWER: 1. 1. Mastalgia (breast pain and tenderness is common during the menstrual cycle. This question assists the nurse in determining if the mastalgia is from the fibrocystic disease. 2. Some women find that avoiding chocolate and stimulants (caffeine) reduces the discomfort of fibrocystic breasts, but it is not considered an allergy. 3. This is a therapeutic response, not an interviewing question to be asked during the assessment of the disease. 4. This is not precancerous; there is no reason for a mastectomy. TEST-TAKING HINT: The test taker could eliminate option "3" because a therapeutic response was not needed; the nurse needs to ask a question that assists in gaining information about the disease being assessed

The nurse is preparing an educational presentation for women in the community. Which primary nursing intervention should the nurse discuss regarding the development of ovarian cancer? 1. Instruct the clients not to use talcum powder on the perineum. 2. Encourage the clients to consume diets with a high-fat content. 3. Teach the women to have a lower pelvic sonogram yearly. 4. Discuss the need to be aware of the family history of cancer.

ANSWER: 1. 1. Research has shown the use of talcum powder perineally increases the risk for developing ovarian cancer, although there is no explanation known for this occurrence. Other risk factors include a highfat diet, nulliparity, infertility, older age (70 to 80 years) has the greatest incidence, mumps before menarche, and family history of ovarian cancer. 2. Nurses should never encourage a high-fat diet. 3. Only clients in a high-risk category should have routine sonograms. The time frame for the high-risk group of clients is six (6) months. This is not primary intervention; early detection is secondary intervention. 4. This alerts the client to participate in activities detecting cancer early, a secondary intervention. TEST-TAKING HINT: The test taker could eliminate options "3" and "4" because of the word "primary" in the stem. Option "2" could be eliminated because of the recommendation of a high-fat diet.

The client in the gynecology clinic asks the nurse, "What are the risk factors for developing cancer of the cervix?" Which statement is the nurse's best response? 1. "The earlier the age of sexual activity and the more partners, the greater the risk." 2. "Eating fast foods high in fat and taking birth control pills are risk factors." 3. "A Chlamydia trachomatis infection can cause cancer of the cervix." 4. "Having yearly Pap smears will protect you from developing cancer."

ANSWER: 1. 1. Risk factors for cancer of the cervix include sexual activity before the age of 20 years; multiple sexual partners; early childbearing; exposure to the human papillomavirus; HIV infection; smoking; and nutritional deficits of folates, beta carotene, and vitamin C. 2. High-fat diets place clients at risk for some cancers but not for cervical cancer. The use of birth control pills may allow increased sexual freedom because of the protection from pregnancy, but it does not increase the risk for cancer of the cervix. 3. Infections with the human papillomavirus are a risk factor for cancer of the cervix. 4. Having a yearly Pap smear increases the chance of detecting cellular changes early, but it does not decrease the risk for developing cancer. TEST-TAKING HINT: The test taker could discard option "4" as a possible answer because it is a yearly test for the early detection of cervical cancer, not a risk factor.

Which is the American Cancer Society's recommendation for the early detection of cancer of the prostate? 1. A yearly PSA level and DRE beginning at age 50. 2. A biannual rectal examination beginning at age 40. 3. A semiannual alkaline phosphatase level beginning at age 45. 4. A yearly urinalysis to determine the presence of prostatic fluid.

ANSWER: 1. 1. The American Cancer Society recommends all men have a yearly prostate-specific antigen (PSA) blood level, followed by a digital rectal examination (DRE) beginning at age 50. Men in the high-risk group, including all African American men, should begin at age 45. 2. A biannual (every 6 months) examination is not recommended. 3. Alkaline phosphatase levels are performed on men with known prostate cancer to determine bone involvement. This is not a screening test. 4. This test is done if the client has signs and symptoms of prostatitis. TEST-TAKING HINT: The nurse must be aware of recommended screening guidelines for a number of diseases. The test taker should carefully look at the time frame for the tests and the age of the client.

The nurse is instructing a group of workers at an industrial plant regarding the transmission of sexually transmitted diseases (STDs). Which information should be included in the presentation? 1. The same behaviors causing one STD could lead to another. 2. Once clients have had an STD, they develop immunity to it. 3. An infection with syphilis protects the client from being infected with HIV. 4. Herpes simplex 1 is a totally different disease from herpes simplex 2.

ANSWER: 1. 1. The behaviors leading to the development of one STD could also lead to the development of another. 2. There is no antigen-antibody reaction development with STDs. A client can be reinfected multiple times. 3. There is no protection provided by one STD from developing another, and, frequently, clients will have more than one STD simultaneously. 4. Herpes simplex 1 and 2 are caused by the same virus. Herpes simplex 1 refers to orolabial lesions and herpes simplex 2 refers to genital lesions, which can be transferred from one area to the other.

The client has an infected Bartholin's cyst and the HCP has performed an incision and drainage (I&D) of the area. Which discharge instructions should the nurse teach the client? 1. Complete all antibiotics as ordered. 2. Report any drainage immediately. 3. Keep all water away from the area. 4. Lie prone as much as possible.

ANSWER: 1. 1. The client has an infection and should complete the ordered antibiotics. 2. The client should be taught to expect some drainage from the area because the area has been opened to allow for exudate to escape the body. 3. Routine hygiene is encouraged. 4. The client can assume any position of comfort.

The client has undergone a wedge resection for cancer of the left breast. Which discharge instruction should the nurse teach? 1. Don't lift more than five (5) pounds with the left hand until released by the HCP. 2. The cancer has been totally removed and no follow-up therapy will be required. 3. The client should empty the Hemovac drain about every 12 hours. 4. The client should arrange an appointment with a plastic surgeon for reconstruction.

ANSWER: 1. 1. The client has had surgery on this side of the body. Pressure on the incision should be limited until the client is released by the HCP to perform normal daily activities. 2. This is providing the client false hope. Cancer cells characteristically move easily in the lymph or bloodstream to other parts of the body. Microscopic disease cannot be determined by the naked eye. 3. A client who has a mastectomy might be discharged with a Hemovac drain, but a wedge resection should not require one. 4. The breast has not been removed; reconstruction is not needed. TEST-TAKING HINT: If the test taker did not know this answer, option "1" is information provided to any client who has had surgery on the upper chest or arm.

Which question is most important for the nurse to ask the client with a cystocele who is scheduled to have a pessary inserted? 1. "Do you know if you are allergic to latex?" 2. "When did you start having incontinence?" 3. "When was your last bowel movement?" 4. "Are you experiencing any pelvic pressure?"

ANSWER: 1. 1. The client should be assessed for allergies to latex as a result of the composition of the pessary. 2. These clients frequently have incontinence, which is assessed prior to recommending a pessary. 3. A pessary is manually inserted to keep a prolapsed uterus in place. Asking about a bowel movement is not an appropriate question. 4. This is a symptom experienced by a client with a cystocele. TEST-TAKING HINT: If the test taker has no idea what the answer is, then the test taker should apply Maslow's hierarchy of needs, which in this question is safety. Only option "1" addresses a safety issue. Allergies are a safety issue.

Which information should the nurse include in the discharge teaching for the client recovering from an abdominal hysterectomy? 1. The client should report any vaginal bleeding or cramping to the surgeon. 2. The client should start a vigorous exercise routine to restore her muscle tone. 3. The client should continue sitting in the bedside chair at least six (6) hours daily. 4. The client should soak in a warm tub bath each night for one (1) hour.

ANSWER: 1. 1. The client should report any vaginal bleeding or gastrointestinal changes such as distention, cramping, or changes in bowel habits. 2. Clients should rest and not start a vigorous exercise program until the surgeon gives permission. 3. Clients should avoid activities increasing pelvic congestion, such as dancing, horseback riding, and sitting for long periods. 4. Clients should avoid taking baths to help prevent infection of the incision site. TEST-TAKING HINT: When the test taker is selecting possible correct answers, the test taker should carefully consider the descriptive words. In option "2," the adjective "vigorous" should cause the test taker to eliminate it, and in option "3" the words "six (6) hours" should help eliminate it

The client has failed to conceive after many attempts over a three (3)-year time period and asks the nurse, "I have tried everything. What should I do now?" Which statement is the nurse's best response? 1. "By 'everything' do you mean you have consulted an infertility specialist?" 2. "You have tried everything. This must be hard for you. Would you like to talk?" 3. "You should get on an adoption list because it can take a long time." 4. "You need to relax and not try so hard. It is your nerves preventing conception."

ANSWER: 1. 1. The nurse should investigate which fertility measures have been attempted. There are many reasons for infertility, and only a specialist in the area can identify the cause. 2. This is a therapeutic response, but the client is asking for information. 3. This is advising and not answering the client's question. 4. The nurse cannot know this to be true, and this does not address the client's concern.

The nurse is caring for a client newly diagnosed with Stage IV ovarian cancer. What is the scientific rationale for detecting the tumors at this stage? 1. The client's ovaries lie deep within the pelvis and early symptoms are vague. 2. The client has regular gynecological examinations and this helps with detection. 3. The client had a history of dysmenorrhea and benign ovarian cysts. 4. The client had a family history of breast cancer and was being checked regularly

ANSWER: 1. 1. The ovaries are anatomically positioned deep within the pelvis, and because of this, signs and symptoms of cancer are vague and nonspecific. Symptoms include increased abdominal girth, pelvic pressure, indigestion, bloating, flatulence, and pelvic and leg pain. Increasing abdomen size as a result of accumulation of fluid is the most common sign. Many women ignore the symptoms because they are so nonspecific. 2. Regular gynecological examinations are recommended, but this is advanced disease. 3. Dysmenorrhea is not a risk factor for developing ovarian cancer. Any enlarged ovary should be evaluated, especially if the client is postmenopausal, when the ovaries shrink in size. 4. A family history of breast cancer is a cause for the client to be assessed regularly for breast and ovarian cancer, but this is late disease. TEST-TAKING HINT: "Stage IV" should help the test taker to eliminate options "2" and "4" because this client has advanced disease and it is hoped regular checkups find problems early

Which statement best describes the responsibility of the public health nurse regarding sexually transmitted diseases? 1. Notify the sexual partners of clients diagnosed with an STD. 2. Determine the course of treatment for clients diagnosed with an STD. 3. Explain the legal aspects of STD reporting to a client diagnosed with an STD. 4. Analyze the statistics regarding STD transmission and reporting the findings

ANSWER: 1. 1. The public health nurse is responsible for attempting to notify sexual partners of a client diagnosed with an STD of a potential infection and urging the partner to be tested for the disease and to receive treatment. Health departments offer confidential testing and treatment. 2. An HCP will determine the course of treatment for a client diagnosed with an STD. 3. The nurse can teach some information about reporting, but the nurse is not qualified to discuss all the legal aspects of reporting an STD. 4. The nurse is not responsible for analyzing statistics. TEST-TAKING HINT: Answer options "2," "3," and "4" ask the nurse to take on roles not within the nurse's expertise. The nurse must know the Nurse Practice Act of the state where the nurse practices. No state allows the nurse to give legal or medical advice.

The nurse is caring for a client who is eight (8) hours postoperative unilateral orchiectomy for cancer of the testes. Which intervention should the nurse implement? 1. Provide an athletic supporter before ambulating. 2. Encourage the client to delay use of pain medications. 3. Place the client on a clear liquid diet for the first 48 hours. 4. Monitor the PT/INR levels and have vitamin K ready.

ANSWER: 1. 1. The scrotum will require support during ambulation. An athletic supporter is designed to provide support in this area. 2. The client should be encouraged to take pain medications before the pain is at a high level to help the pain medication be more effective. 3. The client can be on a regular diet as soon as the client is not nauseated from the anesthesia. 4. This surgery does not increase bleeding times. TEST-TAKING HINT: Option "2" can be eliminated because it states the opposite of the instructions given to clients about pain medication

The client is scheduled for a right breast biopsy for a mass found in the tail of Spence. While the client is waiting in the holding area, the client asks the nurse, "Which lymph nodes will my surgeon take from my body?" Which area should the nurse identify? 1. The tail of Spence at the upper outermost part of the breast. 2. The mediastinal node on the opposite side of the breast. 3. The internal mammary nodes under teh breast. 4. The parasternal nodes on the opposite side of the breast from the tail of Spence

ANSWER: 1. 1. The tail of Spence is the upper outermost part of the breast, which extends toward the arm. The most likely lymph nodes to biopsy are the axillary nodes. 2. This is the mediastinal node area, which is on the opposite side of the breast. 3. The internal mammary nodes are under the breast, and the tail of Spence is at the top of the breast. 4. The parasternal nodes are on the opposite side of the breast from the tail of Spence.

The male client presents to the public health clinic complaining of joint pain and malaise. On assessment, the nurse notes a rash on the trunk, palms of the hands, and soles of the feet. Which action should the nurse implement next? 1. Determine if the client has had a chancre sore within the last two (2) months. 2. Ask the client how many sexual partners he has had in the past year. 3. Refer the client to a dermatologist for a diagnostic work-up. 4. Have the client provide a clean voided midstream urine specimen.

ANSWER: 1. 1. These are signs of second-stage syphilis. The nurse should ask about the development of a chancre sore, one of the first signs of a syphilis infection. 2. This may be required of the public health nurse for notification of the partners, but it is not required to assess this problem. 3. This client does not need a dermatologist to determine an STD infection. The HCP can treat this infection. 4. A urine culture will not diagnose this disease. TEST-TAKING HINT: If the test taker is aware the symptoms are those of an STD, options "3" and "4" can be eliminated.

The client diagnosed with cancer of the testes calls and tells the nurse he is having low back pain that does not go away with acetaminophen, a nonnarcotic analgesic. Which action should the nurse implement? 1. Ask the client to come in to see the HCP for an examination. 2. Tell the client to use a nonsteroidal antiinflammatory drug instead. 3. Inform the client this means the cancer has metastasized. 4. Encourage the client to perform lower back- strengthening exercises.

ANSWER: 1. 1. This information could signal the onset of symptoms of metastasis to the retroperitoneum. The HCP should see the client and discuss follow-up diagnostic tests. 2. This information should be investigated and not put off. 3. This may or may not have occurred. Only diagnostic tests can confirm metastasis. 4. Low back exercises will not help the client if this is metastatic cancer, and they could increase the pain. TEST-TAKING HINT: The nurse cannot diagnose the client as having metastasis, so option "3" can be eliminated. As a rule, any pain unrelieved with pain medication requires the client to see the HCP

The client who has had a mastectomy tells the nurse, "My husband will leave me now since I am not a whole woman anymore." Which response by the nurse is most therapeutic? 1. "You're afraid your husband will not find you sexually appealing?" 2. "Your husband should be grateful you will be able to live and be with him." 3. "Maybe your husband would like to attend a support group for spouses." 4. "You don't know that is true. You need to give him a chance."

ANSWER: 1. 1. This is restating the client's feelings and is a therapeutic response. 2. This is not recognizing the client's concerns and putting the nurse's expectations on the spouse. 3. This is problem-solving and could be offered, but the therapeutic response is to restate the client's feeling and encourage a conversation. 4. The client may know this is true. The nurse is telling the client she has no reason for her feelings. Feelings are what they are and should be accepted as such. TEST-TAKING HINT: When the question asks for a therapeutic response, the test taker should choose an option encouraging the client to ventilate her feelings.

The nurse in the gynecology clinic is assessing a 14-year-old client who reports being sexually active. Which information should the nurse teach the client? Select all that apply. 1. Inform the client that the nurse must tell the parents of her being sexually active. 2. Teach the client about possible birth control options. 3. Instruct the client regarding sexually transmitted disease. 4. Demonstrate how a condom is applied correctly. 5. Tell the client the importance of finishing all antibiotics. 6. Discuss the importance of attending parenting classes

ANSWER: 2, 3, 4. 1. The nurse does not have to inform the parents of the teenager's disclosure of information. 2. The nurse should discuss birth control and sexually transmitted diseases with the client. She is at risk for pregnancy and STDs. 3. The nurse should discuss birth control and sexually transmitted diseases with the client. She is at risk for pregnancy and STDs. 4. The male wears the most commonly used condoms but both partners are responsible for contraception and prevention of STDs. This information will assist the client to know if the device is correctly applied and will have the best chance of preventing both pregnancy and STDs. 5. This would be information to provide if the client had an STD, but is not needed at this time. 6. The client is not pregnant at this time. TEST-TAKING HINT: The test taker could eliminate options "5" and "6" because the client is not currently pregnant

The female client presents to the gynecologist's office for the fifth time with an ovarian cyst and is scheduled for an exploratory laparoscopy. The client asks the nurse, "Why do I need to have another surgery? The other cysts have all been benign." Which statement is the nurse's best response? 1. "Because eventually the cysts will become cancerous." 2. "All abnormal findings in the ovary should be checked out." 3. "The surgery will not be painful and you will have peace of mind." 4. "Are you afraid of having surgery? Would you like to talk about it?"

ANSWER: 2. 1. All cysts do not become cancerous; 98% of ovarian cysts in clients younger than age 29 are benign, whereas in women older than age 50, about half are benign. 2. Any abnormal ovary that cannot be diagnosed with a transvaginal ultrasound should be examined laparoscopically. 3. Any time the client has surgery, she should be prepared to experience some pain. This is a false statement and could cause a breach in the nurse-client relationship. 4. This is a therapeutic statement and the client is asking for information. TEST-TAKING HINT: The test taker should read the stem of the question carefully. Option "1" has a form of absolute, "eventually will become," so it can be eliminated. Option "3" is a false statement. Option "4" is a therapeutic response and the stem asks the nurse to provide information.

The nurse is admitting a client diagnosed with trichomoniasis. Which assessment data support this diagnosis? 1. Odorless, white, curdlike vaginal discharge. 2. Strawberry spots on the vaginal surface and itching. 3. Scant white vaginal discharge and dyspareunia. 4. Purulent discharge from the endocervix and pelvic pain.

ANSWER: 2. 1. An odorless, white, curdlike vaginal discharge is a symptom of Candida albicans, a vaginal yeast infection. 2. A strawberry spot on the vaginal wall or cervix, a fishy smelling vaginal discharge, and itching are symptoms of trichomonas. 3. Scant white vaginal discharge and dyspareunia are symptoms of atrophic vaginitis. 4. Purulent discharge from the endocervix and pelvic pain are symptoms of cervicitis. TEST-TAKING HINT: When studying for a test covering similar diseases, the test taker should concentrate on the information that makes one different from another. Only one STD has a characteristic strawberry spot.

The client is diagnosed with benign uterine fibroid tumors. Which question should the nurse ask to determine if the client is experiencing a complication? 1. "How many periods have you missed?" 2. "Do you get short of breath easily?" 3. "How many times have you been pregnant?" 4. "Where is the location of the pain you are having?"

ANSWER: 2. 1. Benign fibroid tumors in the uterus cause the client to bleed longer with a heavier flow, not miss periods. 2. Many women delay surgery until anemia has occurred from the heavy menstrual flow. A symptom of anemia is shortness of breath. 3. The number of pregnancies does not matter at this time; the client has a different problem. 4. The pain is in the pelvic region to low back, where the uterus lies. TEST-TAKING HINT: This is a high-level question requiring the test taker to make several judgments before arriving at the answer. First, the test taker must decide what happens when a client has fibroid tumors and then which symptoms the client will exhibit.

The client is diagnosed with a rectovaginal fistula which is to be managed medically. Which information should the nurse teach the client prior to discharge? 1. Douche with normal saline. 2. Eat a low-residue diet. 3. Keep ice packs to the area. 4. Use an abdominal binder.

ANSWER: 2. 1. Cleansing douches are prescribed with tepid water, not normal saline. 2. Measures to assist the client to heal without surgical interventions include proper nutrition with a low-residue diet to minimize contamination of the tissues with feces, cleansing douches, enemas, and rest. 3. Warm perineal irrigations and controlled heat-lamp applications promote healing; ice vasoconstricts the area and delays wound healing. 4. The client should wear perineal pads but not an abdominal binder.

The client has had a total abdominal hysterectomy for cancer of the uterus. Which discharge instruction should the nurse teach? 1. The client should take HRT every day to prevent bone loss. 2. The client should practice pelvic rest until seen by the HCP. 3. The client can drive a car as soon as she is discharged from the hospital. 4. The client should expect some bleeding after this procedure.

ANSWER: 2. 1. Clients who are diagnosed with cancer of the uterus have the ovaries removed to reduce hormone production. The client will not be taking HRT. 2. Pelvic rest means nothing is placed in the vagina. The client does not need a tampon at this time, but sexual intercourse should be avoided until the vaginal area has healed. 3. The sitting position a client assumes when driving a vehicle places stress on the lower abdomen. The client should wait until the HCP releases her to drive. 4. The client should not have any vaginal bleeding. TEST-TAKING HINT: The test taker should apply basic postoperative concepts when answering questions and realize bleeding is not expected postoperatively and safety should always be addressed.

The nurse writes a problem of "potential for complications related to ovarian hyperstimulation" for a client who is taking clomiphene (Clomid), an ovarian stimulant. Which intervention should be included in the plan of care? 1. Instruct the client to delay intercourse until menses. 2. Schedule the client for frequent pelvic sonograms. 3. Explain the infusion therapy will take 21 days. 4. Discuss that this may cause an ectopic pregnancy.

ANSWER: 2. 1. Clomid is begun on the fifth day of the menstrual cycle and is taken for five (5) days; ovulation should occur four (4) to eight (8) days after—approximately from day nine (9) to day 13 of the cycle. Intercourse should be planned for the optimum chance of conception. 2. Frequent sonograms are needed to monitor follicular stimulation. The ovaries are monitored to prevent overstimulation, which can cause ascites, pleural effusions, and acute respiratory distress syndrome (ARDS). 3. Clomid is not given by infusion. Gonadotropinreleasing hormone (GnRH) is given by infusion. 4. Ovarian stimulation does not cause ectopic pregnancy

The nurse is admitting a client diagnosed with Stage Ia cancer of the cervix to an outpatient surgery center for a conization. Which data would the client most likely report? 1. Diffuse watery discharge. 2. No symptoms. 3. Dyspareunia. 4. Intense itching

ANSWER: 2. 1. Diffuse, watery, foul-smelling discharge occurs at a much later stage. 2. At this stage the client is asymptomatic and the cancer has been determined by a Pap smear. 3. Dyspareunia is painful sexual intercourse; the client is asymptomatic. 4. Intense itching occurs with vaginal yeast infections. TEST-TAKING HINT: The test taker could either choose option "2" because it is the least presenting symptom or discard it. Staging for all cancers starts with "0" or "1," indicating the least detectable cancer.

Which client has the highest risk for developing cancer of the testicles? 1. The client diagnosed with epididymitis. 2. The client born with cryptorchidism. 3. The client with an enlarged prostate. 4. The client diagnosed with hypospadias

ANSWER: 2. 1. Epididymitis is inflammation of the epididymis, usually caused by bacteria descending from the prostate or bladder. This does not increase the risk of developing cancer of the testes. 2. Cryptorchidism is the medical term for undescended testicle. The testicles may be in the abdomen or inguinal canal at birth. This condition places the client at higher risk for testicular cancer. 3. Prostate enlargement occurs as men age, whereas the most common age range for testicular cancer is age 15 to 35. 4. Hypospadias is a congenital abnormality in which the urethral meatus is on the underside of the penis. This does not increase the risk for cancer. TEST-TAKING HINT: This is a knowledgebased question, but answer options "3" and "4" could be eliminated based on anatomical position

The client is diagnosed with breast cancer and is considering whether to have a lumpectomy or a more invasive procedure, a modified radical mastectomy. Which information should the nurse discuss with the client? 1. Ask if the client is afraid of having general anesthesia. 2. Determine how the client feels about radiation and chemotherapy. 3. Tell the client she will need reconstruction with either procedure. 4. Find out if the client has any history of breast cancer in her family

ANSWER: 2. 1. General anesthesia is used for either procedure. 2. The client should understand the treatment regimen for follow-up care. A lumpectomy requires follow-up with radiation therapy to the breast and then systemic chemotherapy. If the cancer is in its early stages, this regimen has results equal to those with a modified radical mastectomy. 3. A lumpectomy removes only the tumor and a small amount of tissue surrounding the tumor; reconstruction is not needed. 4. A history of breast cancer in the family is immaterial because this client has breast cancer. TEST-TAKING HINT: The test taker should use the nursing process to answer this question and select an assessment intervention, which eliminates option "3" as a correct answer. Option "1" uses the word "afraid," which is an assumption; therefore, this option could be eliminated.

Which nursing task could be delegated to the unlicensed assistive personnel (UAP) for the client who had a total vaginal hysterectomy? 1. Observe the color and amount of drainage on the client's perineal pad. 2. Maintain a current intake and output for the client each shift. 3. Provide the client with a plan of pharmacological pain management. 4. Prepare the client for her discharge scheduled for the next day

ANSWER: 2. 1. Observation is assessment, which cannot be delegated. 2. This nursing task can be delegated, but evaluation is the responsibility of the nurse. 3. Teaching cannot be delegated. 4. Teaching cannot be delegated. TEST-TAKING HINT: The nurse cannot delegate assessment, teaching, evaluation, or an unstable client to a UAP.

The nurse is preparing the client for an insertion of a pessary. Which information should the nurse teach the client? 1. The pessary does not need to be changed. 2. The client should clean the pessary routinely. 3. The pessary must be inserted in surgery. 4. Estrogen cream is necessary for effective use of a pessary.

ANSWER: 2. 1. Pessaries need to be changed to prevent complications. 2. Clients do need to clean the pessary routinely. 3. Pessaries are inserted in areas other than surgery, such as in a health-care provider's office. 4. Hormone cream may be used, but usually an oral estrogen is prescribed. TEST-TAKING HINTS: The test taker should realize any removable device inserted into the body requires routine changing at some time; therefore, option "1" can be eliminated. Option "4" is a medication and a pessary is a device, so option "4" can be eliminated.

The 45-year-old male client has had a circumcision secondary to phimosis. Which intervention should the nurse include in the plan of care? 1. Teach how to care for the glans to prevent recurrence of the phimosis. 2. Assess for pain on a scale of one (1) to 10. 3. Perform wet-to-dry dressing changes daily. 4. Instruct the client to perform a monthly penis check for cancer

ANSWER: 2. 1. Phimosis is a tightness of the prepuce of the penis preventing retraction of the foreskin over the glans. Once the foreskin has been surgically removed, this is not a problem. 2. There is considerable pain after an adult circumcision, and the nurse should assess for the pain. 3. A petroleum-saturated gauze is wrapped around the penis, not a wet-to-dry dressing. 4. Circumcision in infancy may prevent cancer of the penis. The older client who has a circumcision should be aware of the potential for cellular changes, but the risk is greatly reduced after the surgery.

The client is diagnosed with vulvar cancer. Which are the most common symptoms of cancer of the vulva? 1. Red, painful lesions. 2. Vulvar itching. 3. Thin, white vulvar skin. 4. Vaginal dryness

ANSWER: 2. 1. Red, painful lesions are symptoms of lichen planus, which is a benign, although uncomfortable, lesion. 2. Cancer of the vulva may be asymptomatic, but the client usually presents with persistent long-term itching. 3. Thin, white vulvar skin indicates lichen sclerosis. 4. Vaginal dryness is not associated with cancer of the vulva.

The 67-year-old male client reports difficulty initiating a urinary stream, urinary frequency, and inability to completely empty the bladder. Which procedure would the nurse anticipate the health-care provider (HCP) performing first? 1. Digital rectal examination (DRE). 2. Prostate-specific surface antigen (PSA). 3. Prostate ultrasound. 4. Biopsy of the prostate.

ANSWER: 2. 1. The DRE is performed after the PSA to avoid false elevation of the PSA from the manipulation of the prostate during DRE. 2. The PSA is drawn before the DRE to prevent a false elevation due to the prostate being massaged during the DRE. 3. Prostate ultrasound is performed if a nodule is found or suspected and cannot be localized on DRE. It would not be anticipated as the first action by the HCP. 4. A biopsy would be done if all the other data are positive, but not first. TEST-TAKING HINT: The test taker could eliminate options "3" and "4" because they are more invasive than a laboratory test or a digital examination

The school nurse is preparing a class on testicular cancer for male high school seniors. Which information regarding testicular selfexamination should the nurse include? 1. Perform the examination in a cool room under a fan. 2. Any lump should be examined by an HCP as soon as possible. 3. Discuss having a second person confirm a negative result. 4. The procedure will cause mild discomfort if done correctly.

ANSWER: 2. 1. The body temperature should be warm for the scrotum to relax. The best place to perform testicular self-examination is in a warm to hot shower. 2. The client may note a cordlike structure; this is the spermatic cord and is normal. Any lump or mass felt is abnormal and should be checked by an HCP as soon as possible. 3. The client can confirm his own negative result; a negative result is no masses felt. 4. The procedure is painless. If pain is elicited, then an HCP should examine the client. Cancer is usually painless; the presence of pain may indicate an infection. TEST-TAKING HINT: The test taker might choose option "2" as an answer because any abnormality should be examined by an HCP

The male client complains of mucus-like drainage from the rectum accompanied by rectal pain and diarrhea. Which interview question should the nurse ask the client? 1. "Do you have difficulty trying to urinate?" 2. "Have you had rectal sexual intercourse?" 3. "Do you eat a high-fiber diet and drink lots of fluids?" 4. "Does the diarrhea alternate with constipation?"

ANSWER: 2. 1. The client has rectal symptoms, not urinary tract symptoms. 2. The client has described symptoms of proctitis (inflammation of the rectum). Proctitis is commonly associated with anal-receptive intercourse with an infected partner. The pathogens most frequently associated with proctitis are gonorrhea, chlamydia, herpes simplex, and Treponema pallidum. 3. A high-fiber diet does not cause these symptoms. 4. Diarrhea alternating with constipation indicates a possible rectal tumor and does not cause these symptoms.

The nurse is reviewing the laboratory data on a male client. Which interpretation should the nurse make regarding the prostate-specific antigen (PSA) of 6 mcg/L (normal: Male less than 4; female less than 0.5). 1. The client has early-stage prostate cancer. 2. The client should have more tests. 3. The client does not have prostate cancer. 4. The client has benign prostatic hypertrophy.

ANSWER: 2. 1. The client may have cancer of the prostate, but this test does not provide conclusive results. There are several reasons for the PSA to be elevated, not just cancer. 2. The PSA is elevated and more tests should be completed to determine the cause. PSA levels are increased in benign prostatic hypertrophy, urinary retention, prostatic infarct, and prostate cancer. 3. Cancer cannot be eliminated as a diagnosis until other tests have been completed. 4. This may be the actual diagnosis, but the client should undergo more tests to confirm a diagnosis.

The client diagnosed with cancer of the prostate tells the nurse, "I caused this by being promiscuous when I was young and now I have to pay for my sins." Which statement is the nurse's most therapeutic response? 1. "Why would you think prostate cancer is caused by sex?" 2. "You feel guilty about some of your actions when you were young?" 3. "Well, there is nothing you can do about that behavior now." 4. "Have you told the HCP and been checked for an AIDS infection?"

ANSWER: 2. 1. The nurse should never ask "why." The client does not owe the nurse an explanation. 2. The question asks for a therapeutic response from the nurse. This response is restating and clarifying. 3. This is dismissive to the client's feelings and appears to agree with the client. There is no evidence cancer of the prostate is caused by promiscuous behavior. 4. This is a problem-solving statement and does not address the client's feelings. TEST-TAKING HINT: Because the question is asking for a therapeutic response, the test taker should address the client's feelings. However, the test taker should not use this test-taking hint as an absolute because each question should be answered on its own merit.

The 30-year-old male client diagnosed with germinal cell carcinoma of the testes asks the nurse, "What chance do I have? Should I end it all now?" Which response by the nurse indicates an understanding of the disease process? 1. "God does not want you to give up hope and end it all now." 2. "There is a good chance for survival with standard treatment options." 3. "There may be little hope, but ending it all is not the answer." 4. "You have a 50/50 chance of living for at least 5 years."

ANSWER: 2. 1. The nurse should not impose any personal religious beliefs on the client. 2. Testicular cancers have very good prognoses, and even if the tumor returns, there is a good prognosis for extended survival. 3. There is a great deal of hope to offer these clients. 4. This is giving the client incorrect information. TEST-TAKING HINT: This is a question in which the test taker must know the information. Rarely can a nurse provide a specific percentage of survival.

The nurse is caring for a client diagnosed with uterine cancer who has been receiving systemic therapy for six (6) months. Which intervention should the nurse implement first? 1. Determine which antineoplastic medication the client has received. 2. Ask the client if she has had any problems with mouth ulcers at home. 3. Administer the biologic response modifier filgrastim (Neupogen). 4. Encourage the client to discuss feelings about having cancer.

ANSWER: 2. 1. This can be done to determine specific problems resulting from the specific side effects of the medication, but it is not the first action. The nurse can ask general assessment questions to determine how the client is tolerating the treatments. 2. The systemic side effects of chemotherapy are not always apparent, and the development of stomatitis can be extremely distressing for the client. The nurse should assess the client's tolerance to treatments. 3. This is done if the white blood cell count is low. The nurse assesses the WBC count and then obtains an order from the HCP. 4. This is an appropriate action but not before assessing physical problems. TEST-TAKING HINT: When prioritizing nursing interventions, the test taker should apply the nursing process, and assessment is the first step.

The client diagnosed with cancer of the prostate has been placed on luteinizing hormone-releasing hormone (LHRH) agonist therapy. Which statement indicates the client understands the treatment? 1. "I will be able to function sexually as always." 2. "I may have hot flashes while taking this drug." 3. "This medication will cure the prostate cancer." 4. "There are no side effects with this medication."

ANSWER: 2. 1. This hormone will suppress the production of male hormones, and the client will not function sexually as always. 2. The client may have hot flashes because these drugs increase hypothalamic activity, which stimulates the thermoregulatory centers of the body. 3. The medication decreases the growth rate of the cancer, but it does not cure the cancer. 4. There are side effects with all medications. These medications can cause gynecomastia, hot flashes, cardiovascular effects, and decreased sexual functioning. The LHRH agonists have fewer side effects than the estrogens. TEST-TAKING HINT: The test taker could eliminate options "3" and "4" because of the promise of no side effects or a cure.

The female client diagnosed with human papillomavirus (HPV) asks the nurse, "What other problems can HPV lead to?" Which statement is the most appropriate response by the nurse? 1. "HPV is transmitted during sexual intercourse." 2. "HPV infection can cause cancer of the cervix." 3. "It has been known to lead to ovarian problems." 4. "Regular Pap smears can help prevent problems."

ANSWER: 2. 1. This is a true statement, but it does not answer the client's question. 2. Untreated HPV infection is a cause for developing cancer of the cervix. 3. HPV infection does not invade the abdominal cavity and therefore does not cause ovarian cancer. 4. The Pap test was developed to note early cell changes in the cervix. It indirectly monitors the effects of HPV, but it does not help prevent problems. TEST-TAKING HINT: The test taker should choose the answer for the question the client is asking. Option "1" discusses transmission and option "4" discusses prevention; therefore, these two (2) options could be eliminated based on the stem of the question.

The female client has a mother who died from ovarian cancer and a sister diagnosed with ovarian cancer. Which recommendations should the nurse make regarding early detection of ovarian cancer? 1. The client should consider having a prophylactic bilateral oophorectomy. 2. The client should have a transvaginal ultrasound and a CA-125 laboratory test every six (6) months. 3. The client should have yearly magnetic resonance imaging (MRI) scans. 4. The client should have a biannual gynecological examination with flexible sigmoidoscopy

ANSWER: 2. 1. This is appropriate information if the client is in her mid- to late-30s and has completed her family, but this is not discussing early detection of ovarian cancer. 2. A transvaginal ultrasound is a sonogram in which the sonogram probe is inserted into the vagina and sound waves are directed toward the ovaries. The CA-125 tumor marker is elevated in several cancers. It is nonspecific but, coupled with the sonogram, can provide information about ovarian cancer for early diagnosis. 3. Yearly MRI scans will not provide the information the two (2) tests will, and every 12 months is too long an interval. 4. A flexible sigmoidoscopy provides the HCP with a visual examination of the sigmoid colon, not the ovaries. TEST-TAKING HINT: The test taker could eliminate option "4" because of the anatomical site and option "3" because of the time factor. The test taker should ask, "If looking for early detection, at what interval should the client see the HCP?"

The client had a mastectomy for cancer of the breast and asks the nurse about a TRAM flap procedure. Which information should the nurse explain to the client? 1. The surgeon will insert a saline-filled sac under the skin to simulate a breast. 2. The surgeon will pull the client's own tissue under the skin to create a breast. 3. The surgeon will use tissue from inside the mouth to make a nipple. 4. The surgeon can make the breast any size the client wants the breast to be.

ANSWER: 2. 1. This is done for reconstruction of a breast or augmentation of breast size, but it is not a TRAM flap procedure, which uses the client's own tissue. 2. The TRAM flap procedure is one in which the client's own tissue is used to form the new breast. Abdominal tissue and fat are pulled under the skin with one end left attached to the site of origin to provide circulation until the body builds collateral circulation in the area. 3. The plastic surgeon can rebuild a nipple from pigmented skin donor sites or can tattoo the nipple in place. 4. This is true of saline implants but not of TRAM flaps. TEST-TAKING HINT: If the test taker is taking a standard pencil-and-paper test and is not familiar with this procedure, then skipping the question and returning to it at a later time is advisable. Another question might give a clue about the procedure. This is not possible on the NCLEX-RN computerized examination

The client diagnosed with uterine cancer is complaining of lower back pain and unilateral leg edema. Which statement best explains the scientific rationale for these signs/symptoms? 1. This is expected pain for this type of cancer. 2. This means the cancer has spread to other areas of the pelvis. 3. The pain is a result of the treatment of uterine cancer. 4. Radiation treatment always causes some type of pain in the region.

ANSWER: 2. 1. This pain indicates metastasis to the retroperitoneal region. If caught early, a complete hysterectomy is usually the only therapy recommended. This type of pain indicates the cancer is advanced and the prognosis is poor. 2. This pain indicates the cancer is in the retroperitoneal region and the prognosis is poor. 3. Pain is not part of the treatment of cancer. Surgery may cause pain, but most treatments do not. 4. Radiation therapy does not always result in pain; it depends on the area irradiated. TEST-TAKING HINT: Option "4" has the absolute word "always" and should be eliminated as a correct answer. The stem is describing symptoms in regions other than the lower pelvis, so an educated choice is option "2."

The 50-year-old female client complains of bloating and indigestion and tells the nurse she has gained two (2) inches in her waist recently. Which question should the nurse ask the client? 1. "What do you eat before you feel bloated?" 2. "Have you had your ovaries removed?" 3. "Are your stools darker in color lately?" 4. "Is the indigestion worse when you lie down?"

ANSWER: 2. 1. This statement would be appropriate if not for the abdominal girth change. This should alert the nurse to some internal reason for the change in girth. Ascites causes a change in abdominal girth. 2. Ovarian cancer has vague symptoms of abdominal discomfort, but increasing abdominal girth is the most common symptom. If the client has had the ovaries removed, then the nurse could assess for another cause. 3. This could be assessing for a peptic ulcer, but ulcers do not cause increasing abdominal girth. 4. This is a question to determine if the client has gastroesophageal reflux, but this does not cause increased waist size. TEST-TAKING HINT: The test taker must notice all the symptoms the client is reporting. Flatulence and bloating could be associated with a number of problems, but these symptoms, along with increased waist size, narrow the possibilities.

Which laboratory test should the nurse expect for the client to rule out the diagnosis of syphilis? 1. Vaginal cultures. 2. Rapid plasma reagin card test (RPR-CT). 3. Gram-stained specimen of the urethral meatus. 4. Immunological assay

ANSWER: 2. 1. Vaginal cultures are obtained to assess for gonorrhea and chlamydia. 2. The RPR test and the Venereal Disease Research Laboratory (VDRL) test are diagnostic tests for syphilis. 3. Gram stains of the vagina or urethral meatus of a male may be done for gonorrhea but not for syphilis. 4. An immunological assay may be done for chlamydia but not for syphilis. TEST-TAKING HINT: The test taker must memorize the tests used to diagnose specific STDs and the symptoms differentiating one STD from another

The client who is four (4) months pregnant finds a lump in her breast and the biopsy is positive for Stage II cancer of the breast. Which treatment should the nurse anticipate the HCP recommending to the client? 1. A lumpectomy to be performed after the baby is born. 2. A modified radical mastectomy. 3. Radiation therapy to the chest wall only. 4. Chemotherapy only until the baby is born.

ANSWER: 2. 1. Waiting until the baby is born allows the cancer to continue to develop and spread. This might be an option if the client were in the third trimester, but not at this early stage. 2. A modified radical mastectomy is recommended for this client because the client is not able to begin radiation or chemotherapy, which are part of the regimen for a lumpectomy or wedge resection. Many breast cancers developed during pregnancy are hormone sensitive and have the ideal grounds for growth. The tumor should be removed as soon as possible. 3. Radiation therapy cannot be delivered to a pregnant client because of possible harm to the fetus. 4. Chemotherapy is not given to the client while she is pregnant because of potential harm to the fetus. TEST-TAKING HINT: The test taker should eliminate options "3" and "4" because of potential harm to the fetus but also because each option has the word "only." There are very few "onlys" in health care.

The client scheduled for a radical prostatectomy surgical procedure has an intravenous antibiotic medication ordered on call to surgery. The antibiotic is prepared in 100 mL of sterile normal saline. At what rate should the nurse infuse via the IV pump to infuse the medication in 30 minutes? ________

ANSWER: 200 mL/hr. IV pumps work on the principle of number of milliliters per hour to infuse, and, unless otherwise specified, IVPB medications are infused over a 30-minute time frame. Sixty (60) minutes in one (1) hour divided by 30 = 2. Two (2) times the volume of 100 mL = 200 mL, the rate at which the nurse should set the pump. TEST-TAKING HINT: The test taker must know basic rules for medication administration and be able to compute simple math equations.

The client who had a right modified radical mastectomy four (4) years before is being admitted for a cardiac work-up for chest pain. Which intervention is most important for the nurse to implement? 1. Determine when the client had chemotherapy last. 2. Ask the client if she received Adriamycin, an antineoplastic agent. 3. Post a message at the HOB for staff not to use the right arm for venipunctures or BPs. 4. Examine the chest wall for cancer sites.

ANSWER: 3. 1. A client four (4) years postmastectomy should be finished with adjuvant therapy, which lasts from six (6) months to one (1) year. 2. The client may have received Adriamycin, which is a cardiotoxic medication, but knowing this will not change the tests performed or preparation for the tests. 3. The nurse should post a message at the head of the client's bed to not use the right arm for blood pressures or laboratory draws. This client is at risk for lymphedema, and this is a lymphedema precaution. 4. The chest wall is sometimes involved in breast cancer, but the most important intervention is to prevent harm to the client. TEST-TAKING HINT: The question is asking for an intervention common in the health-care industry. There are many breast cancer survivors who go on to develop unrelated problems, but the nurse must still be aware of the lingering needs of the client.

The outpatient clinic nurse is working with clients diagnosed with sexually transmitted diseases (STD). Which long-term complication should the nurse discuss with the clients about STDs? 1. Stress the need for clients to completely finish all antibiotics prescriptions. 2. Inform the clients that, legally, many STIs must be reported to the health department. 3. Sexually transmitted diseases can result in reproductive problems. 4. Discuss the myth that acquired immunodeficiency syndrome is an STI

ANSWER: 3. 1. A general rule when discussing antibiotics is to teach clients to finish all of the prescription, but this is not specific information related to STIs. 2. Most STIs must be reported to comply with public health laws, but this is not a long-term complication of having an STI. 3. Because of scarring of reproductive tissue, infertility may be an issue resulting from STI infection. 4. Acquired immunodeficiency syndrome (AIDS) is a sexually transmitted disease and can also be transmitted by non-sexual blood and body fluid exposure. TEST-TAKING HINT: The test taker can rule out option "1" because of the generalized nature of the option; it is not specific to STDs. Option "2" does not address a longterm complication for the client. And option "4" requires the test taker to know the transmission of the disease.

The nurse is caring for a 30-year-old nulliparous client who is complaining of severe dysmenorrhea. Which diagnostic test should the nurse prepare the client to undergo to determine the diagnosis? 1. A bimanual vaginal exam. 2. A pregnancy test. 3. An exploratory laparoscopy. 4. An ovarian biopsy.

ANSWER: 3. 1. A vaginal examination does not provide a definitive diagnosis to determine the cause of the pain. 2. A pregnancy test is not usually ordered unless the client has a reason to think she may be pregnant. Pregnancy temporarily alleviates the symptoms of endometriosis because neither ovulation nor menses occurs during pregnancy. 3. There is a high incidence of endometriosis among women who have never had children (nulliparity) and those who have children later in life. The most common way to diagnose this condition is through an exploratory laparoscopy. 4. The ovaries lie deep within the pelvic cavity. To reach the ovaries, some form of abdominal procedure must be performed, such as a laparoscopy. However, the symptoms are not those of an ovarian cyst. TEST-TAKING HINT: The test taker could eliminate answer option "1" because "diagnosis" is in the stem. The stem is asking for a procedure providing a definitive diagnosis. Option "2" could be eliminated because, if the client is menstruating (dysmenorrhea means "painful menstruation"), then the client is usually not pregnant.

The nurse is teaching men about early detection of prostate cancer according to the American Cancer Society (ACS) guidelines. Which should the nurse teach the clients? 1. Beginning at age 39 a Digital Rectal Examination (DRE) followed by a prostatespecific antigen (PSA). 2. Beginning at age 45 a rectal sonogram and, if positive, followed by a Digital Rectal Examination (DRE). 3. Beginning at age 50 a prostate-specific antigen (PSA) followed by a Digital Rectal Examination (DRE). 4. Beginning at age 60 a Prostate-Specific Antigen (PSA) followed, if positive, by a prostate biopsy

ANSWER: 3. 1. According to the ACS beginning at age 50 a PSA level followed by a DRE should be done yearly. 2. According to the ACS beginning at age 50 a PSA level followed by a DRE should be done yearly. 3. According to the ACS beginning at age 50 a PSA level followed by a DRE should be done yearly. 4. According to the ACS beginning at age 50 a PSA level followed by a DRE should be done yearly. TEST-TAKING HINT: The test taker must have a level of knowledge concerning basic nursing activities and information to teach a client.

The nurse writes a problem of "anticipatory grieving" for a client diagnosed with ovarian cancer. Which nursing intervention is priority for this client? 1. Request the HCP to order an antidepressant medication. 2. Refer the client to a CanSurmount volunteer for counseling. 3. Encourage the client to verbalize feelings about having cancer. 4. Give the client an advance directive form to fill out.

ANSWER: 3. 1. An antidepressant may be needed at some time, but at this point the nurse should offer his or her time and interest and encourage the client to discuss the feeling of having cancer. 2. CanSurmount volunteers are extremely helpful in talking about having cancer with the client, but they do not provide counseling. The programs work based on the fact someone who has had cancer and gone through treatment can relate to the client about to begin treatment. 3. The nurse should plan to spend time with the client and allow the client to discuss the feelings of having cancer, dying, fear of the treatments, and any other concerns. 4. The client will need to complete an advance directive, but this action does not address the client's grieving process. TEST-TAKING HINT: The test taker could eliminate option "2" because a client is not referred to a volunteer for counseling. Only one (1) option directly addresses the problem and requires the nurse to interact with the client.

The young female client is admitted with pelvic inflammatory disease secondary to a chlamydia infection. Which discharge instruction should be taught to the client? 1. The client will develop antibodies to protect against a future infection. 2. This infection will not have any long-term effects for the client. 3. Both the client and the sexual partner must be treated simultaneously. 4. Once the infection subsides, the pain will go away and not be a problem.

ANSWER: 3. 1. Chlamydia does not cause an antigen- antibody reaction. 2. There are long-term problems associated with any STD. Chlamydia may have the long-term effects of chronic pain and increased risk for ectopic pregnancy, postpartum endometritis, and infertility. 3. If both the client and the sexual partner are not treated simultaneously, the sexual partner can reinfect the client. 4. The client may develop chronic pelvic pain as a result of the infection. TEST-TAKING HINT: Options "2" and "4" have a form of absolute. The words "any," "will," or "will not" are absolutes and in health care, there are very few absolutes.

The day nurse is administering the initial dose of medications to a newly admitted client at 0900. Which medication should the nurse administer first? (Digoxin PO every day at 0900; Furosemide every 12 hours at 0900; Gentamycin every 6 hours at 1000; Acetaminophen every 4 hours PRN) 1. Digoxin. 2. Furosemide. 3. Gentamycin. 4. Acetaminophen.

ANSWER: 3. 1. Digoxin is a routine maintenance medication and the nurse can begin the medication from 0800 to 1000, an hour before or after the 0900 time scheduled. 2. Furosemide is a routine maintenance medication and the nurse can begin the medication from 0800 to 1000, an hour before or after the 0900 time scheduled. 3. An initial dose of IV antibiotic should be considered a STAT or Now medication. The nurse should make sure that any ordered cultures are obtained and the medication initiated within one (1) hour of the prescription being written. 4. Acetaminophen is a mild analgesic or antipyretic medication; this could be administered if needed after the antibiotic. TEST-TAKING HINT: The test taker could eliminate options "1" and "2" because the guidelines allow up to one (1) hour after the scheduled time to administer these medications.

The client is diagnosed with left mastitis. Which assessment findings should the nurse observe? 1. Dimpling of the left breast when the client raises the arm. 2. A round lump in the left breast that is tender during menses. 3. A dull pain in the left breast and tough, doughy feeling skin. 4. Bloody discharge from the nipple and a hard palpable mass.

ANSWER: 3. 1. Dimpling of the breast indicates a tumor has attached itself to the chest wall. 2. This indicates fibrocystic changes in the left breast. 3. Mastitis is an infection of the breast occurring most often in women who are lactating. The breast becomes red and warm to touch. The skin becomes doughy and tough in consistency, and the client develops a dull pain in the affected breast. 4. Bloody discharge indicates a tumor, benign or malignant.

The occupational health nurse is preparing a class regarding sexually transmitted diseases (STDs) for employees at a manufacturing plant. Which high-risk behavior information should be included in the class information? 1. Engaging in oral or anal sex decreases the risk of getting an STD. 2. Using a sterile needle guarantees the client will not get an STD. 3. The more sexual partners, the greater the chance of developing an STD. 4. If a condom is used, the client will not get a sexually transmitted disease.

ANSWER: 3. 1. Engaging in oral and anal sex increases the risk of contracting an STD. 2. Using a sterile needle for drug abuse ensures the client will not get an STD from needle sharing, but the client can still contract an STD from other risky behaviors. 3. The more sexual partners, the greater the risk for contracting an STD. 4. Condom use provides a barrier to contracting an STD, but it is not a guarantee. The condom can break or come off during intercourse. TEST-TAKING HINT: In option "2" the word "guarantees" appears, and the nurse cannot guarantee anything in dealing with healthcare issues. Option "4" is an absolute statement—"will not get"—and can be eliminated on this basis.

The male client is considering a vasectomy for birth control. Which information should the nurse teach the client? 1. Instruct the client to use hot packs to relieve scrotal edema after the surgery. 2. Tell the client to wear loose-fitting boxer underwear after the surgery. 3. Explain initially an alternate form of birth control will be required. 4. Discuss potency will be diminished about 20% after a vasectomy

ANSWER: 3. 1. Ice packs, not hot packs, should be applied after surgery to reduce edema. 2. The client should wear cotton, jockey-type underwear to provide support and added comfort. 3. The client will not be sterile until the sperm stored distal to the surgery site in the tubules have been ejaculated or reabsorbed by the body. Therefore, an alternate form of birth control is needed for a certain period of time following the vasectomy. 4. There is no effect on the client's potency after a bilateral vasectomy

The nurse and an unlicensed assistive personnel (UAP) are caring for clients on a genitourinary floor. Which nursing task can be delegated to the UAP? 1. Increase the drip rate on the Murphy drip irrigation set. 2. Check the suprapubic catheter insertion site for infection. 3. Encourage the two (2)-hour postoperative client to turn and cough. 4. Document the amount of red drainage in the catheter.

ANSWER: 3. 1. Increasing the drip rate on a continuous bladder (Murphy) irrigation requires nursing judgment and cannot be delegated. 2. In this situation, checking for infection is an assessment situation. The nurse cannot delegate assessment. 3. The unlicensed assistive personnel can be asked to help a client turn, cough, and deep breathe. This requires the UAP to perform an action only, not to use judgment or to assess. 4. Red drainage in the catheter implies bleeding. The nurse should assess the amount of bleeding occurring. TEST-TAKING HINT: The test taker should choose the lowest level activity when choosing an action to have unlicensed assistive personnel perform. Blood should always be assessed.

The client is diagnosed with tertiary syphilis. Which signs and symptoms should the nurse expect the client to exhibit? 1. Lymphadenopathy and hair loss. 2. Warts in the genital area. 3. Dementia and psychosis. 4. Raised rash covering the body

ANSWER: 3. 1. Lymphadenopathy and hair loss are symptoms of secondary syphilis, not tertiary syphilis. 2. Genital warts are not signs of tertiary syphilis. 3. Aortitis and neurosyphilis (dementia, psychosis, stroke, paresis, and meningitis) are the most common manifestations of tertiary syphilis. 4. A rash covering the body is a symptom of gonorrhea. TEST-TAKING HINT: The key word in this question is "tertiary." The test taker must decide which disease has three (3) distinct phases and then which symptoms accompany each phase.

The nurse is assessing a client with rule-out testicular cancer. Which assessment data support the client having testicular cancer? 1. The client complains of pain when urinating. 2. There is a chancre sore on the shaft of the penis. 3. The client complains of heaviness in the scrotum. 4. There is a red, raised rash on the testes

ANSWER: 3. 1. Pain when urinating indicates a urinary tract infection. 2. A chancre sore on the shaft of the penis indicates syphilis, a sexually transmitted disease. 3. Classic signs of cancer of the testes are a mass on the testicle, painless enlargement of the testes, and heaviness of the scrotum or lower abdomen. 4. There is no rash associated with cancer of the testes. TEST-TAKING HINT: The test taker could eliminate option "2" because the penis and testes are separate body parts. The testicles lie within the scrotum. Only one (1) option concerns that part of the body.

The nurse is discussing pelvic floor exercises with a client. Which information should the nurse teach? 1. Perform the exercises four (4) times per day. 2. The exercises will prevent stress incontinence. 3. Contract the perineal muscles and hold for 10 seconds. 4. Contract the abdominal and buttock muscles to increase strength.

ANSWER: 3. 1. Pelvic floor (Kegel) exercises should be performed 30 to 80 times per day. 2. The exercises will help reduce stress incontinence, but they may not relieve all stress incontinence. 3. Perineal muscles should be contracted and held for 10 seconds followed by 10 seconds of rest. 4. The pelvic floor muscles are contracted without contracting the abdominal, buttock, or inner-thigh muscles

The nurse is working in a health clinic. Which disease is required to be reported to the public health department? 1. Pelvic inflammatory disease. 2. Epididymitis. 3. Syphilis. 4. Ectopic pregnancy

ANSWER: 3. 1. Pelvic inflammatory disease (PID) does not have to be reported, but the cause of the PID may need to be reported. 2. There are causes for epididymitis other than an STD. 3. Syphilis is an STD and therefore must be reported to the appropriate health department. 4. An ectopic pregnancy may have numerous causes. TEST-TAKING HINT: Only one (1) answer option is an STD. The other diseases/conditions may be caused by STDs, but they all have other causes as well.

The 80-year-old male client has been diagnosed with cancer of the prostate. Which treatment should the nurse discuss with the client? 1. Radiation therapy every day for four (4) weeks. 2. Radical prostatectomy with lymph node dissection. 3. Diethylstilbestrol (DES), an estrogen, daily. 4. Penile implants to maintain sexual functioning

ANSWER: 3. 1. Radiation therapy is considered aggressive therapy and is not recommended for the elderly client unless needed to alleviate pain from bony metastasis. 2. A radical prostatectomy with lymph node dissection is extensive surgery and only recommended for clients with a life expectancy of greater than 10 years. 3. DES is a hormone preparation that suppresses the male hormones and slows the growth of the tumor. Some men with a life expectancy of less than 10 years choose not to treat the cancer at all and will usually die from causes other than prostate cancer. 4. Penile implants do not treat prostate cancer. Sexual functioning may or may not be impaired, depending on whether the client is treated surgically by a radical prostatectomy. If so, then the client may be prescribed Viagra or Cialis. Eighty-year-old men are not candidates for a radical prostatectomy. TEST-TAKING HINT: The test taker should notice the age of the client. When an age is provided in the question, it is significant. The elderly do not tolerate many treatments well, so the test taker should choose the least invasive treatment.

The nurse is caring for a client diagnosed with uterine cancer who has received afterload intracavitary radiation. Which precaution should the nurse implement? 1. Wear rubber gloves to protect the nurse from all exposure. 2. Allow any visitor the client wishes to see. 3. Minimize the amount of time spent with the client. 4. Encourage the client to ambulate in the hallway.

ANSWER: 3. 1. Rubber gloves should be worn to dispose of any soiled material, but they will not protect the nurse from sealed radiation sources. 2. Visitors who may be pregnant or who are younger than 18 years old should not be allowed in the client's room. 3. Afterload intracavitary radiation therapy treatments are completed in the client's room after prepared applicators are placed in surgery. This minimizes exposure of the health-care workers to radiation. The nurse should plan care to minimize exposure to the client and the radiation. 4. The client is placed in a room at the far end of the hallway, and in some facilities clients on either side of the client's room may have to be moved to limit exposure to radiation. The client is not allowed to leave the room until the radiation safety department clears the client for discharge

The client has had an exploratory laparotomy to remove an ovarian tumor. The pathology report classifies the tumor as a "low malignancy potential" (LMP) tumor. Which statement explains the scientific rationale for this pathology report? 1. The client does not have cancer but will need adjuvant therapy. 2. The client would have developed cancer if the tumor had not been removed. 3. These borderline tumors resemble ovarian cancer but have better outcomes. 4. The client has a very poor prognosis and has less than six (6) months to live.

ANSWER: 3. 1. The client has a low-grade cancer occurring in approximately 15% of ovarian tumors. The affected ovary usually is removed, and the client may or may not require adjuvant therapy. Women with this type of tumor are usually younger than age 40 years. 2. The tumor is classified as cancer. The followup care is not as extensive because of the characteristics the tumor displays. 3. These tumors are low-grade cancers with fewer propensities for metastasis than most ovarian cancers. 4. This client has a better prognosis than 85% of clients diagnosed with ovarian cancer. TEST-TAKING HINT: The test taker could eliminate option "4" because "low malignancy potential" and "poor prognosis" do not match. The statement in option "1" says the client does not have cancer but will need therapy for cancer, so the test taker could eliminate this option.

The client diagnosed with endometriosis experiences pain rated a "5" on a 1-to-10 pain scale during her menses. Which intervention should the nurse teach the client? 1. Teach the client to take a stool softener when taking morphine, a narcotic. 2. Instruct the client to soak in a tepid bath for 30 to 45 minutes when the pain occurs. 3. Explain the need to take the nonsteroidal anti-inflammatory drugs with food. 4. Discuss the possibility of a hysterectomy to help relieve the pain.

ANSWER: 3. 1. The client taking a narcotic medication should be placed on a bowel regimen, but this client would not be prescribed narcotic medication. 2. A tepid bath for 30 to 45 minutes is not appropriate because the lukewarm water gets cold. A heating pad to the abdomen sometimes helps with the pain. 3. The medication of choice for mild to moderate dysmenorrhea is an NSAID. NSAIDs cause gastrointestinal upset and should be taken with food. 4. This may be an option eventually, but the stem did not give an age nor state the client has decided she does not want to get pregnant. TEST-TAKING HINT: The test taker should not read into the question. Option "4" is only correct when more information is provided. The test taker must know about the scales used to rate pain, nausea, or depression. The client's report of midrange symptoms does not indicate the need for routine narcotic administration

Which vaccination should the nurse recommend to the postpubertal male to prevent orchitis? 1. Yearly flu injections. 2. Herpes varicella inoculations. 3. Mumps vaccination. 4. Rubella injections

ANSWER: 3. 1. The flu vaccine will not prevent orchitis. 2. This is the chickenpox vaccine and will not prevent orchitis. 3. When postpubertal males contract mumps, one (1) in five (5) develops some form of orchitis (infection of the testes) within four (4) to seven (7) days after the neck and jaw swell. The testes may show atrophy after the infection, and the client may become sterile. 4. Rubella (measles) does not cause orchitis

LOOK The 32-year-old client diagnosed with an ovarian cyst is complaining of acute pain. Which IS the area where the client would be experiencing pain? 1. Midabdominal area 2. RUQ 3. Lower abdomen 4. LUQ

ANSWER: 3. 1. The midabdominal area is where clients diagnosed with peptic ulcers complain of pain. 2. The right upper quadrant is where clients with gallbladder problems complain of pain. 3. The lower abdominal quadrants are where a client with an ovarian cyst will complain of pain. The appendix is also located in the right lower quadrant. 4. The left upper abdominal quadrant is where clients with spleen infarcts or rupture, stomach problems, or pancreas problems complain of pain

The client diagnosed with ovarian cancer is prescribed radiation therapy for regional control of the disease. Which statement indicates the client requires further teaching? 1. "I will not wash the marks off my abdomen." 2. "I will have a treatment every day for six (6) weeks." 3. "Nausea caused by radiation therapy cannot be controlled." 4. "I need to drink a nutritional shake if I don't feel like eating."

ANSWER: 3. 1. The radiation markings on a client are there to guide the technician to irradiate only the area within the marks. The marks must remain until the client has completed the treatments. 2. Radiation therapy is administered in fractionated (divided) doses to allow for regeneration of normal cells. Cancer cells do not regenerate as rapidly as normal cells. 3. There are many medications prescribed for cancer or treatment-induced nausea. The client should notify the HCP if adequate relief is not obtained. 4. Cancer treatments frequently interfere with the client's appetite, but supporting the nutritional status of the client is important. TEST-TAKING HINT: The question is an "except" question. All options except one (1) will be statements indicating the client does understand the teaching. If the test taker missed the information making this an "except" question, finding two (2) options with correct answers might clue the test taker to reread the stem.

The client has been diagnosed with cancer of the breast. Which referral is most important for the nurse to make? 1. The hospital social worker. 2. CanSurmount. 3. Reach to Recovery. 4. I CanCope

ANSWER: 3. 1. The social worker assists clients in finding nursing home placement and financial arrangements and does some work with clients to discuss feelings, but this is not the best referral. 2. CanSurmount volunteers work with all types of clients diagnosed with cancer, not just clients with breast cancer. 3. Reach to Recovery is a specific referral program for clients diagnosed with breast cancer. 4. I CanCope is a cancer education program for all clients diagnosed with cancer and their significant others. TEST-TAKING HINT: The question asks for the most appropriate referral, and the test taker should choose the one specific to breast cancer.

The nurse and an unlicensed assistive personnel (UAP) are caring for clients on a gynecology surgery floor. Which intervention cannot be delegated to the UAP? 1. Empty the indwelling catheter on the three (3)-hour postoperative client. 2. Assist the client who is two (2) days posthysterectomy to the bathroom. 3. Monitor the peri-pad count on a client diagnosed with fibroid tumors. 4. Encourage the client who is refusing to get out of bed to walk in the hall.

ANSWER: 3. 1. The unlicensed assistive personnel can empty the indwelling catheter and record the output. 2. This is an appropriate assignment. 3. Monitoring a peri-pad count is done to determine if the client is bleeding excessively; the nurse should do this as part of the assessment. 4. All personnel should encourage the client to ambulate. TEST-TAKING HINT: The nurse cannot delegate assessment. "Monitor" is a word interpreted as "assess." This is an "except" question, so the test taker should not jump to option "1" as the correct answer because the UAP can perform this task.

The nurse enters the room of a 24-year-old client diagnosed with testicular cancer. The fiancée of the client asks the nurse, "Will we be able to have children?" Which is the nurse's best response? 1. "Your fiancée will be able to father children like always." 2. "You will have to adopt children because he will be sterile." 3. "You and he should consider sperm banking prior to treatment." 4. "Have you discussed this with your fiancée? I can't discuss this with you."

ANSWER: 3. 1. The usual treatment for testicular cancer is removal of the involved testicle followed by radiation to the area and chemotherapy. Every attempt is made to shield the remaining testicle from the radiation, but sterility sometimes occurs. 2. With artificial insemination the client may be able to father children, if the client has banked his sperm. 3. Sperm banking will allow the client to father children through artificial insemination with the client's sperm. 4. The nurse is in the client's room. The client's presence implies consent for the nurse to discuss his case with the fiancée. TEST-TAKING HINT: The nurse must abide by HIPAA rules. It is important for the nurse to know and understand the confidentiality laws. Options "1" and "2" are opposites, so only one could be correct or neither may be correct. Option "3" actually gives an option that lies between the extremes.

What intervention should the nurse implement for a client diagnosed with a rectocele? 1. Limit oral intake to decrease voiding. 2. Encourage a low-residue diet. 3. Administer a stool softener daily. 4. Arrange for the client to take sitz baths

ANSWER: 3. 1. There is no reason to limit oral intake or to decrease voiding. 2. The client should be eating a high-fiber diet to prevent constipation. 3. Stool softeners and laxatives are used to prevent and treat constipation, which is common with a rectocele. Because of the positioning of the rectum, stool can stay in the rectal pouch, causing constipation. 4. Sitz baths are not used to treat a rectocele. TEST-TAKING HINT: The test taker who is knowledgeable of medical terminology could eliminate option "1," which has the word "voiding," because "rectocele" refers to the rectum.

The nurse is performing the admission assessment on a 78-year-old female client and observes bilateral pendulous breasts with a stringy appearance. Which intervention should the nurse implement? 1. Request a mammogram. 2. Notify the HCP of the finding. 3. Continue with the examination. 4. Assess for peau d'orange skin

ANSWER: 3. 1. These are normal findings in a postmenopausal breast and do not require a mammogram. The client should have a mammogram yearly. 2. These are normal findings in the postmenopausal breast so there is no need to notify the HCP. 3. These are normal findings in the postmenopausal breast. Glandular tissue is replaced with fibrous tissue, the breasts become pendulous, and the Cooper's ligaments become prominent. 4. Peau d'orange skin occurs in advanced breast cancer

The client has had a total abdominal hysterectomy for cancer of the ovary. Which diet should the nurse discuss when providing discharge instructions? 1. A low-residue diet without seeds. 2. A low-sodium, low-fat diet with skim milk. 3. A regular diet with fruits and vegetables. 4. A full liquid-only diet with milkshake supplements.

ANSWER: 3. 1. This diet is appropriate for a client diagnosed with diverticulitis. 2. This diet applies to a client with coronary artery disease and hypertension. 3. The client is not placed on a specific diet, but it is always a good recommendation to include fruits and vegetables in the diet. 4. There is no reason to limit the consistency of the foods consumed to full liquids. TEST-TAKING HINT: The test taker should recognize option "3" as a recommended diet for all clients without a specific disease process limiting the types of foods consumed.

The client is diagnosed with metastatic prostate cancer to the bones. Which nursing intervention should the nurse implement? 1. Prepare for a transurethral resection of the prostate. 2. Keep the foot of the bed elevated at all times. 3. Place the client on a scheduled bowel regimen. 4. Discuss the client's altered sexual functioning

ANSWER: 3. 1. This intervention addresses the prostate cancer but not the metastatic process of bone involvement. 2. There is no reason to keep the foot of the bed elevated. 3. Bone metastasis is very painful, and the client should be placed on a scheduled regimen of pain medication. Pain medication slows peristalsis and causes constipation. The client should be placed on a routine bowel management program to prevent impactions. 4. This does not address the metastasis to the bone. TEST-TAKING HINT: The test taker must decide which intervention addresses both the cancer and the metastasis to the bone. Only one (1) option does this.

The nurse is preparing the care plan for a 45-yearold client who has had a radical prostatectomy. Which psychosocial and physiological problem should be included in the plan? 1. Altered coping. 2. High risk for hemorrhage. 3. Sexual impotence. 4. Risk for electrolyte imbalance.

ANSWER: 3. 1. This is a psychosocial problem. 2. This is a physiological problem. 3. This problem has both physiological and psychosocial implications. 4. This is a physiological problem. TEST-TAKING HINT: The test taker should read the stem carefully. The stem asks for a physiological and psychosocial problem. Options "1," "2," and "4" can be sorted into only one of the categories.

The client frequently finds lumps in her breasts, especially around her menstrual period. Which information should the nurse teach the client regarding breast self-care? 1. This is a benign process, which does not require follow-up. 2. The client should eliminate chocolate and caffeine from the diet. 3. The client should practice breast selfexamination monthly. 4. This is the way breast cancer begins and the client needs surgery

ANSWER: 3. 1. This is symptomatic of benign fibrocystic disease, but follow-up is always needed if the lumps do not go away when the hormone levels change. 2. Some practitioners suggest eliminating caffeine and chocolate from the diet if the breasts become tender from the changes, but there is no research supporting this to be effective in controlling the discomfort associated with fibrocystic breasts. 3. The American Cancer Society no longer recommends breast self-examination (BSE) for all women, but it is advisable for women with known breast conditions to perform BSE monthly to detect potential cancer. 4. The client may need a breast biopsy for potential breast cancer at some point, but breast cancer develops when there is an alteration in the DNA of a cell. TEST-TAKING HINT: The test taker could eliminate option "1" because of the clause "does not require follow-up." The question is asking about self-care, and only two (2) options—"2" and "3"—involve the client doing something. The test taker should choose between these.

Which recommendation is the American Cancer Society's (ACS) 2015 guideline for the early detection of breast cancer? 1. Beginning at age 18, have a biannual clinical breast examination by an HCP. 2. Beginning at age 30, perform monthly breast self-exams. 3. At age 45 through 54, receive a yearly mammogram. 4. Beginning at age 50, have a breast sonogram every five (5) years.

ANSWER: 3. 1. Unless there is a personal history of breast cancer or a strong family history, clinical breast examinations should begin at age 30 years and should be performed yearly. 2. If the client is going to perform breast selfexamination (BSE), it should begin at age 18. The ACS no longer includes monthly BSE as part of its guidelines. 3. The ACS recommends a yearly mammogram for the early detection of breast cancer beginning at age 45 and going to age 55 and approximately every two (2) years after the age of 55. Before age 45 and after age 55 it should be a discussion between the woman and her HCP to determine if more frequent mammograms are warranted. A mammogram can detect disease that will not be large enough to feel. 4. Breast sonograms are performed to diagnose specific breast disease when a screening mammogram has shown a suspicious area. TEST-TAKING HINT: This is a knowledge-based question. The test taker might be swayed by the option about BSE, but the age must be considered.

The nurse is formulating a care plan for a client post-abdominal hysterectomy. Which nursing diagnosis is appropriate for the client who has developed a complication? 1. Potential for urinary retention. 2. Potential for nerve damage. 3. Potential for intestinal obstruction. 4. Potential for fluid imbalance

ANSWER: 3. 1. Urinary retention is a complication of a vaginal hysterectomy. 2. Nerve damage is a possible complication of improper positioning during surgery, but the client is "post" surgery. 3. Clients who have had a total abdominal hysterectomy are at risk for intestinal obstruction. 4. An imbalance of fluid is a complication of several surgeries, not specifically a total abdominal hysterectomy. TEST-TAKING HINT: When the test taker does not know the answer, the test taker should consider the similarities and the differences in surgeries. A total abdominal hysterectomy has complications similar to all types of abdominal surgeries.

The nurse is assessing a male client for symptoms of gonorrhea. Which data support the diagnosis? 1. Presence of a chancre sore on the penis. 2. No symptoms. 3. A CD4 count of less than 200. 4. Pain in the testes and scrotal edema.

ANSWER: 4. 1. A chancre sore is a symptom of syphilis, not gonorrhea. 2. Gonorrhea is more likely to be asymptomatic in females. 3. A CD4 count of less than 200 is a diagnostic indicator for acquired immunodeficiency syndrome (AIDS). 4. Pain in the testes and scrotal edema can indicate epididymitis, an inflammatory process of the epididymis. This and urethritis are the most common presenting symptoms in a male with gonorrhea. TEST-TAKING HINT: Two (2) answer options mention male anatomy. If the test taker did not know the information, then choosing between these two (2) options might be the appropriate method of elimination.

The nurse is teaching the client diagnosed with uterine prolapse. Which information should the nurse include in the discussion? 1. Increase fluids and daily exercise to prevent constipation. 2. Explain there is only one acceptable treatment for uterine prolapse. 3. Instruct the client to visually check the uterine prolapse daily. 4. Discuss limiting coughing and lifting heavy objects. 23. The nurse is

ANSWER: 4. 1. A uterine prolapse is not caused by constipation; it is caused by a weakening of the pelvic muscles. It is a protrusion of the uterus through the vagina. It can pull on the vaginal wall, bladder, and rectum. 2. There are multiple treatment modalities for uterine prolapse. The selection of treatment is determined by the degree of the prolapse and the medical history of the client. 3. The protrusion from the vagina can be seen in some cases, but this is not so with all clients. 4. Symptoms can be aggravated by coughing, sneezing, lifting heavy objects, standing for prolonged periods, and climbing stairs. TEST-TAKING HINTS: The test taker should eliminate options containing words such as "only," "never," "always," "all," or "most of the time." If the test taker does not have a clue, "uterine" is not in the same body system as constipation, so option "1" can be eliminated.

The nurse is caring for a young adult client who has been diagnosed with gonorrhea. Which statement reflects an understanding of the transmission of sexually transmitted diseases? 1. Only lower socioeconomic income people are at risk for gonorrhea and syphilis. 2. The longer a client waits to become sexually active, the greater the risk for an STD. 3. Females can transmit infectious diseases more rapidly than males. 4. If a client is diagnosed with an STD, the client should be evaluated for other STDs.

ANSWER: 4. 1. All socioeconomic levels of clients contract STDs. 2. The longer the client abstains from sexual activity and the fewer partners the client has, the less the risk of an STD. 3. Females and males can spread STDs equally. Specific diseases may be asymptomatic in the sexes (in females, gonorrhea; in males, chlamydia) and they can transmit them unknowingly. 4. If the client has one STD, there is a great likelihood the client has another disease also. If one STD is found, the client should be monitored for others. TEST-TAKING HINT: Option "2" does not make sense: If sexual activity is put off, there cannot be an increased risk. Socioeconomic reasons may be a reason for delaying treatment of a disease, but diseases are not financially based and occur in all socioeconomic levels.

The client diagnosed with ovarian cancer has had eight (8) courses of chemotherapy. Which laboratory data warrant immediate intervention by the nurse? 1. Absolute neutrophil count of 3,500. 2. Platelet count of 150 103 . 3. Red blood cell count of 5 106 . 4. Urinalysis report of 100 WBCs.

ANSWER: 4. 1. An absolute neutrophil count of 3,500 indicates the client has sufficient mature white blood cells, or granulocytes, to act as a defense against infections. 2. A platelet count of 150,000 is within normal range (150 103 [1,000] 150,000). Thrombocytopenia is less than 100,000. 3. A red blood cell count of 5,000,000 is within normal limits (5 106 [1,000,000] 5,000,000). 4. A normal urinalysis contains one (1) to two (2) WBCs. A report of 100 WBCs indicates the presence of an infection. A clean voided specimen should be obtained and a urine culture should be done. This client should be prescribed antibiotics immediately. TEST-TAKING HINT: The test taker should memorize normal values for common laboratory tests. Urine will not have a large number of white blood cells unless there is a pathological process occurring. The kidneys filter the blood but do not process the destruction of blood cells.

The nurse is discharging a client diagnosed with pelvic inflammatory disease (PID). Which statement by the client indicates an understanding of the discharge instructions? 1. "I should expect pelvic pain after intercourse." 2. "I need to douche every day to prevent PID." 3. "I will have a vaginal examination every two (2) years." 4. "My partner should use a condom if he is infectious."

ANSWER: 4. 1. Any pelvic pain after sexual exposure, childbirth, or pelvic surgery should be evaluated as soon as possible. 2. Douching reduces the natural flora, which combats infecting organisms and may help infecting bacteria move upward into the uterus, but douching will not prevent PID. 3. The client should have a vaginal examination at least once a year. 4. The client and partner should consistently use a condom if there is any chance of transmission of any organism.

Which tumor marker information is used to follow the progress of a client diagnosed with testicular cancer? 1. CA-125. 2. Carcinogenic embryonic antigen (CEA). 3. DNA ploidy test. 4. Human chorionic gonadotropin (hCG).

ANSWER: 4. 1. CA-125 is a tumor marker used for a number of cancers but not for testicular cancer. 2. CEA is a nonspecific tumor marker used for a number of cancers, but it does not apply to testicular cancer. 3. The DNA ploidy test is used to determine the number of chromosomes and arrangement of chromosomes in a tumor cell. It is used for prognosis in some cancers, such as breast cancer, but not as a tumor marker to determine response to treatment. 4. Tumor markers are substances synthesized by the tumor and released into the bloodstream. They can be used to follow the progress of the disease. Testicular cancers secrete hCG and alpha-fetoprotein. TEST-TAKING HINT: This is a knowledge-based question. If the test taker were aware of what the test tells the HCP, then the DNA ploidy test could be eliminated. The other options are all tumor markers.

The nurse is teaching a class to older women concerning cancer of the uterus. Which situation is a risk factor for developing endometrial cancer? 1. Age 40 years or younger. 2. Perimenopausal bleeding. 3. Progesterone given with estrogen. 4. Truncal obesity

ANSWER: 4. 1. Clients at risk for uterine cancer are usually 55 years old or older. 2. Postmenopausal bleeding places the client at risk, but perimenopausal bleeding is expected as the ovaries begin to slow production of eggs. 3. Unopposed estrogen replacement therapy predisposes women to developing uterine cancer, but progesterone offsets the risk. 4. Truncal obesity is one of the risk factors for developing endometrial cancer, although it has not been determined how this occurs

The client diagnosed with gestational diabetes delivered a 10-pound 5-ounce infant. Which is priority for the nursery nurse to monitor? 1. Failure to latch on to the breast during feeding. 2. Jaundice and clay-colored stools. 3. Parchment-like skin and lack of lanugo. 4. Low blood glucose readings.

ANSWER: 4. 1. Failure of the baby to latch onto the breasts is not the priority for the nurse to assess. 2. There is nothing that indicates this baby will have jaundice and/or clay-colored stools. 3. The baby would have parchment-like skin and lack lanugo if the baby were over 40 weeks gestation. This is not stated in the stem. 4. The neonate is high birth weight and the mother had gestational diabetes. This infant had a high glucose content passing through the placenta in utero and the infant's pancreas has been producing insulin to take care of the glucose content of the blood. The infant's pancreas must adjust to lower levels of glucose in the system. TEST-TAKING HINT: The test taker could eliminate option "2" because this is associated with liver or gallbladder issues and a basic knowledge of the pathophysiology of post-maturity gestation could eliminate option "3."

The nurse is teaching a class on breast health to a group of ladies at a senior citizen's center. Which risk factor is the most important to emphasize to this group? 1. The clients should find out about their family history of breast cancer. 2. Men at this age can get breast cancer also and should be screened. 3. Monthly breast self-examination is the key to early detection. 4. The older a woman gets, the greater the chance of developing breast cancer.

ANSWER: 4. 1. Most women who develop breast cancer do not have a family history of the disease. Specific genes—BRCA-1 and BRCA-2— implicated in the development of breast cancer have been identified, but most women with breast cancer do not have these genes. 2. Approximately 1,000 men are diagnosed every year with breast cancer, but, as with women, it can occur at any age. Breast cancer in men frequently goes undetected because men consider this a woman's disease. 3. Mammograms can detect breast cancer earlier than breast self-examination and are the current recommendation by the American Cancer Society. 4. The greatest risk factor for developing breast cancer is being female. The second greatest risk factor is being elderly. By age 80, one (1) in every eight (8) women develops breast cancer. TEST-TAKING HINT: The test taker cannot overlook the age when it is given in a question. "Senior citizen's center" should alert the test taker to the older age group. The test taker should decide what the age has to do with the answer.

The client is eight (8) hours post-transurethral prostatectomy for cancer of the prostate. Which nursing intervention is priority at this time? 1. Control postoperative pain. 2. Assess abdominal dressing. 3. Encourage early ambulation to prevent DVT. 4. Monitor fluid and electrolyte balance.

ANSWER: 4. 1. Pain does not have priority over a fluid and electrolyte imbalance. 2. There is no dressing for a transurethral resection. The body cavity is entered through the penis. 3. Early ambulation prevents several complications, but the immediate complication is electrolyte imbalance and fluid overload. 4. With irrigation of the surgical site through the indwelling three (3)-way catheter to prevent blood clots, fluids may be absorbed through the open surgical site and retained. This can lead to fluid volume overload and electrolyte imbalance (hyponatremia). TEST-TAKING HINT: This client is eight (8) hours postoperative, so the test taker could eliminate preventing DVT (option "3"). Option "2" could be eliminated if the test taker looks at the surgical approach described in the procedure. Pain is a priority, but the test taker should read to see if another option has a higher priority.

Which intervention should the nurse include when teaching the client who is having an anterior colporrhaphy to repair a cystocele? 1. Discuss the need to perform perineal care every four (4) hours. 2. Discuss the care of an indwelling catheter for at least one (1) month. 3. Instruct the client how to care for the pessary inserted in surgery. 4. Teach the client how to perform Kegel exercises.

ANSWER: 4. 1. Perineal care is given every shift or as needed. 2. The client may have an indwelling catheter for two (2) to four (4) days postoperative but not for a month. 3. A pessary is used in place of surgery. 4. The client should be taught how to perform Kegel exercises to strengthen the muscles. TEST-TAKING HINT: When selecting a correct answer, the test taker should always look at the adjectives, especially numbers such as four (4) in option "1" or one (1) in option "2."

Which statement indicates further instruction is needed for the client with a cystocele? 1. "I need to have a sonogram to diagnose this problem." 2. "I need to practice Kegel exercises to help strengthen my muscles." 3. "I lose my urine when I sneeze because of my cystocele." 4. "I can never have sexual intercourse again."

ANSWER: 4. 1. Sonograms and pelvic examinations are used to diagnose cystoceles. 2. Kegel exercises strengthen the pelvic floor muscles. 3. Clients with cystoceles frequently have urinary incontinence when they cough, sneeze, laugh, lift heavy items, or make sudden jarring motions. 4. Clients with cystoceles may have sexual intercourse unless contraindicated by another medical reason. TEST-TAKING HINT: This type of question is confusing to test takers because the correct option provides incorrect information. Most of the time absolutes such as "never," "only," and "always" make the option incorrect. Option "4" has an absolute—"never"—but it is the correct answer because all of the other options contain correct information the client has learned.

The 24-year-old female client presents to the clinic with lower abdominal pain on the left side she rates as a "9" on a 1-to-10 scale. Which diagnostic procedure should the nurse prepare the client for? 1. A computed tomography scan. 2. A lumbar puncture. 3. An appendectomy. 4. A pelvic sonogram

ANSWER: 4. 1. The client has symptoms of an ovarian cyst, usually diagnosed by a pelvic sonogram. 2. The client has abdominal pain, not back or neurological pain, which is when a lumbar puncture is performed. 3. The appendix is in the right lower abdomen, not the left. 4. Ovarian cysts are fluid-filled sacs located on the surface of the ovary. A lower pelvic sonogram is the preferred diagnostic tool. It is not invasive and usually not painful. TEST-TAKING HINT: The test taker could eliminate options "2" and "3" by using knowledge of basic anatomy and physiology. The age of the client places this client in the typical age range for a benign ovarian cyst; before age 29 years, 98% of ovarian cysts are benign.

The HCP has prescribed two (2) IV antibiotics for the female client diagnosed with diabetes and pneumonia. Which order should the nurse request from the HCP? 1. Request written information on antibioticcaused vaginal infections. 2. Request yogurt to be served on the client's meal trays. 3. Request a change of one of the antibiotics to an oral route. 4. Request L. acidophilus, a probiotic medication, three (3) times a day.

ANSWER: 4. 1. The nurse does not require an order to teach. Teaching is an independent nursing function. 2. The nurse can request the dietitian to include yogurt in the client's calorie restrictions without an order. 3. If the HCP has ordered an IV antibiotic, then there is no reason to request a change to an oral route. 4. Female clients on antibiotics are at risk for killing the good bacteria, which keep yeast infections in check. This is especially true in clients diagnosed with diabetes. Lactobacillus acidophilus is a yeast replacement medication. TEST-TAKING HINT: The test taker must be aware of independent nursing functions. This eliminates options "1" and "2."

The nurse writes a client problem of urinary retention for a client diagnosed with Stage IV cancer of the prostate. Which intervention should the nurse implement first? 1. Catheterize the client to determine the amount of residual. 2. Encourage the client to assume a normal position for urinating. 3. Teach the client to use the Valsalva maneuver to empty the bladder. 4. Determine the client's normal voiding pattern.

ANSWER: 4. 1. The nurse should assess the client's normal voiding pattern before taking any action. 2. This is a good intervention, but it comes after assessment. 3. The Valsalva maneuver will help the client to empty the bladder, but assessment comes before teaching. 4. Determining the client's normal voiding patterns provides a baseline for the nurse and client to use when setting goals. TEST-TAKING HINT: Assessment is the first step of the nursing process. In any question requiring the test taker to choose the first action, an answer option with the words "check," "assess," or "determine" should be considered as a possible answer.

The postmenopausal client reveals it has been several years since her last gynecological examination and states, "Oh, I don't need exams anymore. I am beyond having children." Which statement should be the nurse's response? 1. "As long as you are not sexually active, you don't have to worry." 2. "You should be taking hormone replacement therapy now." 3. "You are beyond bearing children. How does that make you feel?" 4. "There are situations other than pregnancy that should be checked."

ANSWER: 4. 1. This client is at risk for cancer of the ovary and uterus because of advancing age, regardless of sexual activity, and should see an HCP yearly. 2. Hormone replacement therapy (HRT) is not recommended for most postmenopausal clients because research has shown HRT increases the risk of myocardial infarctions and cerebrovascular accidents (strokes). 3. This is a therapeutic response and the client did not state a feeling. 4. The client should have a yearly clinical examination of the breasts and pelvic area for the detection of cancer. TEST-TAKING HINT: If the stem is not asking for a therapeutic response, then factual information should be provided to the client. This eliminates option "3" as a possible answer.

The county health department nurse is reviewing a client record and notes the RPR laboratory results as being reactive for Treponema pallidum. Which question should the nurse ask the client? 1. "When was your last tetanus shot?" 2. "Have you had a cold recently?' 3. "Do you have diabetes mellitus?" 4. "Are you allergic to penicillin?"

ANSWER: 4. 1. This is a test for syphilis and does not require a tetanus injection. 2. Having a cold does not affect the diagnosis of syphilis. 3. Diabetes places a client at risk for many illnesses, but sexual behavior places the client at risk for a sexually transmitted disease. 4. The test is positive for syphilis, and the nurse anticipates the HCP ordering an antibiotic; penicillin is the antibiotic of choice for syphilis. Syphilis is caused by the Treponema pallidum bacteria.

The client is diagnosed with early cancer of the prostate. Which assessment data would the client report? 1. Urinary urgency and frequency. 2. Retrograde ejaculation during intercourse. 3. Low back and hip pain. 4. No problems have been noticed.

ANSWER: 4. 1. Urgency and frequency are obstructive symptoms and are late signs. 2. Retrograde ejaculation occurs when the sperm are ejaculated into the urine; it occurs in some cases of male infertility. 3. Low back pain and hip pain are symptoms of metastasis to the bone and are late symptoms. 4. In early-stage prostate cancer, the man will not be aware of the disease. Early detection is achieved by screening for the cancer. TEST-TAKING HINT: The test taker should notice the word "early" in the stem of the question and choose the option with the least amount of symptoms.

The nurse is conducting a reproductive assessment of a young adult client. What assessment questions does the nurse ask? (Select all that apply). A) "Have you had any sexually transmitted diseases?" B) "How would you describe yourself?" C) "If you engage in sexual activities, do you practice 'safe' sex?" D) "What changes would you like to see in your appearance?" E) "When did you first start menstruating?"

Answer: A, B, C, E. Asking the client about a history of sexually transmitted diseases is a question included in the health perception/health management pattern for performing a reproductive assessment. If the answer is "yes," the nurse continues with "When?" and "What type?" Asking the client to describe himself or herself is also included in the self-perception/self-concept pattern. The nurse follows with "Do you feel good or not-so-good about yourself?" It is important to note, if the client is sexually active, that he or she practices (and understands) "safe" sex. This might include the use of condoms, being tested for human immunodeficiency virus, and other measures to keep from acquiring sexually transmitted diseases. The age of onset of menses in women is important to note. Either early or late onset may indicate a problem or the increased likelihood for one to develop.Although the nurse might inquire whether a client has experienced changes in his or her body appearance or function, asking about changes the client might want to see is not important in doing a reproductive assessment.

A client wants to know what complications may occur after a phalloplasty. About which complications does the nurse inform the client? (Select all that apply). A) Dissatisfaction with results B) Donor graft site scarring C) Penile necrosis D) Prostate cancer E) Urinary tract stenosis

Answer: A, B, C, E. Phalloplasty is one of the most difficult reconstructive genital surgeries to perform, and many female-to-male (FtM) clients do not choose to have phalloplasty surgery due to its low success/satisfaction rate. Donor graft scarring and urinary tract stenosis are possible. Although not a common occurrence, necrosis of the neopenis may occur.(FtM) clients do not have a prostate gland.

A client with benign prostatic hyperplasia is being discharged with alpha-adrenergic blockers. Which information is important for the nurse to include when teaching the client about this type of pharmacologic management? (Select all that apply). A) Avoid drugs used to treat erection problems. B) Be careful when changing positions. C) Keep all appointments for follow-up laboratory testing. D) Hearing tests will need to be conducted periodically. E) Take the medication in the afternoon.

Answer: A, B, C. Drugs used to treat erectile dysfunction can worsen side effects, such as hypotension. Alpha-adrenergic blockers may cause orthostatic hypotension and can cause liver damage. The nurse needs to remind the client to be careful when changing positions and to keep all appointments for follow-up laboratory testing.These drugs do not affect hearing. Alpha-adrenergic blockers need to be taken in the evening to decrease the risk of problems related to hypotension.

A client is preparing to be discharged home after her gender reassignment surgery. What are the key points that the nurse will include in her discharge teaching? (Select all that apply). A) "The drain will be removed in about 3 to 5 days." B) "You will need to douche routinely to prevent infection." C) "Place the stents or dilators several times a day." D) "Do not engage in sexual intercourse for at least 6 weeks." E) "When you insert the dilators, remember to use petroleum jelly."

Answer: A, B, C. The Jackson-Pratt drain will be removed in approximately 3 to 5 days when the drainage is less than 15 to 20 mL/24 hours. Douching routinely with a vinegar and water solution will help prevent infection. Stents or dilators will need to be inserted using a water-based lubricant (not petroleum jelly) several times per day and left in place for 30 to 45 minutes. This will need to continue for several months.Sexual intercourse is also important to keep the vagina dilated. In general, there is no need to refrain from sexual intercourse unless directed by the surgeon. Petroleum jelly is not used when placing the stents or dilators.

A client with prostate cancer asks the nurse for more information and counseling. Which resources does the nurse suggest? (Select all that apply). A) American Cancer Society's Man to Man program B) Us TOO International C) Save My Prostate Society D) National Prostate Cancer Coalition E) Client's church, synagogue, or place of worship

Answer: A, B, D, E. The American Cancer Society's Man to Man program helps the client and partner cope with prostate cancer by providing one-on-one education, personal visits, education presentations, and the opportunity to engage in open and candid discussions. Us TOO International is a prostate cancer education and support group that is sponsored by the Prostate Cancer Education and Support Network. The National Prostate Cancer Coalition provides prostate cancer information. The client's church, synagogue, or place of worship is a community support service that may be important for many clients.There is no such organization as the Save My Prostate Society.

The nurse is planning care for a client who has a sexually transmitted disease (STD). Which interventions address the client's psychosocial needs? (Select all that apply). A) Allowing the client to express fears and anxieties B) Approaching the client with a nonjudgmental attitude C) Ensuring that the client's sexual partner is aware of the diagnosis D) Referring the client to the appropriate support groups E) Reporting the STD to the public health department F) Sharing experiences about working with clients with STDs

Answer: A, B, D. Many clients with STDs have fears and anxieties about having such diseases. They are reluctant to share a very private and personal side of their lives. It is therapeutic for a nurse to encourage clients' expressions about these feelings. Nurses must always be completely nonjudgmental about communicating with clients. Support groups can be very helpful to clients with STDs.Nurses have a responsibility to encourage clients with STDs to contact their sexual partners. However, if the client will not disclose whom they may have received the infection from, or to whom they may have given it, the nurse has no means of "ensuring" contact. Reporting the occurrence to a public health agency is a medical responsibility and would not be categorized as meeting the client's psychosocial needs. Nurses do not share personal experiences about their contacts with other clients. This is a breach of confidentiality and entirely inappropriate, as well as both illegal and unethical.

The nurse is teaching a client how to adapt to physical and psychological changes after surgery for ovarian cancer. What is included in the teaching plan? (Select all that apply). A) Encouraging the use of support groups and counseling. B) Encouraging the expression of grief and fears. C) Offering vaginal dilators D) Refraining from sexual intercourse for 6 weeks after surgery. E) Suggesting the use of oil-based lubricants.

Answer: A, B, D. Support groups such as Gilda's Club are advisable for clients with ovarian cancer because the loss of reproductive organs involves a grief reaction. Ovarian cancer particularly carries the connotation of being serious and incurable in the view of many women. The client needs to refrain from sexual intercourse for 6 weeks after surgery.The use of a vaginal dilator is not indicated. After the woman becomes sexually active, she may have a problem with vaginal dryness as the result of hormonal changes. Water-based, rather than oil-based, lubricants would be suggested.

The community health nurse is providing education to a group of women about risks for breast cancer. Which of these risk factors will the nurse include in the education session? (Select all that apply). A) High breast density. B) Nulliparity. C) Male with gynecomastia. D) Middle-aged woman. E) First child at age 25

Answer: A, B. Individuals at high increased risk for breast cancer include women with high breast density as well as nulliparous women.Men are not at high increased risk for breast cancer, but obesity can cause gynecomastia. Being middle-aged and bearing the first child before age 30 does not indicate a high increased risk for breast cancer.

A client who is transitioning from female to male asks the nurse what options are available to make his voice "more real to who he is." What options does the nurse discuss with this client? (Select all that apply). A) Masculinizing hormones such as testosterone B) Voice specialist to help with pitch and intonation C) Surgery to alter the vocal cords D) Adaptation to current vocal pattern E) Human growth hormone

Answer: A, B. Testosterone will cause the voice to deepen, although not immediately. Voice and communication therapists can help a client develop certain voice characteristics such as pitch and intonation.Surgery would not be undertaken on the vocal cords to alter the voice. Telling a client to "learn to adapt" to his vocal pattern is negating his feelings. Human growth hormone is not indicated for gender dysphoria.

The nurse is discussing methods of preventing sexually transmitted diseases (STDs). Which approaches does the nurse suggest for sexually active clients? (Select all that apply). A) Abstinence. B) Spermicidal cream. C) Latex condoms. D) Mutual monogamy. E) Polyurethane condoms.

Answer: A, C, D, E. Abstinence, latex condoms, polyurethane condoms, and mutual monogamy all decrease the risk for acquiring an STD.Spermicidal creams do not provide protection from STDs.

A client is beginning testosterone therapy and asks the nurse what effects are to be expected. What physical changes does the nurse tell the client to anticipate? (Select all that apply). A) Breast atrophy. B) Decreased libido. C) Deepening voice. D) Development of penile tissue. E) Menstruation cessation

Answer: A, C, E. Testosterone will cause breast and uterine tissue to atrophy. The clitoris will also atrophy. The client's voice will become deeper with testosterone therapy. Menstruation will also cease, but the nurse needs to advise the client that these effects may take up to 1 year to occur.Libido will increase with testosterone therapy. A penis will not develop but will require surgical intervention if the client desires.

After returning from transurethral resection of the prostate, the client's urine in the continuous bladder irrigation system is a burgundy color. Which client needs does the nurse anticipate after the surgeon sees the client? (Select all that apply) A) Antispasmodic drugs. B) Emergency surgery. C) Forced fluids. D) Increased intermittent irrigation. E) Monitoring for anemia.

Answer: A, E. Although not a common occurrence, bleeding may occur in the postoperative period. Venous bleeding is more common than arterial bleeding. The surgeon may apply traction on the catheter for a few hours to control the venous bleeding. Traction on the catheter is uncomfortable and increases the risk for bladder spasms, so analgesics or antispasmodics are usually prescribed. Hemoglobin and hematocrit would be monitored and trended for indications of anemia.Emergency surgery and increased intermittent irrigation would be indicated for an arterial bleed, which would be a brighter red color. Forced fluids are indicated after the catheter is removed.

A client has undergone transurethral resection of the prostate (TURP). Which interventions does the nurse incorporate in this client's postoperative care? (Select all that apply). A) Administer antispasmodic medications. B) Encourage the client to urinate around the catheter if pressure is felt. C) Perform intermittent urinary catheterization every 4 to 6 hours. D) Place the client in a supine position with his knees flexed. E) Assist the client to mobilize as soon as permitted.

Answer: A, E. Antispasmodic drugs can be administered to decrease the bladder spasms that may occur due to catheter use. Assisting the client to a chair as soon as permitted postoperatively will help to decrease the risk of complications from immobility. An indwelling catheter and continuous bladder irrigation are in place for about 24 hours after TURP.The client would not try to void around the catheter. This would cause the bladder muscles to contract and may result in painful spasms. Intermittent urinary catheterization is not necessary and increases the risk for infection. Typically, the catheter is taped to the client's thigh, so he needs to keep his leg straight.

The nurse is counseling a client and the client's sexual partner on safe sex practices. Which of the couple's comments indicates a need for further education? A) "Condoms are for vaginal and anal sexual encounters, not for oral sex." B) "If the guy won't wear a condom, the female condom is just as effective." C) "We heard that latex condoms were better than natural membrane condoms." D) "We only use condoms once, and then throw them away."

Answer: A. A condom needs to be used for every sexual encounter, including oral, vaginal, and anal.Female condoms (polyurethane sheaths in the vagina) are effective for preventing the transmission of sexually transmitted diseases, including HIV. Latex or polyurethane condoms need to be used rather than natural membrane condoms. A condom would never be used more than once.

An 18-year-old female is diagnosed with possible toxic shock syndrome (TSS) and has these vital signs: T 103.2°F (39.6°C), P 124 beats/min, R 36 breaths/min, BP 84/30 mm Hg. Which primary health care provider request does the nurse implement first? A) Administer O2 at 6 L/min. B) Give cefazolin (Ancef) 500 mg IV. C) Infuse normal saline IV at 500 mL/hr. D) Obtain blood cultures × 2 sites.

Answer: A. Administration of oxygen would be the nurse's first action. The highest priority action for clients with shock is to maintain adequate gas exchange. Remember the ABCs (Airway, Breathing, Circulation) to assist in answering this question.Infusing normal saline IV at 500 mL/hr and giving cefazolin (Ancef) 500 mg IV will need to be implemented rapidly for this client because she must be on antibiotics to fight infection and sepsis. Obtaining blood cultures will need to be implemented rapidly, prior to administration of antibiotics (cefazolin) for the best opportunity to identify the organism.

The nurse is assessing a client during a routine gynecologic examination. Which comment by the nurse shows the least respect for the client's personal values and beliefs? A) "Have you participated in homosexual relationships?" B) "How many sexual partners do you currently have?" C) "How often do you have intercourse?" D) "What type of protection do you use when you have sex?"

Answer: A. Asking the client if she participates in homosexual relationships is not an appropriate assessment question. The nurse needs to allow opportunities for the client to discuss sexual orientation and gender preference. While it is appropriate to determine the client's sexual orientation during completion of the health history, a more culturally sensitive open-end question is required, such as: "How would you describe your sexual orientation?"Asking the client about how many sexual partners she has is an appropriate assessment question because it gets at certain risk factors for sexually transmitted diseases (STDs) and other sexually related problems. Monogamous clients generally have fewer risk factors. The client's level of sexual activity is also appropriate for the nurse to question because it relates to the number of possible risk factors that the client may have. Asking the client what type of protection she uses is appropriate because it is related to the client's becoming pregnant or to the client's exposure to an STD.

A client is struggling with body image after breast cancer surgery. Which behavior indicates to the nurse that the client's coping is maladaptive? A) Avoiding eye contact with staff B) Saying, "I feel like less of a woman" C) Requesting a temporary prosthesis D) Saying, "This is the ugliest scar ever"

Answer: A. Avoiding eye contact may be an indication of decreased self-image.The client stating that she feels like less of a woman or that her scar is ugly illustrates an expected emotional state. By verbalizing her frustration, the client suggests a willingness to discuss and express feelings. Requesting a prosthesis can be a sign of healing and working through body image changes.

Decreased estriol levels in a pregnant client may be associated with which condition? A) Impending miscarriage B) Gestation of twins C) Infertility D) Normal pregnancy

Answer: A. Decreased levels of estradiol, total estrogens, and estriol in women may indicate impending miscarriage, amenorrhea, climacteric, or hypothalamic disorders.Decreased estriol levels in the pregnant client do not indicate gestation of twins or infertility.

A client who has recently been prescribed finasteride (Proscar) calls the clinic to report having dizziness, cold sweats, and chills. What will the nurse tell this client? A) "These are common effects that usually diminish over time." B) "These effects are not related to the finasteride." C) "Schedule an appointment to discuss these symptoms with your primary health care provider." D) "You need to stop taking the medication immediately."

Answer: A. Dizziness, cold sweats, and chills are common side effects of finasteride that usually diminish over time. Finasteride is a 5-alpha reductase inhibitor that is typically used to treat benign prostatic hyperplasia (BPH). This drug blocks the conversion of testosterone to a more active ingredient to decrease the hair loss associated with estrogen therapy and shrink prostate tissue.It is not necessary for clients to report these side effects to their primary health care provider unless the symptoms persist. It is also not necessary to discontinue the drug, since the side effects are not harmful.

The nurse is assessing a male client who has been diagnosed with Chlamydia trachomatis. What clinical manifestation does the nurse expect to see? A) Dysuria. B) Painless maculopapular rash in the perineal area C) Pustules on the glans of the penis D) Testicular edema.

Answer: A. Dysuria is a clinical manifestation of C. trachomatis in males.Painless maculopapular rash in the perineal area and testicular edema are not clinical manifestations of C. trachomatis. Pustules on the glans of the penis are a clinical manifestation of herpes simplex type 2.

A male-to-female client wishes to discuss breast augmentation surgery. What statement by the client indicates the need for further education by the nurse? A) "Fat can be used instead of implants for a more natural look." B) "I may take hormones for about a year before my surgery." C) "If I take hormones, I will have to get a mammogram." D) "The surgeons will use either silicone or saline implants."

Answer: A. Fat injections are not used as a substitute for implants.The options for breast augmentation include saline and silicone. The use of feminizing hormones for 12 months may yield better results postoperatively but is not mandatory. If hormones are used, once breast tissue enlarges, mammograms would be obtained on a regular basis.

The nurse is reviewing options and providing education for a client who is experiencing gender dysphoria. What statement by the client indicates that further discussion with and education from the health care team is needed? A) "To avoid scrutiny at the pharmacy, I'll buy my hormones on the Internet." B) "I could live as the other sex full time if I want to." C) "Surgery is an option to change my breasts and face." D) "Talking to a psychotherapist might help me understand my identity and strengthen my coping skills."

Answer: A. Gender dysphoria is a mismatch or incongruence between one's gender identity and natal sex, causing discomfort in the individual. This feeling of mismatch can often extend back into early childhood. The use of hormones to feminize or masculinize the body is an option for a client experiencing gender dysphoria. Their use needs to be prescribed and monitored by primary health care providers, and frequently endocrinologists are part of the multidisciplinary team. Products purchased via the Internet are from an uncontrolled source and may be hazardous.Individuals experiencing gender dysphoria may live either full- or part-time as the other gender. If the client desires a more permanent change, surgery is an option. Psychotherapy or counseling can provide an outlet to help improve self-image and strengthen coping mechanisms.

A client with newly diagnosed gynecologic cancer is being discharged home. Which health care team member does the nurse contact to coordinate nursing care at home for this client? A) Case manager. B) Primary Health care provider. C) Hospice. D) Social services.

Answer: A. If nursing care is needed at home, the hospital nurse or case manager makes referrals to a home health care agency.The primary health care provider is not the correct team member to coordinate home care. Hospice care is provided for clients who are at the end of their lives. This type of care is not necessary (or indicated) for this client. A referral to a social service agency is needed if the client is unable to meet the financial demands of treatment and long-term care follow-up.

The nurse is evaluating a client's response to antibiotic treatment for pelvic inflammatory disease (PID). Which finding indicates that the treatment is effective? A) Decreased pelvic tenderness. B) Decreased vaginal discharge. C) Increased appetite. D) Increased libido.

Answer: A. Pain management of PID begins with treatment of the infection. Antibiotic therapy relieves pain by decreasing the inflammation caused by infection.Vaginal discharge may be increased at first as the infection drains. Increased appetite and increased libido may be secondary findings but are not indicative of effective treatment.

The nurse is caring for a client with erectile dysfunction who has not had success with common treatment modalities. The nurse anticipates that the primary health care provider will recommend which treatment for this client? A) Penile implants. B) Penile injections. C) Transurethral suppository. D) Vacuum constriction device.

Answer: A. Penile implants (prostheses), which require surgery, are used when other modalities fail. Devices include semirigid, flexible, or hydraulic inflatable and multicomponent or one-piece instruments.Penile injections and transurethral suppositories are tried before using the option of last resort. A vacuum constriction device is easy to use and is often the first option that is tried.

A young adult with testicular cancer is admitted for unilateral orchiectomy and retroperitoneal lymph node dissection. Which nursing action is best for the nurse to delegate to unlicensed assistive personnel (UAP)? A) Encourage the client to cough and deep-breathe after surgery. B) Discuss reproductive options with the client and significant other. C) Teach about the availability of a gel-filled silicone testicular prosthesis. D) Evaluate the client's understanding of chemotherapy and radiation treatment.

Answer: A. Reminding clients to perform coughing and deep-breathing activities can be delegated to UAPs.Client education and evaluation are more complex skills that must be done by licensed nurses.

The nurse is teaching postmastectomy exercises to a client. Which statement made by the client indicates that teaching has been effective? A) "For the pulley exercise, I'll drape a 6-foot (1.8 meter)-long rope over a shower curtain rod or over the top of a door." B) "In rope turning, I'll hold the rope with my arms flexed." C) "In rope turning, I'll start by making large circles." D) "With hand wall climbing, I'll walk my hands up the wall and back down until they are at waist level."

Answer: A. Teaching has been effective when the client says that to perform the pulley exercise properly, the client would drape a 6-foot (1.8 meter)-long rope over a sturdy structure.In rope turning, the client holds the end of the rope and steps back from the door until the arm is almost straight out in front, not flexed. Also in rope turning, the client starts with small circles and gradually increases to larger circles as the client becomes more flexible. With hand wall climbing, the client walks the hands up the wall and then back down until they are at shoulder level, not waist level.

Which statement is true regarding the prostate-specific antigen (PSA) test which is used in screening for prostate cancer? A) The PSA test can be used to monitor the disease after prostate cancer treatment. B) PSA levels less than 7.5 ng/mL (7.5 mcg/L) may be considered normal. C) Elevated PSA levels are diagnostic for prostate cancer. D) Younger men, particularly African Americans, often have a higher normal PSA.

Answer: A. The PSA test is used to screen for prostate cancer and to monitor the disease after cancer treatment.PSA levels less than 2.5 ng/mL (2.5 mcg/L) may be considered normal, although there is no agreement on that value and how it is affected by age. Elevated PSA levels may be associated with prostate cancer. Older men, particularly African-American men, often have a higher normal PSA, especially as they age.

A client who has just been notified that the breast biopsy indicates a malignancy tells the nurse, "I just don't know how this could have happened to me." Which response by the nurse is best? A) "Please tell me what you mean when you say you don't know how this could have happened to you." B) "Do you have a family history that might make you more likely to develop breast cancer?" C) "Would you like me to help you find more information about how breast cancer develops?" D) "Many risk factors for breast cancer have been identified, so it is difficult to determine what might have caused it."

Answer: A. The best response by the nurse is "Please tell me what you mean when you say you don't know how this could have happened to you." It is beneficial to recognize the client's comment and then ask an open-ended question to encourage the client to talk. The client's statement that she does not know how this could have happened may indicate shock and denial.The nurse needs to further assess the client's emotional status before asking about family history of cancer or obtaining information for the client. Telling the client that many risk factors have been determined and that it's difficult to decide what caused the cancer is not therapeutic and does not encourage the client to express her feelings.

The nurse is educating a 22-year-old female about the Papanicolaou (Pap) test. Which client statement indicates that further teaching is needed? A) "I can have sexual intercourse the night before the test." B) "I need to have yearly Pap tests until I am 30 years old." C) "Pap smears help detect precancerous and cancerous cells." D) "The specimen will be sent to a laboratory for evaluation."

Answer: A. The client must not have sexual intercourse for at least 24 hours before the test.Annual screening is recommended to 30 years of age with the conventional Pap test. After age 30 and three or more consecutive negative test results, Pap tests may be performed less frequently until 70 years of age. The Pap smear is a cytologic study that is effective in detecting precancerous and cancerous cells in the cervix. The specimen-containing slides from a Pap smear are sent to a laboratory for evaluation.

A client is receiving chemotherapy treatment for breast cancer and asks for additional interventions for managing the associated nausea and vomiting. Which complementary and integrative therapy does the nurse suggest? A) Ginger. B) Journaling. C) Meditation. D) Yoga.

Answer: A. The complementary and integrative health therapies the nurse suggests to manage nausea and vomiting are: acupuncture, aromatherapy, ginger, hypnosis, progressive muscle relaxation, and shiatsu.Journaling is good for reducing anxiety, stress, and fear. Meditation helps reduce stress, improve mood, improve quality of sleep, and reduce fatigue. Yoga has been shown to improve physical functioning, reduce fatigue, improve sleep, and improve one's overall quality of life.

A client scheduled for a hysterosalpingogram is interviewed by the nurse. What interview information is critical for the nurse to report to the primary health care provider before the procedure? A) Allergy to shellfish B) Abortion 2 months ago C) Menstrual period that ended 3 days ago D) Administration of a rectal suppository 4 hours ago

Answer: A. The contrast medium used during hysterosalpingography is iodine-based, so the primary health care provider will need to know if the client is allergic to shellfish.Obstetric history, menstrual history, and recent medications are communicated to the primary health care provider but do not require any change in the procedure. Two months between an abortion and this procedure is adequate. This test is done just at the completion of menses so that it would not interrupt a pregnancy in the uterus or the fallopian tube.

The nurse is caring for a client undergoing mastectomy who asks the nurse about breast reconstruction. Which of these will the nurse include in the discussion? A) Many women want breast reconstruction using their own tissue immediately after mastectomy. B) Placement of saline or gel-filled prostheses is not recommended because of the nature of the surgery. C) Reconstruction of the nipple-areola complex is the first stage in the reconstruction of the breast. D) The surgeon would offer the option of breast reconstruction surgery once healing has occurred after a mastectomy.

Answer: A. The correct statement about breast reconstruction surgery is, "Many women want breast reconstruction using their own (autogenous) tissue immediately after mastectomy."Saline or gel-filled prostheses are recommended as breast expanders in breast augmentation surgery, not for reconstructive surgery. Reconstruction of the nipple-areola complex is the last stage in breast reconstruction surgery. Breast reconstruction surgery would be discussed before mastectomy takes place.

The RN working in the hospital emergency department is assigned to care for these four clients. Which client does the nurse attend to first? A) Adolescent with an erection for "10 or 11 hours" who is reporting severe pain B) Young adult with a swollen, painful scrotum who has a recent history of mumps infection C) Middle-aged adult discharged 2 days ago after a transurethral resection of the prostate who has increased hematuria D) Older adult with a history of benign prostatic hyperplasia and palpable bladder distention

Answer: A. The nurse first attends to the client who has had an erection for "10 or 11 hours." This client has symptoms of priapism which is considered a urologic emergency because the circulation to the penis may be compromised. With an erect penis, the client may also be unable to void.The client with a swollen, painful scrotum, the client with hematuria, and the client with a history of benign prostatic hyperplasia do not require the nurse's immediate attention.

The nurse is orienting a newly hired unlicensed assistive personnel (UAP) to the clinic. One of the clients has self-identified as transgender. The UAP states "I don't want to say the wrong thing. What do I call him or her?" What is the nurse's best response? A) "Ask how the client would like to be addressed." B) "Just call the client by his or her name." C) "Just be polite and it won't matter." D) "Look at the client's driver's license and use that."

Answer: A. The nurse's best response is to ask the client how he/she would like to be addressed. This comment shows respect and does not make assumptions.Calling the client by his or her name does not address the client's preference. Being polite is a given but may make the client uncomfortable if it is not how he or she prefers to be recognized. A client's given name and sex as listed on a driver's license may not be how the client chooses to be addressed.

A client wishes to begin hormone therapy. What criteria must be met for the client to be eligible? (Select all that apply). A) Over age 25 B) Continued and well-documented gender dysphoria. C) Evaluation by a qualified mental health professional. D) Signed informed consent. E) No medical history issues

Answer: B, C, D. To be eligible for hormone therapy, the client must have continued and well-documented gender dysphoria, since hormone therapy is not without risks and does not take effect immediately. Gender dysphoria presents as an inner conflict between the person's natal (birth) sex and perceived gender identity. The client must also have been evaluated by a qualified mental health professional, and the client must give informed consent.The client must be over the age of 18. If the client has any medical or mental health diagnoses, they must be well controlled. Just because the client has a positive medical history, does not mean that the client is ineligible for treatment.

The nurse is discussing transvaginal repair for pelvic organ prolapse (POP) using surgical vaginal mesh with a client who plans to have the procedure. What teaching does the nurse include? (Select all that apply). A) Incisional care instructions B) Manufacturer's labeling and information C) Signs and symptoms of infection D) Statements from women who have had successful outcomes E) When to contact the surgeon after the procedure

Answer: B, C, E. The manufacturer's labeling and information is included so that the client has an understanding of the product, its qualifications, and the recommendations from the U.S. Food and Drug Administration (FDA). Although rare, infection is one of the possible complications, so women need to know the signs and symptoms. Clients who have had this procedure need to know when to seek help from their gynecologist/surgeon, so the nurse would include these criteria. Since 2008, client report of complications associated with the use of transvaginal mesh has required the U.S. Food and Drug Administration (2011) to release a classic report and update advising about the safety and effectiveness of the use of this product for POP. Such complications include vaginal mesh erosion, painful sexual intercourse, infection, urinary problems, bleeding, and organ perforation.No surgical incision is involved with the procedure. Statements (testimonials) from other women are not appropriate for client education.

The nurse is facilitating a discussion at an LGBTQ gathering at the local community college. One student asks what kind of genital surgeries are available for someone who wants to transition from female to male (FtM). What options will the nurse tell the group are available for FtM clients? A) Penectomy. B) Vaginectomy. C) Mastectomy. D) Metoidioplasty. E) Scrotoplasty

Answer: B, D, E. The types of genital surgeries the FtM client may undergo include vaginectomy (removal of the vagina), metoidioplasty (creation of a small penis using hormone-enhanced clitoral tissue), and scrotoplasty (creation of a scrotum).Penectomy is surgical removal of the penis, which is not a genital surgery associated with FtM sexual reassignment. A mastectomy is the removal of breast tissue and, although manyclients opt for this surgery, it is not a genital surgery.

The nurse is teaching a client how to perform breast self-examination (BSE). Which of these techniques does the nurse include in the teaching session? (Select all that apply). A) Instruct the client to keep her arm by her side while performing the examination. B) Ensure that the setting in which BSE is demonstrated is private and comfortable. C) Ask the client to remove her shirt. The bra may be left in place. D) Ask the client to demonstrate her own method of BSE. E) Use the fingertips, which are more sensitive than the finger pads, to palpate the breasts.

Answer: B, D. The setting must be private and comfortable to promote an environment conducive to learning and to prevent potential client embarrassment. Before teaching breast palpation, ask the client to demonstrate her own method so that the nurse can assess the client's understanding of BSE.For better visualization, the arm would be placed over the head. The client would undress completely from the waist up. The finger pads, which are more sensitive than the fingertips, are used when palpating the breasts.

A newly graduated RN is orienting to a same-day surgery unit. Which client does the charge nurse assign to the new graduate? A) A 25-year-old with infertility having a laparoscopy under general anesthesia B) A 32-year-old with a breast lump scheduled for a needle biopsy under local anesthesia C) A 40-year-old with possible cervical cancer having a laser excision conization D) A 66-year-old with prostatic enlargement scheduled for a transrectal needle biopsy

Answer: B. A needle biopsy of the breast has the least risk for possible complications and the least complex client teaching of the listed procedures.A laparoscopy under general anesthesia, laser excision conization, and transrectal needle biopsy are all procedures that will require complex client teaching and postoperative monitoring.

A female-to-male client wishes to retain the option of having biological children after transitioning. What available option does the nurse suggest to the client? A) Oocyte freezing can occur after hormone therapy has started and before menstruation ceases. B) Oocyte freezing can occur prior to hormone therapy or gender reassignment surgery. C) Oocyte freezing can occur any time before gender reassignment surgery. D) No options are available to a client with gender dysphoria.

Answer: B. Although expensive, oocyte or embryo freezing would take place prior to the use of any hormone therapy or gender reassignment surgery if the client wishes to preserve reproductive options. Frozen gametes or embryo can be implanted in a surrogate woman to become pregnant and carry to birth.Once hormone therapy or gender reassignment surgery has taken place, the options regarding oocyte freezing are no longer viable. Oocyte freezing is available to clients with gender dysphoria.

A client has gynecologic cancer. Which client statement demonstrates a correct understanding of her treatment options? A) "Chemotherapy will be used to shrink my cancer before I have my operation." B) "External beam radiation therapy (EBRT) may be used after my cancer surgery." C) "Brachytherapy is given on an outclient basis for 4 to 6 weeks before surgery." D) "The purpose of brachytherapy will be to dissolve the cancer."

Answer: B. EBRT may be used to treat any stage of gynecologic cancer in combination with surgery.Chemotherapy is used as palliative treatment for advanced and recurrent disease when it has spread to other parts of the body. External beam treatment, not brachytherapy, is given on an ambulatory care basis after surgery, if needed. The purpose of brachytherapy is to assist in preventing disease recurrence. Brachytherapy involves a radiologist placing an applicator within the woman's uterus through the vagina, and after checking for correct position of the applicator, the radioactive isotope is placed in the applicator and remains for several minutes.

For feminizing surgery, in what position would the client be placed? A) Fowler's. B) Lithotomy. C) Prone. D) Trendelenburg.

Answer: B. For feminizing surgery, the lithotomy position with feet in stirrups is utilized.The other positions do not allow adequate access and visualization of the area for the multiple components of this procedure.

A client has been prescribed goserelin (Zoladex). The nurse is reviewing discharge teaching with the client. Which statement by the client indicates a need for further teaching? A) "I must call 9-1-1 right away if I have chest pain." B) "This medicine goes deep into my muscle." C) "I will be careful when I dispose of my needles." D) "I'll want to keep an eye on my heart rate and follow up with my primary health care provider."

Answer: B. Goserelin (Zoladex) is a gonadotropin-releasing hormone agonist often used to block the effects of testosterone in male-to-female clients. Goserelin is administered subcutaneously.Calling 9-1-1 is appropriate. Tachycardia and dysrhythmias are major side effects of goserelin, and between 1% and 5% of clients sustain a myocardial infarction. Proper administration technique and sharps disposal would be taught before the client is given the first prescription. The nurse would teach the client how to correctly monitor pulse, and the parameters that would be reported to the primary health care provider.

A 32-year-old client has small uterine fibroids and is considering options for treatment. To assist the woman to make a decision about whether to have magnetic resonance-guided focused ultrasound or uterine artery embolization, what will the nurse determine? A) If the woman has had one or more children B) Whether the woman wants to preserve her fertility and desires to have children. C) The age of onset of the woman's first menstrual period D) The woman's risk for uterine cancer

Answer: B. If the woman wants to preserve her fertility, magnetic resonance-guided focused ultrasound is a good option. If the client does not desire pregnancy, then uterine artery embolization may be used.The woman's previous childbearing history, the age of menarche, and the risk for uterine cancer do not influence the decision about which of these two types of procedures are used.

Why is prostate cancer screening often emphasized to the African-American population in the United States? A) Metastasis of prostate cancer is higher. B) Prostate cancer occurs at an earlier age. C) There is no relation to having any first-degree relative with prostate cancer prior to age 65. D) Clinical presentation is different.

Answer: B. In the United States, prostate cancer affects African-American men the most and at an earlier age. African-American men have a slighter higher normal PSA value, but the reason for this difference is not known.There is no difference in prostate cancer metastasis, or clinical presentation of prostate cancer in the African-American population as compared to other populations. African Americans or men who have a first-degree relative with prostate cancer before the age of 65 years have a higher risk for prostate cancer. Men who have multiple first-degree relatives with prostate cancer at an early age need to discuss screening at age 40 years (ACS, 2016).

A client with possible prostate cancer has a transrectal ultrasound and needle biopsy. The next day, which client statement is of greatest concern to the nurse? A) "I am really worried about the test results." B) "I feel like I have a fever and my back aches." C) "I had some bright-red spotting after the procedure." D) "I haven't had a bowel movement since the biopsy."

Answer: B. Low back pain and fever are indications of infection and could indicate a potentially life-threatening complication.Worrying about test results, some bright-red spotting, and having no bowel movement since the biopsy are not abnormal for a client who had a transrectal biopsy one day ago.

A nurse is counseling a client who tests positive for syphilis in an inner-city clinic that offers free sexually transmitted disease testing. What will the nurse tell the client about providing confidential treatment for the client and the client's partners? A) The client would notify sexual partners and encourage them to seek treatment. B) The client would provide accurate information about partners so prophylactic care can be initiated. C) Syphilis is not reportable to the local health agency and follow-up is voluntary. D) The clinic will provide information about this client to other potential partners until treatment is complete.

Answer: B. Prophylactic care for all sexual partners of clients infected with syphilis is necessary to prevent spread and long-term effects of the disease. The client must provide accurate information so that all partners may be notified.Asking the client to suggest to partners that treatment may be necessary doesn't help the clinic get information about partners. Syphilis is reportable. The clinic will keep the client's information confidential.

The nurse is teaching care principles to a client who plans uterus-sparing surgery to remove uterine fibroids. Which client statement indicates that further teaching is needed? A) "I will be able to return to my usual activities in about 2 weeks." B) "It is important to avoid having sexual intercourse for 3 weeks after surgery." C) "Probably I will be able to go home on the day of the surgery." D) "Fewer complications occur with this procedure than with hysterectomies."

Answer: B. The client must avoid having sexual intercourse for at least 6 weeks (not 3 weeks) after the surgery.Most clients can return to their usual activities within 2 weeks of having uterus-sparing surgery for fibroids. Most clients do go home on the day of the surgery. Postoperative pain is less and complications fewer with these procedures than with routine hysterectomies.

A client had a total abdominal hysterectomy 2 days ago and is to be discharged on antibiotics. What does the nurse include in her discharge teaching about antibiotics? A) "After your first day at home, you can stop them if you do not have a fever." B) "It is important to take them as directed until they are all gone." C) "Stop the antibiotic if you feel nauseated because it will lose its effectiveness." D) "You will need to take the drug until your incision heals."

Answer: B. The client must finish her entire course of antibiotics and take them even after she has diminished signs or symptoms. This is a fundamental principle of antibiotic administration.The client would never be instructed to stop a course of antibiotics. In cases in which clients are unable to take an antibiotic (due to nausea or another problem), an alternative antibiotic will be prescribed. The client's incision would be healed by the time that the antibiotic course is completed.

The nurse in the clinic is administering benzathine penicillin G (Bicillin C-R) intramuscularly to a client with primary syphilis. The client has never been treated with this particular form of the drug. What precaution does the nurse implement? A) Applies an ice pack to the injection site to minimize trauma to the client's skin B) Keeps the client in the clinic for 30 minutes to monitor for a possible allergic reaction C) Makes certain the client has a skin test for penicillin allergy prior to the injection D) Tells the client to refrain from all sexual activities for a minimum of 72 hours

Answer: B. The clinic nurse needs to keep all clients on-site for at least 30 minutes after they have received this antibiotic so that manifestations of an allergic reaction can be detected and treated.The application of ice to the injection site is not recommended because it would actually slow the absorption of the drug into the muscular tissues. The nurse must be sure that the client who has never had any form of penicillin has a skin test before receiving the injection. The nurse would recommend sexual abstinence until the treatment of both the client and his or her partner(s) is completed.

A client for whom a mastectomy is planned tells the surgical nurse, "I wish there was someone who went through this that I could speak to about how she managed it all." Which community resource will the nurse recommend? A) National Breast Cancer Coalition B) Reach to Recovery C) Susan G. Komen for the Cure D) Young Survival Coalition

Answer: B. The nurse recommends the Reach to Recovery program to the pre-mastectomy client. The American Cancer Society's program Reach to Recovery provides volunteers who visit clients in the hospital or at home. They bring personal messages of hope; informational materials on breast cancer recovery; and a soft, temporary breast form.The National Breast Cancer Coalition is an organization dedicated to ending breast cancer through action and advocacy. Susan G. Komen for the Cure is an organization that supports breast cancer research. The Young Survival Coalition is an organization dedicated to educating the medical, research, breast cancer, and legislative communities about breast cancer, as well as serving as a point of contact for young women living with breast cancer. None of these other community resources provide volunteers to visit the home.

A client who has been diagnosed with breast cancer tells the nurse she wishes to use only natural and complementary interventions. Which of these will the nurse explain to the client? A) If chemotherapy has been recommended, complementary therapies are contraindicated B) This type of therapy would not replace standard treatment C) There are many natural hormonal therapy replacements that can be used D) Complementary therapies can only be used after surgery

Answer: B. The nurse would explain that complementary and integrative therapies cannot replace standard treatment for breast cancer. Complementary and integrative health options include prayer, herbal therapy, cancer diets, guided imagery, acupuncture, and others. No proven benefit has been found with using these interventions alone as a cure for breast cancer.Complementary therapies are not contraindicated when chemotherapy is recommended, but it is important to ensure that the client's choices can be safely integrated with conventional treatment for breast cancer. Not all hormonal therapies can be used. Encourage clients who are interested in trying these therapies to check with their primary health care provider before using them. Teach the client that all ingested complementary agents potentially risk interaction with conventional drugs. Also, cost may be a factor in decision-making since not all insurances provide coverage for complementary and integrative therapies. If approved by the primary health care provider, complementary and integrative therapies can be used before surgery as well as after surgery.

A clinic nurse is providing expedited partner treatment (EPT) to a client who is newly diagnosed with chlamydia. What does the nurse do first? A) Confirms the diagnosis of the partner through appropriate testing B) Gives the drug and directions to the client for the partner C) Makes an appointment at the clinic for the partner D) Provides condoms for the client and partner to use

Answer: B. The nurse would first give the drug or a prescription with specific instructions for administration to the client for the partner, without direct evaluation by a primary health care provider. EPT is successful in reducing chlamydia infection rates. Although there has been some discussion about the legality of this practice, it is supported by the Centers for Disease Control and Prevention.It is not required for the nurse to confirm the diagnosis of the partner or to make an appointment for the partner. Providing condoms is not a component of EPT.

A transwoman presents to her primary health care provider's office reporting difficulty sleeping, anxiety, and hypervigilance. She states "I just can't stop thinking about what they did to me last New Year's Eve at work. They slashed my tires. They took my purse. I see it over and over." What is the nurse's best action? A) Ask if the client has been abusing drugs or alcohol. B) Consult with the primary health care provider for referral to a counselor. C) Document the statements and proceed with the remainder of the assessment. D) Look for signs of self-harm.

Answer: B. The nurse's best action is to consult with the primary health care provider for referral to a counselor. Although accurate documentation is important with regard to the client's statements, this client's symptoms indicate she may be experiencing posttraumatic stress disorder.Male-to-female individuals are more than two times more likely to experience physical violence and discrimination than non-transwomen. Asking the client about drug and alcohol use is an important part of the assessment, but it is only one component. Also, the use of the word "abusing" implies it is the client who is in the wrong. Just documenting and continuing the assignment is not sufficient. Assessing for signs of self-harm is also important and part of the physical assessment, but again, it is only one small component.

A client with prostate cancer asks why he must have surgery instead of radiation, even if his cancer is the least-invasive type. What is the nurse's best response? A) "It is because your cancer growth is large." B) "Surgery is the most common intervention for a cure." C) "Surgery slows the spread of cancer." D) "The surgery is to promote urination."

Answer: B. The nurse's best response is that surgery is the most common intervention for a cure, because some localized prostate cancers are resistant to radiation.The size of the tumor is not likely to be the reason for the client to have surgery. A bilateral orchiectomy (removal of both testicles) is palliative surgery that slows the spread of cancer by removing the main source of testosterone. A transurethral resection of the prostate (TURP) is done to promote urination for clients with advanced disease. It is not used as a curative treatment.

A male-to-female client and her partner come to the preoperative appointment. "My partner has some questions. He wants to know about my new vagina. What will it be made of?" What is the nurse's best response? A) "The neovagina is made from silicone." B) "It is made with inverted penile tissue." C) "It will be made from parts of the scrotum." D) "The surgeon uses skin grafts to create the neovagina."

Answer: B. The nurse's best response is that the neovagina (new vagina) is created during a vaginoplasty from inverted penile tissue or a colon graft.Silicone is used during breast augmentation surgery. Tissue taken from the scrotum and skin grafts may be utilized to create a clitoris or labia.

A client presents to the clinic to discuss options for treatment for gender dysphoria. He states "I'm confused and I need to talk to somebody, but I don't know what to do and who to talk to. I don't want my parents to know. Can you help me?" What is the nurse's priority response? A) "I'll tell the doctor you want to talk to him." B) "What you say here will be confidential." C) "Let's see if we can get a therapist to see you." D) "It depends on what kind of insurance you have."

Answer: B. The nurse's priority response is to tell the client that this discussion is confidential. It is most important to reassure the client that, as long as there is no evidence of abuse or concern of immediate self-harm, discussions with all health care providers are confidential. Collaborating with other members of the health care team, including the physician and therapists, is important to provide comprehensive care, but the client must first be reassured of confidentiality; otherwise the client may not be communicative.Collaborating with other members of the health care team, including the physician and therapists, is important to provide comprehensive care, but the client must first know that this topic will be confidential. Insurance coverage is not the top priority in client care.

The nurse is completing preoperative teaching for a client who is having a phalloplasty. Which statement by the client indicates further education is necessary? A) "Fat may be needed to make my penis bigger." B) "I will get my implant during the last surgery." C) "They will take skin flaps to make my penis." D) "This is going to take several surgeries to complete."

Answer: B. The penile implant will not be placed until several months after all of the surgeries have been completed and the incisions are healed.Skin flaps may be taken from the back, radial forearm, or anterior lateral thigh to create the penis, and fat grafts may be needed to increase its circumference. Phalloplasties are one of the most difficult types of genital surgeries and are usually done in stages.

A client is beginning transdermal estrogen (Climara) therapy. Which statement by the client indicates the need for additional health teaching by the nurse? A) "I will monitor my blood pressure while I am taking this medicine." B) "I will need to change out the patch once a month." C) "My blood work will be checked regularly." D) "This medicine will increase my risk of blood clots."

Answer: B. The transdermal estrogen patch (Climara) is typically dosed as two 0.1-mg patches that are changed twice weekly. If the patch were only changed once a month, the dose would likely be insufficient.Estrogen therapy may cause hypertension, so blood pressure would be regularly monitored. Glucose and lipids are also monitored because estrogens may elevate triglyceride levels. The risk of venous thromboembolism is markedly increased with the use of estrogens.

What is the correct way to refer to a client who self-identifies as the opposite gender? A) A transgender. B) Transgender. C) Transgendered. D) Transvestite.

Answer: B. Transgender describes clients who self-identify as the opposite gender or whose gender does not match the one with which they were born (or natal sex).It is never correct to refer to someone as "a transgender," or to say someone is "transgendered." Transvestite may have negative connotations in some cultures. The best way to determine how someone wishes to be addressed is always to ask the client.

A client is being discharged after a total abdominal hysterectomy (TAH). What principle guides the nurse in planning discharge care and teaching? A) Clients in their childbearing years generally adapt better. B) No special home equipment will be necessary for the client. C) Psychological reactions would be evident by discharge. D) The client can resume normal activities upon discharge.

Answer: B. Usually, no special home equipment is needed for the client who has undergone a TAH.Generally, clients adjust better to the surgery if they have completed their childbearing years. Psychological reactions can occur months to years after surgery, particularly if sexual functioning and libido are diminished. The client who has undergone a TAH would be taught about the expected physical changes, including any activity restrictions. A 2-6 week convalescent period is usually required.

For the 12 months prior to surgery, what is one of the requirements for a client requesting a vaginoplasty or a phalloplasty? A) Therapy sessions with a licensed psychotherapist as needed B) Continuously living in the role of the desired gender identity C) Hormone therapy D) Monthly vocal coaching

Answer: B. Vaginoplasty (creation of a vagina) and phalloplasty (creation of a penis) are very extensive surgeries that alter the external appearance of a client. Continuously living in the gender role that is congruent with the client's gender identity for 12 months is required prior to surgery.It is highly recommended that the client be engaged in regular visits with a mental health professional. There is no requirement for hormone therapy. Vocal therapy may be of benefit to male-to-female clients, but this is not a requirement for surgery.

Which action can the same-day surgery charge nurse delegate to an experienced unlicensed assistive personnel (UAP) who is helping with the care of a client who is having a breast biopsy? A) Assess anxiety level about the surgery. B) Monitor vital signs after surgery. C) Obtain data about breast cancer risk factors. D) Teach about postoperative routine care.

Answer: B. Vital sign monitoring is included in UAP education and usually is part of the job description for UAPs working in a hospital setting.Nursing assessment, obtaining data, and client teaching are outside the scope of practice for UAPs and would be done by licensed nursing staff.

The nurse will conduct prostate screening and provide education about prostate cancer with which male client? A) A young adult with a history of urinary tract infections B) A client who has sustained an injury to the external genitalia C) An adult male who is older than 50 years D) A sexually active client

Answer: C. A man who is 50 years or older is at highest risk for prostate cancer. The risk increases for men who have a first-degree relative (brother, father) with the disease.A history of urinary tract infections, injury to the external genitalia, and sexual activity are not risk factors for prostate cancer.

Which assessment finding causes the nurse to suspect that a client may have testicular cancer? A) Hematuria. B) Penile discharge. C) Painless testicular lump. D) Sudden increase in libido.

Answer: C. A painless lump or swelling in the testicles is the most common assessment finding of testicular cancer.Hematuria is not a symptom of testicular cancer but could be indicative of other conditions such as bladder cancer. Penile discharge is not a symptom of testicular cancer but could be indicative of another condition. A sudden increase in libido is not a symptom of testicular cancer.

A client with pelvic pain is admitted to the same-day surgery unit for a laparoscopic procedure. Which nursing action does the RN delegate to unlicensed assistive personnel (UAP)? A) Educating the client about analgesic use for referred pain B) Inserting a retention catheter using sterile technique C) Taking the client's admission blood pressure and heart rate D) Teaching the client about postoperative activity restrictions

Answer: C. Although most of the admission assessment and history will be completed by the RN, the admission vital signs can be delegated to a UAP.Client education and teaching is a higher-level skill and must be done by the RN. Catheter insertion is also a higher-level skill and would be done by the RN, unless the UAP has had specialized training to perform this skill safely.

What task does the RN delegate to unlicensed assistive personnel (UAP) working on the medical-surgical unit? A) Inserting a catheter in a client who has a history of uterine prolapse B) Giving report to a receiving nurse about a client who is being transferred C) Assisting with a sitz bath for a client with ulcerative vulvitis D) Providing discharge teaching for a client who is scheduled for brachytherapy

Answer: C. Assisting with a client's sitz bath is within the UAP scope of practice and can safely be delegated.Some specially trained UAPs do catheterize clients, but a client with a uterine prolapse poses additional problems and needs to be managed by a licensed nurse. Giving report to a receiving nurse about a client who is being transferred is an interaction that would be "nurse-to-nurse." Providing discharge teaching for a client who will be having brachytherapy (intracavitary radiation) is a complex nursing action and would be done by an RN.

The nurse is teaching a group of young women about the risks for developing cervical cancer. What cancer risk is included in the content of the nurse's presentation? A) Eating a diet that is high in fat content B) Having more than six pregnancies C) Initiating sexual intercourse at a very early age D) Using a diaphragm with spermicidal jelly as a contraceptive device

Answer: C. Having intercourse at a very early age and/or multiple sex partners places a woman at high risk for the development of cervical cancer.Eating a diet that is high in fat content, the number of pregnancies, and using a diaphragm have not been identified as increasing the risk for cervical cancer.

The human papilloma virus (HPV) test may be collected at the same time as the Papanicolaou (Pap) test for screening. Which finding indicates the highest risk for development of cervical cancer? A) Normal Pap results and no HPV infection B) Abnormal Pap results and no HPV infection C) Abnormal Pap results and positive HPV test D) Normal Pap results and positive HPV results

Answer: C. If not treated, women with an abnormal Pap result and a positive HPV test have the highest risk for developing cervical cancer.Women who have normal Pap test results and no HPV infection are at the lowest risk for developing cervical cancer.

A premenopausal client diagnosed with breast cancer will be receiving therapy with leuprolide. The nurse teaches the client to expect which of the following side effects? A) Heavy menses. B) Nausea and vomiting. C) Hot flashes. D) Weight loss.

Answer: C. Leuprolide, used in premenopausal women, is a luteinizing hormone agonist that inhibits estrogen synthesis. As a result, women experience symptoms of menopause including hot flashes and mood changes.As symptoms of menopause develop, the client will not have heavy or regular menses. Nausea and vomiting is not a side effect of Leuprolide. Edema, rather than weight loss, may occur during treatment with leuprolide.

Which client history places a woman at highest risk for developing endometrial (uterine) cancer? A) Multiparity, human papilloma virus (HPV), smoking, and African-American ethnicity B) Nulliparity, endometriosis, diabetes mellitus, first pregnancy at older than 20 years C) Nulliparity, smoking, uterine polyps, hypertension D) Oral contraceptive use, smoking, localized pain in the thigh

Answer: C. Nulliparity, smoking, uterine polyps, and hypertension are all risk factors for endometrial cancer.Multiparity, HPV, smoking, and African-American ethnicity are all risk factors for cervical cancer. Nulliparity, endometriosis, diabetes mellitus, and first pregnancy at older than 30 years are all risk factors for ovarian cancer. Oral contraceptive use, smoking, and localized pain in the thigh are all risk factors for a thrombus.

A client has returned to the floor after a vaginoplasty. Which assessment finding would concern the nurse? A) Edema of the perineum B) Drainage in the Jackson-Pratt drain C) Numbness in the right leg D) Request for pain medication every 4 hours

Answer: C. Numbness in the right leg is an assessment finding that would be of concern to the nurse. Clients placed in the lithotomy position for prolonged periods of time are at risk for either compartment syndrome or nerve injury due to pressure on the femoral or peroneal nerve.Edema and pain of the perineum are to be expected, as is drainage in the Jackson-Pratt drain. Drainage would be monitored carefully for amount and color. It is not unexpected for the client to request pain medication every four hours.

What will the nurse include in teaching to assist the client who has undergone open radical prostatectomy surgery to overcome urinary incontinence? A) Ask the primary health care provider about prescribing sildenafil (Viagra). B) Use analgesic medications as needed to maintain comfort C) Practice Kegel exercises to improve muscle control D) Practice routine self-catheterization and bladder retraining

Answer: C. Practicing Kegel exercises may help to regain urinary continence. A radical prostatectomy may lead to urinary incontinence because the internal and external sphincters may be damaged during surgery.Sildenafil is useful after radical prostatectomy to help restore erectile function, not for incontinence. Analgesic medications do not help with urinary continence. Self-catheterization and bladder retraining are used for neurogenic bladder.

What does the nurse tell a client prior to a mammogram to help prepare for the test? A) "A mammogram will x-ray the hard tissue of your breasts." B) "Do not eat anything for 12 hours before having your mammogram." C) "You must not wear deodorant the day of your mammogram." D) "You will not feel any discomfort during the mammography procedure."

Answer: C. Remind the client not to use creams, lotions, powders, or deodorant on the breasts or underarms before the study because these products may be visible on the mammogram and lead to misdiagnosis.Mammography is an x-ray of the soft tissue of the breast. Dietary restrictions are not necessary before a mammogram. The client may experience some temporary discomfort when the breast is compressed during positioning and the test itself. If there is any possibility that the client is pregnant, the test would be rescheduled. Explain the purpose of the procedure and its anticipated discomforts. The technician or assistant provides a gown and privacy for the woman to undress above the waist. Allow the client to express concerns about the mammogram and the presence of any lumps.

The nurse is obtaining a personal health history on a 24-year-old male whose male partner is present. How does the nurse approach questions about his sexual practices? A) Defer questions about his sexual practices to the primary health care provider. B) Direct questions about sexual practices to both the client and his partner. C) Respect the client's choice to answer or refuse to answer questions about sexual practices. D) Skip the assessment questions about his sexual practices.

Answer: C. Respecting the client's choice to answer or not answer questions about sexual practices is an important part of the process of taking the sexual history.The nurse must be sensitive about knowing when to ask and when to permit the client his or her privacy. Deferring questions about sexual practices to the primary health care provider or skipping questions is missing potentially important data. The nurse must establish trust with the client and then proceed with data collection. The nurse is collecting data on the client only and not on the partner or the client's relationship with his partner. Directing questions to both of them could be very uncomfortable for the client.

The nurse is discussing the prevention of sexually transmitted diseases (STDs) with a group of young adults. What information does the nurse include? A) Female condoms are not effective in preventing the transmission of STDs. B) Spermicidal agents, when used with condoms, will prevent the transmission of STDs. C) The risk of STDs increases with the number of sexual partners. D) Using latex condoms always keeps STDs from spreading and infecting others.

Answer: C. STD risk factors for sexually active people include multiple sexual partners.Female condoms (polyurethane sheaths in the vagina) are effective for preventing transmission of STDs, including human immunodeficiency virus. The use of spermicide with condoms, either lubricated condoms or vaginal application, has not been proved to be more or less effective against STDs than use without spermicide. Latex condoms do not completely prevent the spread of STDs, but they do substantially decrease the risk.

A client is scheduled for a total hysterectomy with a laparoscopic vaginal approach after a diagnosis of microinvasive cervical cancer. What psychological and/or social changes does the nurse expect this client to experience? A) Because the surgery does not affect a visible site, altered body image issues are not as common. B) The client will be actively involved in her own care in the immediate postoperative period. C) Sexual counseling may be needed, especially if the client has doubts about her ability to feel like a woman and engage in sexual activities. D) The client would demonstrate reality testing and would experience a grief reaction immediately after her surgery.

Answer: C. Sexual function may be (or feel) different after a hysterectomy. Couples may need counseling about intercourse or alternative sexual activities. The nurse assesses the need for sexual counseling by listening for cues about altered perceptions of body image and anxiety in either of the sexual partners' responses.For many women, hysterectomy can mean the loss of their femininity, so altered body image issues must be expected with the client. Hysterectomy is major surgery, so the client will be recovering for days to a week or longer. Active involvement in her self-care will be delayed until she has moved past the initial surgical procedure recovery period. Reality testing is a later step in the grief and acceptance processes experienced by women who have had hysterectomies.

The nurse is teaching a female client about managing her sexually transmitted disease with antibiotics. Which client statement indicates that teaching has been effective? A) "I can resume having sex 24 hours after my first dose of antibiotics." B) "I will need to take an antacid 2 hours after taking the antibiotic." C) "I need to take these antibiotics for 14 days." D) "This antibiotic must be taken with food."

Answer: C. The CDC recommends oral and/or parenteral antibiotics for PID for 14 days (CDC, 2015b). The nurse needs to stress that antibiotics must be taken as prescribed and continued until all have been taken.Sexual intercourse must be avoided until after antibiotic therapy is completed. If the client's partner is being treated, sex can resume after the partner also completes his or her treatment. Antacids containing calcium, magnesium, or aluminum (Tums, Maalox, or Mylanta) would not be taken with antibiotics because they may decrease their effectiveness. Antibiotics need to be taken on an empty stomach unless the provider instructs the client to take them with food.

A client who has undergone breast surgery is struggling with issues concerning her sexuality. What is the best way for the nurse to address the client's concerns? A) Ask the client if she is using her surgery as an excuse not to have any more sexual relations with her partner. B) Remind the client to avoid sexual intercourse for 2 months after the surgery. C) Ask the client about satisfaction with sexual relations with her partner. D) Teach the client that birth control is a priority.

Answer: C. The best way for the nurse to address the client's concerns about sexuality after undergoing breast surgery is to ask the client about her satisfaction with sexual relations with her partner. Refer the client and her partner to counseling if appropriate.Asking the client if she is using her surgery as an excuse not to have sexual relations is accusatory and not therapeutic. Sexual intercourse can generally be resumed 4-6 weeks after surgery whenever the client is comfortable. Sexually active clients receiving chemotherapy or radiotherapy must use birth control because of the therapy's teratogenic effects, but this is not necessary for clients who only have had surgery.

The nurse is performing discharge teaching for a client who is recovering from a total abdominal hysterectomy (TAH). Which client statement indicates a need for clarification? A) "I cannot jog for 2 to 6 weeks." B) "I must take my temperature twice a day for the first few days after surgery." C) "I will need to find a new form of birth control." D) "I will no longer have menstrual periods."

Answer: C. The client who has had a TAH can no longer become pregnant. Therefore, birth control is no longer necessary.The client must avoid jogging, aerobic exercise, participating in sports, or any strenuous activity for 2 to 6 weeks. The client must take her temperature twice a day for the first few days after surgery as a precaution to monitor for infection. The client will no longer have a period, although she may have some vaginal discharge for a few days after going home.

A client with newly diagnosed pelvic inflammatory disease (PID) is being started on antibiotics as an outpatient. What does the nurse tell the client about her home treatment regimen? A) "A rise in your temperature is expected for the first several days." B) "If you engage in sexual activity, be certain to use a latex condom." C) "Rest in a semi-Fowler's position to help with the infection and pain." D) "Return to the clinic in 7 to 10 days for a checkup."

Answer: C. The client would be instructed to maintain rest in a semi-Fowler's position to promote gravity drainage of the infection. This may also help relieve the pain involved with PID.A rise in temperature is not expected and clients must be taught to report any increase in temperature to their primary health care provider. Nurses need to instruct women to cease sexual activity until the antibiotic therapy is completed and they no longer have symptoms. The client must be seen by the primary health care provider within 72 hours of starting the antibiotics and then 1 and 2 weeks from the time of the initial diagnosis.

A 42-year-old woman with an intramural leiomyoma (myomas or fibroids) has been taking estrogen replacement therapy for menopausal symptoms. What does the nurse tell her about estrogen replacement therapy and how it relates to her fibroids? A) "Estrogen will help shrink your fibroids." B) "Increasing the amount of estrogen you are taking will be necessary." C) "The fibroids may continue to grow." D) "Your estrogen dosage will not change."

Answer: C. The fibroids may continue to grow because of the estrogen stimulation.The client would be instructed to see her primary health care provider to monitor their growth. The client's estrogen dose will most likely need to be decreased at some point or eliminated.

The nurse is instructing a client with breast cancer who will be undergoing chemotherapy about the side effects of doxorubicin (Adriamycin). Which side effect does the nurse instruct the client to report to the primary health care provider? A) Diaphoresis. B) Dysphagia. C) Edema. D) Hearing loss.

Answer: C. The side effect of doxorubicin that the nurse tells the client to report to the primary health café provider is edema. Doxorubicin, anthracycline, which has cardiotoxic effects; clients must be instructed to be aware of and to report edema, shortness of breath, chronic cough, and excessive fatigue.Diaphoresis (profuse sweating), dysphagia (difficulty swallowing), and hearing loss are not associated side effects of doxorubicin.

The nurse is teaching a client how to prevent vaginal inflammation and itching. What information does the nurse include? A) Wear snug-fitting latex undergarments to prevent chafing. B) Cleanse the inner labia daily with soap and water. C) Do not have unprotected sex with multiple partners. D) Monthly douching may help reduce symptoms.

Answer: C. Unprotected sex with multiple partners can lead to vaginal infection.Tight clothing, such as pantyhose or tight jeans, would be avoided because it can cause chafing. Tight clothing can also cause the vaginal area to get hot and sweaty, which can lead to infection. Encourage the client to wear breathable fabrics such as cotton and to avoid irritants or allergens in products such as laundry detergents or bath products. During a bath or shower, the inner labial mucosa would be cleansed with only water, not soap. Soap is an irritant to the sensitive skin in those areas. The use of douches or feminine hygiene sprays is not recommended because they disturb the balance of both pH and bacteria and can aggravate irritation.

The nurse is teaching a group of young women about screening for chlamydia. Which client statement shows a correct understanding of these practices? A) "As a sexually active 19-year-old, I need to be screened every 2 years." B) "At age 30, I still need yearly testing, even if I am monogamous." C) "If I am a 40-year-old woman with a 'new' partner, I need to be screened again." D) "Self-collected urine specimen testing is not a reliable method for screening."

Answer: C. Women older than 25 years with new or multiple partners need to be screened annually for chlamydia.All sexually active women 25 years old or younger need to be screened annually for chlamydia. The 30-year-old woman who is monogamous does not need to be screened. Only women older than 25 years with new or multiple partners would be screened annually. The urine self-collection method has been found to be more acceptable and highly sensitive and specific and has resulted in increased identification of asymptomatic clients with chlamydia.

A 48-year-old woman who has been diagnosed with breast cancer with the BRCA genetic mutation requests information about early detection for her daughter because of her daughter's genetic risk. Which information will the nurse convey? A) Breast self-examination (BSE) beginning at 20 years of age is the best way to detect breast cancer B) Hormone replacement therapy (HRT) combining estrogen and progesterone may be recommended by your daughter's primary health care provider C) MRI for screening typically begins at an age that is 10 years younger than your present age. D) The primary health care provider will most likely discuss prophylactic mastectomy with daughters of women with genetic mutations

Answer: C. Women with a high family history risk for developing breast cancer need to have a MRI for screening at an age that is 10 years younger than the age at which the affected cancer client was initially diagnosed.BSE is an option for everyone, not just those at high genetic risk for breast cancer. Use of HRT containing both estrogen and progestin increases risk, but risk diminishes after 5 years of discontinuation. With a prophylactic mastectomy, there is a small risk that breast cancer will develop in residual breast glandular tissue because no mastectomy reliably removes all mammary tissue.

The nurse is teaching a local young women's group about health promotion and maintenance measures for prevention of gynecologic cancers. Which preventive factors does the nurse stress? (Select all that apply). A) Annual endometrial biopsy B) Annual human papilloma virus (HPV) vaccination C) Annual Papanicolaou (Pap) test D) Safe sex practices E) Well-balanced diet

Answer: D, E. Using barrier protection, especially if a woman has multiple sexual partners, is recommended. Knowing the history of partners is also a factor in having safe sex. Eating a diet that includes a variety of healthy food choices (fruits, vegetables, low-fat protein, and healthy dairy products) is known to help a woman have a healthy reproductive system.Endometrial biopsies are not routinely performed annually except when risk for the development of the disease is increased. HPV vaccination is given to young girls (and boys) in a series of three injections over a 6-month time frame. It is best administered before they become sexually active. Women need to begin screening precautions at the age of 21 years. Between ages 21 and 29 years, women need to have a Pap test every 3 years. Women between ages 30 and 65 years need to have a Pap test plus a human papilloma virus (HPV) test (co-testing) every 5 years. In the absence of co-testing, this population needs to still have a Pap test every 3 years.

The potential problem of grief is most relevant to a client after which procedure? A) Cystocopy. B) Transurethral microwave therapy. C) Radical prostatectomy. D) Sperm banking.

Answer: D. A radical prostatectomy may lead to erectile dysfunction, which could present a potential problem of grief at loss of function.Cystoscopy, a test to view the interior of the bladder, the bladder neck, and the urethra, does not affect sexuality. Transurethral microwave therapy is a minimally invasive procedure involving high temperatures that heat and destroy excess prostate tissue and does not affect sexuality. The process of sperm banking would not result in a diagnosis of altered self-image. However, the diagnosis leading to the necessity of sperm banking might cause grief.

A client has recently had a vaginoplasty and has noticed stool coming from her vagina. She calls her primary health care provider's office and the nurse advises her to immediately go the emergency department. What is the nurse's concern? A) Dislodgement of the urinary catheter B) Labial hematoma C) Prolapse D) Vaginal-rectal fistula

Answer: D. A vaginal-rectal fistula is caused by a rectal perforation during the surgery to create the vagina and may lead to passage of stool into the vagina. This is a major complication that will necessitate a temporary colostomy as well as extensive wound care.A urinary catheter is usually left in during the immediate postoperative period. A labial hematoma would be apparent on the labia but would not cause passage of stool. A prolapse or herniation of contents would not create passage of stool.

A new transgender client is admitted to the unit for treatment. The nurse uses the wrong pronoun when addressing the client and taking the admitting history. What would the nurse do? A) Apologize repeatedly throughout the shift. B) Ask for reassignment to another client. C) Report the error to the charge nurse and write a variance. D) Self-correct and continue with the admitting history.

Answer: D. Errors may occur and transgender clients may very well have encountered them before. If there was no intent on the part of the nurse to be disrespectful, the best action is to self-correct and continue with the admitting history.Apologizing repeatedly focuses on the error and emphasizes it. The nurse would not ask for reassignment or fill out a variance as this would imply that something was wrong with the client's care, the nurse, or the client.

A client who had an anterior colporrhaphy is being discharged. What does the nurse tell the client before her discharge? A) "Avoid lifting more than 25 pounds (11.3 kg)." B) "Do not have sexual intercourse for at least 2 weeks." C) "Return to the clinic in 6 weeks for suture removal." D) "Take a hot bath or use a moist heating pad for discomfort."

Answer: D. For discomfort, the client would be instructed to use heat—either a moist heating pad or warm compresses applied to the abdomen. A hot bath may also be helpful.The client must avoid lifting anything heavier than 5 pounds (2.3 kg) and avoid sexual intercourse for 6 weeks. Sutures do not need to be removed because they may be absorbable or they may fall out (slough off) as healing occurs.

The nurse is discussing treatment options with a client newly diagnosed with breast cancer. Which statement by the client indicates a need for further teaching? A) "Hormonal therapy is only used to prevent the growth of cancer. It won't get rid of it." B) "I might have chemotherapy before surgery." C) "If I get radiation, I am not radioactive to others." D) "Radiation will remove the cancer, so I might not need surgery."

Answer: D. Further teaching is needed when the client says that, "Radiation will remove the cancer, so I might not need surgery." The purpose of radiation therapy is to reduce the risk for local recurrence of breast cancer. Typically, radiation therapy follows surgery to kill residual tumor cells. Radiation therapy plays a critical role in the therapeutic regimen and is an effective treatment for almost all sites where breast cancer can metastasize.The purpose of hormonal therapy is to reduce the estrogen available to breast tumors to stop or prevent their growth. Chemotherapy drugs destroy breast cancer cells that may be present anywhere in the body. They are typically administered after surgery for breast cancer, although neoadjuvant chemotherapy may be given to reduce the size of a tumor before surgery. The client receiving radiation therapy is radioactive only if the radiation source is dwelling inside the breast tissue.

Hormone treatment for prostate cancer works by which action? A) Decreased blood flow to the tumor B) Destruction of the tumor C) Shrinkage of the tumor D) Suppression of the growth of the tumor

Answer: D. Hormone therapy, particularly antiandrogen drugs, inhibits tumor progression by blocking the uptake of testicular and adrenal androgens at the prostate tumor site. Because most prostate tumors are hormone dependent, clients with extensive tumors or those with metastatic disease may be managed by androgen deprivation. Luteinizing hormone-releasing hormone (LH-RH) agonists or antiandrogens can be used. Antiandrogens may be used alone or in combination with luteinizing hormone-releasing hormone agonists for a total androgen blockade (hormone ablation).Hormone treatment for prostate cancer does not decrease blood flow to the tumor, destroy the tumor, or shrink the tumor.

A client with testicular cancer is worried about sterility and the ability to conceive children later. Which resource does the nurse refer the client to before surgery takes place? A) American Cancer Society B) American Fertility Society C) RESOLVE: The National Infertility Association D) A sperm bank facility

Answer: D. If the client is interested in having children, he would be encouraged to arrange for semen storage as soon as possible after diagnosis. A sperm bank facility provides comprehensive information on semen collection, storage of semen, the storage contract, costs, and the insemination process. Sperm collection needs to be completed before radiation therapy or chemotherapy is started. After radiation therapy or chemotherapy has been started, the client is at increased risk for producing mutagenic sperm, which may not be viable or may result in fetal abnormalities.The client is referred to the American Cancer Society for more generalized information on testicular cancer. The American Fertility Society and RESOLVE: The National Infertility Association are appropriate referrals if permanent sterility occurs and sperm storage has not been feasible.

The nurse is educating a group of young men about testicular self-examination (TSE). Which statement by a member of the group indicates teaching has been effective? A) "I will examine my testicles right before taking a shower." B) "I will squeeze each testicle in my hand to feel any lumps." C) "I will only report any large lumps to my primary health care provider." D) "I will look and feel for any lumps or changes to my testes."

Answer: D. In TSE, the client would look and feel for any lumps or changes to the testes. Any lumps that are detected would be immediately reported. With early detection by monthly TSE and treatment, testicular cancer can be successfully cured.A TSE would be performed immediately following a shower. The client would gently roll each testicle between the thumb and forefinger. All lumps must be reported to the primary health care provider, no matter the size.

An older client reports uncomfortable sexual intercourse associated with vaginal dryness. What does the nurse suggest? A) "Be sure to tell your primary health care provider about this." B) "I think that you need to have additional pelvic examinations." C) "Let me teach you how to do Kegel exercises." D) "Products such as water-soluble lubricants may be helpful."

Answer: D. Information about vaginal estrogen therapy and water-soluble lubricants need to be provided to the older woman with vaginal dryness.There is no need to inform the primary health care provider because vaginal dryness is normal with aging. Additional pelvic examinations are not indicated for this client. Kegel exercises are used for clients with incontinence.

The nurse at the gynecology clinic is examining a woman's breasts. Which assessment finding will the nurse report immediately to the primary health care provider? A) A 1-cm freely mobile rubbery mass discovered by the client B) Ill-defined painful rubbery lump in the outer breast quadrant C) Backache and breast fungal infection D) Nipple discharge and dimpling

Answer: D. Nipple discharge and dimpling are high-risk assessment findings for a malignant breast lesion and requires immediate follow up.On clinical examination, fibroadenomas as benign lesions are oval, freely mobile, rubbery masses usually discovered by the woman herself. Their size varies from smaller than 1 cm in diameter to as large as 15 cm in diameter. Although the immediate fear is breast cancer, the risk of its occurring within a fibroadenoma is very small. Breast pain and tender lumps or areas of thickening in the breasts are typical symptoms of a fibrocystic breast condition. The lumps are rubbery, ill-defined, and commonly found in the upper outer quadrant of the breast. Many large-breasted women develop fungal infection under the breasts, especially in hot weather, because it is difficult to keep this area dry and exposed to air. Backaches from the added weight are also common.

The nurse has completed discussing the process of gender reassignment surgery with a male-to-female client during her first visit to the office. Which statement by the client indicates a need for more education? A) "I will need to take hormones for 12 months before my surgery." B) "My insurance probably won't cover this." C) "I will need to have at least one referral from a therapist before I have surgery." D) "The surgeon who performed my appendectomy can do my surgery."

Answer: D. Not all surgeons are comfortable performing gender reassignment surgeries. Frequently these surgeries are performed by urologists and plastic surgeons. Careful evaluation of the surgeon's expertise would be undertaken.Hormone therapy is required for 12 months prior to surgery, and at least one referral from a qualified psychotherapist is required prior to an orchiectomy. In many cases, insurance will not cover costs associated with gender reassignment surgery.

A 68-year-old client has recently undergone a prostate biopsy. Which symptom would indicate immediate referral to the primary health care provider? A) Slight rectal bleeding B) Rust-colored semen C) Mild pain and soreness at the site D) Temperature of 101.6°F (38.7°C)

Answer: D. Rarely, sepsis can develop after a prostate biopsy. However, clients need to be told to contact their primary health care provider immediately if they experience fever, pain when urinating, or penile discharge.Expected findings after a prostate biopsy may include slight soreness, light rectal bleeding, and blood in the urine or stools for a few days. Semen may be red or rust-colored for several weeks.

A 52-year-old client has been diagnosed with endometrial (uterine) cancer. She says to the nurse, "I was told that my cancer is stage II. What does that mean?" How does the nurse respond? A) "It means that your cancer remains confined to your uterus." B) "The spread of your cancer is beyond your pelvic area." C) "The cancer is in your vagina or lymph node areas." D) "Your cancer has spread from your uterus to your cervix."

Answer: D. Stage II means that the cancer now also involves the client's cervix.Cancer that remains confined to the endometrium (innermost lining) of the uterus is classified as stage I. The spread of cancer that is beyond the pelvic area is classified as stage IV cancer. Vaginal or lymph node areas of involvement indicate that the cancer is classified as stage III.

A 12-year-old girl says to the nurse, "I've had my first period, so can I have a baby now?" How does the nurse respond? A) "A boy's sperm must unite with your egg to make a baby." B) "Technically yes you can; but how can you take care of a baby?" C) "Yes, you can." D) "You are physically able to, but let's discuss becoming a parent."

Answer: D. Telling the client that she is able to have a baby and encouraging her to discuss it addresses the physiologic, psychological, and developmental facts related to the client's question.Telling the client that a boy's sperm must unite with her egg addresses physiologic facts but does not relate to the psychological issues involved with pregnancy and parenthood. Asking the client how she will take care of a baby projects the nurse's own values and is accusative and could place the girl on the defensive. Simply telling the client that she is able to have a baby is too simple and uninformative.

Which client being cared for on the medical-surgical unit will be best to assign to a nurse who has floated from the intensive care unit (ICU)? A) Recent radical mastectomy client requiring chemotherapy administration B) Modified radical mastectomy client needing discharge teaching C) Stage III breast cancer client requesting information about radiation and chemotherapy D) Client with a Jackson-Pratt drain who just arrived from the postanesthesia care unit after a mastectomy

Answer: D. The best client to assign to a nurse who has been floated from the ICU to the medical-surgical unit is the client with a Jackson-Pratt drain who just arrived for the postanesthesia care unit after a mastectomy. This nurse would be most familiar with postoperative monitoring and care of surgical clients.The recent radical mastectomy client requires chemotherapy, so it is more appropriate to assign her to nurses who are familiar with teaching, monitoring, and providing chemotherapy for clients with breast cancer. The modified radical mastectomy client who requires discharge teaching and the stage III breast cancer client requiring information about radiation and chemotherapy are more appropriate to assign to nurses who are familiar with breast cancer.

The certified nurse-midwife (CNM) completes a cervical biopsy on a client and performs postprocedure teaching. What does the CNM tell the client? A) "Abstain from intercourse for 24 hours after the procedure." B) "Rest for at least 6 hours after the procedure." C) "There is no limit on activity or weight-lifting." D) "Use the antiseptic solution rinses to clean your perineum."

Answer: D. The client must keep the perineum clean and dry by using antiseptic solution rinses as directed by the primary health care provider, and needs to change pads frequently.The client is told not have intercourse or lift heavy objects for about 2 weeks after the procedure. The client also needs to rest for 24 hours after the procedure.

Which client does the RN assess first after receiving change-of-shift report? A) A 45-year-old with a history of hypothyroidism who is scheduled for a hysterectomy and bladder suspension B) A 48-year-old who is reporting abdominal pain and light vaginal spotting after an endometrial biopsy C) A 50-year-old who is receiving morphine through a client-controlled analgesia (PCA) device after a hysterectomy and who rates her pain at a level 3 (0-to-10 scale) D) A 54-year-old with an anterior and posterior colporrhaphy who has an elevated heart rate and an oral temperature of 101.2°F (38.4°C)

Answer: D. The client with an anterior and posterior colporrhaphy with an elevated heart rate and fever is showing signs of postoperative infection and warrants frequent assessments. This information needs to be communicated to the surgeon as soon as possible.The client with a history of hypothyroidism who is scheduled for a hysterectomy and bladder suspension, the client with abdominal pain and light vaginal spotting after an endometrial biopsy, and the client receiving morphine through a PCA device with a pain level of 3 are not unusual cases and do not require rapid intervention by the nurse.

The nurse is caring for a female client with uterine leiomyoma. What is the most likely problem this client will experience as a result of this condition? A) Pain. B) Constipation. C) Infection. D) Bleeding.

Answer: D. The most likely problem for women with fibroids is the potential for prolonged or heavy bleeding.Pain is not common but may occur if the fibroid becomes twisted. Constipation may occur if fibroids are large and press on other organs. Infection is not common as a result of fibroids.

The nurse is teaching a 19-year-old female with genital warts about her condition. Which client statement requires further education from the nurse? A) "There is no known treatment that will cure genital warts." B) "The warts may actually disappear or resolve without any treatment at all." C) "Genital warts may reappear at the same site." D) "Wart remover treatment from the drugstore will help me get rid of them."

Answer: D. The nurse must teach clients that over-the-counter wart treatments must not be used on genital tissue.There is no treatment that cures genital warts at the present time. The desired outcomes of management are to remove the warts and treat the symptoms. Warts may disappear or resolve on their own without treatment. They may occur once or recur at the original site.

A client receiving radiation therapy calls the nurse to report rectal urgency, cramping, and passing of mucus and blood. What is the nurse's best response? A) "This is an emergency. Go directly to the emergency department." B) "This is normal and will resolve as soon as the treatment stops." C) "Avoid caffeine and continue drinking plenty of water and other fluids." D) "Limit spicy or fatty foods, caffeine, and dairy products."

Answer: D. The nurse's best response is to limit spicy or fatty foods, caffeine, and dairy products. The client's symptoms indicate that he is experiencing radiation proctitis (rectal mucosa inflammation), a common complication of radiation therapy.The client's symptoms do not indicate an emergency, but they need to be reported to the primary health care provider. The client's symptoms would resolve 4 to 6 weeks after the treatment stops. Avoiding caffeine and drinking water and other fluids describe what the client would do if he is experiencing radiation cystitis.

A client tells the nurse in the gynecology clinic that she doesn't get a yearly mammogram because she is afraid of what might be found. Which teaching will the nurse provide? A) If your screening for breast cancer is low risk, you can always have a MRI which is cheaper. B) Detection of breast cancer before or after axillary node invasion yields the same outcome. C) Mammography as a baseline screening is recommended by the American Cancer Society at 30 years of age. D) Mammograms help with early detection of localized breast cancer which has a high 5-year survival rate.

Answer: D. The purpose of screening is early detection of breast cancer before it spreads.MRI is used for screening high-risk women and better examination of suspicious areas found by a mammogram (ACS, 2016b). MRI is more expensive than mammography. Detection of breast cancer before axillary node invasion increases the chance of survival. It is recommended that the clinical breast examination be part of a periodic health assessment at least every 3 years for women in their 20s and 30s, and every year for asymptomatic women who are at least 40 years of age. Women ages 40 to 45 would have the choice to start annual mammograms. Women age 55 and older may switch to mammograms every 2 years.

A client is scheduled for her final preoperative visit before a vaginoplasty. Which statement by the client indicates a need for further teaching prior to surgery? A) "My surgeon wants me to take vitamin C so I will heal more quickly." B) "I have to let my doctor know I am allergic to metronidazole (Flagyl)." C) "I will need to start taking the laxatives the day before the surgery." D) "I cannot drink anything at all once I start the bowel preparation."

Answer: D. Up until the client goes to bed the night before surgery, the client would increase liquid intake since the bowel preparation can be very dehydrating. Laxatives are part of the bowel preparation that precedes vaginoplasty. This bowel preparation helps to decrease the risk of infection.Vitamin C can decrease bruising as well as promote wound healing. The client would alert the surgical team about an allergy to metronidazole so a substitution can be made because antibiotics such as neomycin and metronidazole are given prior to surgery. The bowel preparation is done the day before surgery.

A male-to-female client is beginning estrogen therapy. Which data obtained from the client's history are of particular concern to the nurse? (Select all that apply). A) Body mass index of 32 B) Client has a twin sibling C) One pack-per-day smoker D) History of environmental allergies E) Takes multiple medications for blood pressure control

Answer; A, C, E. Estrogen therapy is associated with various health risks, including venous thromboembolism, hypertension, and elevated glucose levels. A client who is already hypertensive by history would be evaluated carefully before estrogen therapy is started. Smoking and obesity increase the risk of complications. A body mass index over 30 is considered obese.Being a twin is not a cause for concern. There is no correlation between environmental allergies and estrogen therapy.

The nurse is preparing to discharge a client who has had drains placed after a mastectomy. The nurse recognizes the client understands care of the drains when she states which of these? A) "The surgeon will pull the drains before I leave the hospital." B) "The drain will help prevent postoperative infection." C) "I may shower tomorrow morning as long as I don't face the shower spray." D) "Drains are generally removed when drainage is less than an ounce (30 ml) for three consecutive days."

Answer; D. The client statement that shows that she understands care of postoperative drains is that, "Drains are generally removed when drainage is less than an ounce (30 mL/day) for three consecutive days." This may take 1-3 weeks.Due to shorter hospital stays, drains remain in place when clients are discharged. Drains are used to remove fluid that would accumulate in the incisional area and delay healing. They do not specifically prevent postoperative infection. The client may shower only after the stitches, staples, and drains are removed.

A patient in the medical-surgical unit says to the nurse, "My doctor told me I have advanced breast cancer and I want to give you this bracelet because you have been so sweet to me today." What does the nurse do first? a. Contact the health care provider because the comment signals suicide intent. b. Sit with the patient and allow her to take the lead in the conversation. c. Suggest that the bracelet should be given to a daughter or other family member. d. Explain to the patient that it is unethical for nurses to accept expensive gifts.

B

A patient is admitted with toxic shock syndrome. What organism is frequently associated with this syndrome when it occurs as a menstrual-related infection? a. Escherichia coli b. Staphylococcus aureus c. Haemophilus influenzae d. Beta-hemolytic streptococcus

B

A patient is diagnosed with uterine leiomyomas. What does the nurse expect to see in the documentation as the patient's chief presenting symptom? a. Foul-smelling vaginal discharge b. Heavy vaginal bleeding c. Intermittent abdominal pain d. Urinary incontinence

B

A patient is one day postsurgery after a mastectomy and is anxious to begin the prescribed exercises. Which exercise is appropriate for the patient's first efforts? a. Flex the fingers so that the hands slowly "walk" up the wall. B. Squeeze the affected hand around a soft, round object. C. Swing a rope in small circles and gradually increase to larger circles. D. Grab the ends of a rope and extend the arms until they are straight.

B

A patient is recently diagnosed with a fibro- cystic breast condition (FBC) and she is very fearful that she will develop breast cancer. What does the nurse tell the patient? a. "FBC is considered precancerous, so monitoring is essential." b. "There is no increased risk of developing breast cancer." c. "Features of FBC automatically rule out cancer." d. "We can review lifestyle changes that will reduce the risk."

B

A patient is taking estrogen therapy. Which patient report is cause for greatest concern? a. Breast tenderness b. Tenderness and swelling in the calf c. Nausea with vomiting d. Decreased erectile function

B

A patient received treatment for prostate cancer. Which test is most likely to be ordered to monitor the disease after treatment? a. Transrectal biopsy b. Prostate-specific antigen test c. Human papillomavirus test d. Routine prostate examination

B

A patient receiving chemotherapy treatments reports fatigue, loss of energy, and experienc- ing an "emotional crisis every day and my hair is falling out." What does the nurse do first to help the patient adapt to body changes? a. Suggest participation in self-management. b. Encourage the patient to ventilate feelings. c. Help the patient to select a wig or scarf. d. Encourage the patient to talk to her family.

B

A patient with uterine leiomyomas reports a feeling of pelvic pressure, constipation, and urinary retention. She says, "I can't button my pants anymore." What does the nurse do to further evaluate the patient's condition? a. Check the lower extremities for fluid retention. b. Assess the abdomen for distention or enlargement. c. Measure the fluid intake and urine output. d. Palpate the urinary meatus for inflammation.

B

A woman recently diagnosed with breast cancer confides, "I am going to use nutritional and herbal therapy instead of taking drugs and radiation that would make my hair fall out." What is the nurse's best response? a. "Research does not support the use of complementary therapy." b. "Have you reviewed all treatment options with your health care provider?" c. "Alternative nutritional therapies would delay or interfere with treatment." d. "Where did you hear about this nutritional and herbal treatment?"

B

A young woman discovers she has chlamydia after going to her health care provider for a routine Pap smear and pelvic exam. She is re- luctant to accept the diagnosis because she is asymptomatic and "does not have any money for unnecessary treatment right now." What is the nurse's best response? a. Explain that there is a single-dose of medication which has a one-time cost. b. Talk to the woman about her financial situation and help her find resources. c. Encourage the patient to express her reluctance and disbelief. d. Tell her that it is possible to have chlamydia without having any symptoms.

B

After surgery, a female patient has been told her breast tumor contains estrogen receptors. What is the clinical significance of this infor- mation and how will this type of cancer be treated? a. This is a triple-negative breast cancer and additional surgery is the best option. b. This type of cancer has a better prognosis and usually responds to hormonal therapy. c. This tumor is localized; therefore radiation therapy should effectively eradicate the cancer. d. There are metastases, so long-term survival rate is low; systemic therapy is the only option.

B

The health care provider tells the nurse that the patient needs testing for prostatitis. Which specimen needs to be obtained and sent to the laboratory? a. Blood sample for serum creatinine b. Prostatic fluid for culture and sensitivity c. Semen sample to test for sperm count d. Blood sample for prostate specific antigen

B

The nurse encourages a teenage patient to receive the human papillomavirus (HPV) vaccine be- cause it protects against which type of cancer? a. Endometrial cancer b. Cervical cancer c. Ovarian cancer d. Uterine cancer

B

The nurse is assessing a woman with very large breasts. In addition to the routine assessment of the breasts, what specific assessment will the nurse perform on this patient? a. Pay special attention to the size and shape of the nipples. b. Observe underneath the breasts for fungal infection. c. Ask if the patient has considered reduction mammoplasty. d. Assess for pain in the joints or bones.

B

The nurse is caring for a patient admitted for pel- vic inflammatory disease. Which task does the nurse delegate to unlicensed assistive personnel? a. Apply a heating pad to the lower abdomen or back. b. Place the patient in semi-Fowler's position. c. Ask the patient if the pain is a 2-3 on a pain scale of 0-10. d. Report to the nurse if the patient is anxious about infertility

B

The nurse is caring for a patient who had an open radical prostatectomy. During the assessment, the nurse notes that the penis and scrotum are swollen. What does the nurse do next? a. Notify the health care provider and monitor for an inability to void or increasing pain. b. Elevate the scrotum and penis; apply ice intermittently to the area for 24-48 hours. c. Assist the patient to increase mobility, especially early ambulation. d. Observe the urethral meatus for redness and discharge and monitor urine output.

B

The nurse is caring for a patient who is one day post-op for a total abdominal hysterectomy. Which assessment finding is cause for greatest concern? a. Urinary catheter is in place with a moderate amount of dark amber urine. b. Patient reports saturating one pad in an hour with dark red blood. c. Patient reports difficulty with bowels and asks for a stool softener. d. Incision is intact but appears more in- flamed than previously.

B

The nurse is caring for an older transgender adult who transitioned years ago from male-to-female. Which question is the nurse most likely to ask? a. "Have you had any hot flashes or vaginal dryness?" b. "Do you experience urinary retention or dribbling?" c. "How old were you when you went through menopause?" d. "Did you have any problems after your hysterectomy?"

B

The nurse is giving discharge teaching to a patient who had a transvaginal repair for pelvic organ prolapse using a surgical mesh. What does the nurse include? a. Avoid cigarette smoking for at least 1 month. b. Abstain from sexual intercourse for 6 weeks. c. Reduce calories to lose 2 pounds a month. d. Avoid tub baths to prevent soaking the mesh.

B

The nurse is giving instructions to unlicensed assistive personnel (UAP) about hygienic care for an older adult patient who is uncircumcised. What does the nurse instruct the UAP to do? a. Defer cleaning the penis because of patient embarrassment. B. Replace the foreskin over the penis after bathing. C. Observe the penis and the foreskin for redness or odor. D. Avoid touching the foreskin because of hypersensitivity

B

The nurse is interviewing a 56-year-old woman who reports irregular and decreased flow of menses for several months. Which question does the nurse ask the patient? a. "Have you been exposed to sexually transmitted disease?" b. "Are you having any discomfort during intercourse?" c. "Have you noticed any lumps or tenderness in your breasts?" d. "Do you have a history of frequent urinary tract infections?"

B

The nurse is interviewing a patient with erectile dysfunction. Which question would the nurse ask to assist the health care provider in differentiating the etiology as organic versus functional? a. "Have you ever had an elevated prostate- specific antigen level?" b. "Do you ever have nocturnal emissions or morning erections?" c. "Have you tried any medications or therapies for erectile dysfunction? d. "Do you have trouble passing urine or starting the stream?"

B

The nurse is interviewing a transgender patient who reports depression related to con- tinuous verbal harassment, threats, and intimi- dation. Which patient statement is the greatest concern for failure to cope with stressors? a. "I smoke three packs of cigarettes every day." b. "I have tried to commit suicide several times." c. "When I get really down, I drink and use recreational drugs." d. "Last night I went to a bar and picked up a d. stranger for sex."

B

The nurse is reviewing the laboratory results from a patient being evaluated for lower urinary tract symptoms. What does an elevated prostate-specific antigen (PSA) level and serum acid phosphatase level in this patient indicate? a. Infection b. Prostate cancer c. Benign prostatic hyperplasia d. Infertility

B

The nurse is talking to a patient who reports get- ting genital herpes (GH) several years ago. Based on the nurse's knowledge of viral shedding, what advice would the nurse give the patient? A. "If you haven't had any more episodes, the GH is inactive, so you are not contagious." B. "GH can recur and not show symptoms; infection is still possible, so always use a condom." C. "If you don't have any open sores or notice any drainage, the GH is not contagious." D. "The majority of adults have already been exposed to GH; so don't worry about passing it."

B

The nurse is teaching a patient at risk for pros- tate cancer about food sources of omega-3 fatty acids. Which food does the nurse suggest? a. Red meat b. Fish c. Watermelon d. Oatmeal

B

The nurse is teaching self-care management to a 39-year-old woman who had an abdomi- nal hysterectomy. Which point would be emphasized to avoid complications of this surgery? a. Bathe and douche daily to prevent infection. b. Take temperature twice a day for 3 days after surgery. c. Resume typical exercise routines as soon as possible. d. Gently massage calves if tenderness or swelling occurs.

B

The nurse is volunteering at a walk-in clinic where, compliance, continuity of care, and limited resources are issues for patients, but the staff tries to give good service. For which circumstance, would the nurse question the provider's decision? A. Instructs on how to use podofilox 0.5% cream for self-treatment. B. Declines to perform biopsy and prepares to eradicate warts on patient's cervix. C. Prescribes a single 2.4 million-unit dose of IM benzathine penicillin G for syphilis. D. Declines expedited partner treatment because partner may have pelvic inflammatory disease.

B

The nurse must assign unlicensed assistive personnel (UAP) to assist a patient who is undergoing brachytherapy for breast cancer treatment. What is the most important question that the nurse will ask the UAP prior to making the assignment? a. "Do you know how to dispose of radioactive body fluids?" b. "Is there any chance that you could be pregnant?" c. "Have you ever cared for a patient during brachytherapy?" d. "Have you ever had any radiation exposure?"

B

The nurse notes that the patient has just started taking an alpha-blocker medication to treat benign prostatic hyperplasia. What instruction, related to the medication side effects, will the nurse give to unlicensed assistive personnel who will assist the patient with activities of daily living? A. Frequently offer the patient the urinal. B. Have him sit up slowly and pause before standing. C. Remind the patient to drink plenty of extra fluids. D. Frequently check the linens for soiling and moisture.

B

The nurse reads in the patient's chart that the patient is experiencing surgical menopause after having a total hysterectomy and bilateral salpingo-oophorectomy. What expected sign/symptom does the nurse anticipate that the patient will report? A. Masculinization b. Vaginal changes c. Rejection by partner d. Weight gain

B

The nurse reads in the patient's medical record that the patient has gender dysphoria. Which question is the nurse most likely to ask first? A. "Are you seeking interventions for sex reassignment?" b. "How do you prefer to be addressed?" c. "Do you think of yourself as female or male?" d. "What issues of sexuality would you like to discuss?"

B

The patient had several diagnostic tests to evalu- ate lower urinary tract symptoms. Which finding suggests that the patient may have kidney disease? a. Elevated white blood cell count b. Elevated serum creatinine c. Elevated red blood cell count d. Elevated prostate-specific antigen

B

What postprocedure instructions would the nurse give to a patient who just had a colposcopy? a. Do not drive or operate heavy machinery while taking prescribed pain medication. b. Do not use tampons and abstain from sexual intercourse for at least 1 week. c. Wear a perineal pad and expect bleeding with small clots for the first 24 hours. d. Perform breast self-examination every month and report changes to provider.

B

Which woman has the highest risk for developing cervical cancer? a. Has normal Pap test result and decreased luteinizing hormone b. Has an abnormal Pap test result and a positive human papillomavirus test c. Has decreased levels of estradiol, total estrogens, and estriol d. Has abnormal findings on hysterosalpin- gography

B

Why do women develop complications more often than men when being treated for gonorrhea? a. Treatment for the disease can leave women infertile. b. Men are more likely to have symptoms and will seek curative treatment. c. Estrogen leaves the woman more resistant to antibiotic therapy. d. The disease is much more difficult to cure in women.

B

A patient is diagnosed with late tertiary syphilis. In addition to benign lesions of the skin and mucous membranes, what other findings does the nurse expect to see documented in the patient's record? a. Chronic renal failure b. Latent tuberculosis c. Cardiovascular syphilis d. Contagious chancroid lesions

C

A patient is receiving external radiation therapy for treatment of endometrial cancer. What task does the nurse delegate to unlicensed assistive personnel? a. Gently wash the markings outlining the treatment site. b. Monitor for signs of skin breakdown, especially in the perineal area. c. Assist the patient to ambulate if she feels fatigue or tiredness. d. Clean the urinary catheter and meatus with mild soap and water.

C

A patient with pelvic inflammatory disease is discharged home on oral antibiotics. What important measure should the nurse include in patient teaching? a. "Douche 3 times a week to aid healing." b. "Take antacids if the antibiotics upset your stomach." C. "Continue taking the antibiotics until the medication is gone." D. "Do not have intercourse until after the follow-up appointment."

C

A transwoman patient of color comes to the clinic for treatment because she was physically assaulted and beaten. Before the nurse begins the interview, the patient becomes angry and defensive and accuses the nurse of discrimina- tion. How does the nurse interpret the patient's behavior? a. Recognizes that own verbal and/or nonverbal behavior has offend the patient. b. Suspects that the patient is likely to be unstable because of gender dysphoria. c. Realizes that the patient is reacting to bias- related violence and emotional distress. d. Projects a lack of understanding about the sociology of transgender issues.

C

A woman has had several discussions with the health care provider about her risk for breast cancer. What would the nurse say in order to reinforce the role of prophylactic mastectomy on the risk for developing breast cancer? a. "Prophylactic mastectomy has no impact for some women." b. "The procedure eliminates the risk completely." c. "The surgery reduces the risk for breast cancer." d. "An added benefit is the decreased risk for uterine cancer."

C

According to the American Cancer Society guidelines, annual screening mammograms are not recommended for women less than 40 years old. What is the underlying rationale for this recommendation? a. Breast tumors are not very common among women under the age of 40 years. b. Amount of radiation exposure outweighs the benefit for women of childbearing age c. In younger women, there is little differ- ence in the density of normal tissue and malignant tumors. d. In younger women, the tumors are likely to be too small to be detected by mammography.

C

According to the American Psychiatric Association, which circumstance best describes gender identity? a. At birth, midwife informs parents that they have a healthy baby girl. b. Biologically, the person has obvious male sexual organs. c. A person takes on the social roles of mother, wife, and sister. d. Family, friends, and others treat the child as a little boy.

C

An African American male patient has a prostate-specific antigen (PSA) level less than 2.5 ng/mL. Which information should the nurse give to the patient? a. African American men typically have lower-than-normal PSA levels. b. Level indicates a need for follow-up for possible prostate cancer. c. PSA level of less than 2.5 ng/mL is generally considered normal. d. Test should be repeated on an annual basis to monitor the abnormality.

C

An older patient reports that he has an enlarged prostate with chronic urinary retention but declines to seek treatment because "it's been that way for a long time." The nurse would encourage a follow-up appointment to prevent which complication of this chronic condition? a. Prostate cancer b. Erectile dysfunction c. Hydronephrosis d. Testicular cancer

C

For a patient with mild discomfort from a fibrocystic breast condition, what will the nurse teach the patient about self-care measures? a. Avoid or limit reaching upward or lifting objects above the head. b. Avoid wearing a bra to decrease the pressure on the breast tissue. c. Take analgesics and limit salt before menses to help decrease swelling. d. Take selective estrogen receptor modulator as prescribed.

C

In the emergency department, a patient is diagnosed with pelvic inflammatory disease and there is an order to discharge the patient home with a prescription for antibiotics. What circumstance would cause the nurse to question the order for discharge to home? a. The patient has history of sexually transmitted disease but did not always seek treatment. b. The patient is nauseated but able to tolerate small amounts of oral fluids. c. The patient is pregnant but willing to attempt self-care if properly instructed. d. The patient is afraid to go home, but the sister and husband are available to help.

C

The health care provider informs the nurse that a male-to-female patient had a feminiza- tion laryngoplasty. The patient is requesting information on additional therapies because the surgery has not fully met her expectations. Which brochure would the nurse prepare for the patient? a. "Expected Effects of Testosterone Therapy in Sexual Reassignment" b. "How Using Estrogen Helps You to Transition Through Gender Changes" c. "Vocal Therapy: Voice and Communica- tion That Reflect Gender Identity" d. "Thyroid Chonodroplasty for Feminiza- tion of Pitch and Intonation"

C

The health care provider tells the nurse that a patient has Jarisch-Herxheimer reaction. Which interventions does the nurse anticipate? a. Oxygen, epinephrine, and antihistamines b. Emergency IV fluid resuscitation c. Analgesics and antipyretics. D. Monitoring for symptom resolution

C

The nurse has just finished teaching an 18-year-old male patient about sexually transmitted diseases and safe sex practices. Which patient statement most strongly suggests that the patient will succeed in making a behavior change? A. "I don't intend to have sex anymore until I get married." B. "You have given me a lot to think about. I know I am taking risks." C. "I am going directly to the pharmacy and buy a supply of condoms." D. "Wearing gloves during foreplay seems weird, but I gues I could try it."

C

The nurse hears in shift report that a 32-year- old patient had a prophylactic oophorectomy. What subjective symptom(s) does the nurse anticipate that the patient would report? a. Swelling of upper arms b. Tenderness of breasts c. Menopausal symptoms d. Nausea related to medications

C

The nurse is assisting a health care provider perform a genital examination on a transgender patient. During the exam, the provider is respectful and professional toward the patient. Later, the nurse hears the provider making jokes within earshot of the patient. What should the nurse do first? A. Apologize to the patient and reassure that the provider is caring and trustworthy. B. Report the provider to the appropriate licensing board for discipline. C. Take the provider aside and explain that the patient overheard what was said. D. Remind the provider about ethical responsibilities to treat patients with dignity.

C

The nurse is caring for a patient who had a right-sided modified radical mastectomy. Which task does the nurse delegate to unli- censed assist personnel? a. Observe the drainage in the Jackson-Pratt drain. b. Take blood pressure on the right arm only. c. Assist the patient to ambulate the day after surgery. d. Instruct the patient about arm positioning.

C

The nurse is caring for a patient who is taking finasteride, a 5-alpha reductase inhibitor. What question would the nurse ask to deter- mine if the medication is having the desired therapeutic effect? a. "Have you had any discharge from your penis?" b. "Has your libido returned to the way it was before?' c. "Are you having any problems with urination?" d. "Have you gotten any relief from the testicular pain?"

C

The nurse is caring for several patients who had total abdominal hysterectomies. All patients are coming to the clinic for their 6-week follow-up appointment. Which patient demeanor is the strongest indicator that there is a need for psychological referral? a. Quiet and withdrawn but asks appropriate questions b. Tense and impatient but answers questions correctly c. Disheveled and lackluster and displays a lack of interest in questions d. Cheerful and distractible and answers questions with excessive detail

C

The nurse is giving discharge teaching to a woman who had local cervical ablation. What information would be included? a. Sexual activity may be resumed usually in 1 week. b. Change tampons every 4 hours. c. Report heavy vaginal bleeding or foul-smelling drainage. d. Avoid lifting heavy objects for several days.

C

The nurse is giving instructions to a patient who is undergoing brachytherapy for cervical cancer. What information does the nurse include? a. "Limit interactions with others between treatments for their protection." b. "We will give you pain medication prior to every treatment." c. "Report any blood in the urine or severe diarrhea immediately." d. "Expect heavy vaginal bleeding during this time."

C

The nurse is interviewing a young woman who is considering the option of uterine artery embolization for the treatment of uterine fibroids. Which question would the nurse ask to assist the patient in making a decision? a. "How has the uterine fibroid condition been affecting your lifestyle?" b. "Do you have a family history of breast or uterine cancer?" c. "What did the health care provider tell you about the procedure? d. "Would you like a brochure about plan- ning future pregnancies?"

C

The nurse is reviewing prostate-specific antigen (PSA) results for a patient who had a prostatectomy for prostate cancer several months ago. The PSA level is 40 ng/mL. How does the nurse interpret this data? a. At this stage, PSA level of 40 ng/mL is expected. b. The cancer was completely removed. c. The cancer is most likely recurring. d. Prostate irritation and infection are present.

C

The nurse is reviewing the laboratory results for a patient with prostate cancer. Which laboratory result suggests metastasis to the bone? a. Decreased alpha fetoprotein b. Increased blood urea nitrogen c. Elevated serum alkaline phosphatase d. Decreased serum creatinine

C

The nurse is talking to a patient who had the relatively new procedure, prostate artery embolization. Which patient report indicates that the intended goal of therapy has been met? a. "My problem with ejaculation is much better." b. "I used the sperm bank and now I'm less anxious." c. "I am not having any more urinary symptoms." d. "My doctor said the prostate-specific antigen was good."

C

The nurse is talking with an older patient who has benign prostatic hyperplasia. Which report by the patient requires emergent care? a. "I leak and dribble urine." b. "I have to get up at night to pee." c. "I can't pass my urine today." d. "I am passing dark yellow urine."

C

The nurse is teaching a group of women about prevention of toxic shock syndrome. What preventive measure does the nurse include? A. "Use superabsorbent tampons at night." B. "Use sanitary napkins on heavy low days." C. "Change your tampon every 3-6 hours." D. "Void immediately after intercourse."

C

The nurse must insert a urinary catheter in a transgender patient who is in early transition from female-to-male. The patient appears to be a male and identifies self as male. Why would the nurse assess the genitalia prior to opening the sterile catheter kit? a. The nurse would expect to see a larger- than-average-sized male penis which may require a larger catheter. b. The presence of hormonally enhanced clitoral tissue makes catheterization very difficult and a smaller catheter may be needed. c. Genitals may not match physical appearance, so technique and equipment must be modified accordingly. d. There is a high risk for necrosis of the neopenis, which should be reported to the provider before catheterization.

C

The nurse reads in the patient's chart that the patient had a transurethral needle ablation. Which question would the nurse ask the patient to determine if the procedure achieved the intended therapeutic goal? a. "Did the pain resolve completely after the procedure?" b. "Are you able to achieve and sustain an erection?" c. "Have your problems with urination been resolved?" d. "Have you had a follow-up prostate- specific antigen level?"

C

The nurse reads in the patient's chart that there is a history of blockage in the fallopian tubes. Based on this history, what is the patient most likely to report? a. Vaginal discharge b. Amenorrhea c. Difficulty conceiving d. Difficulty controlling weight

C

The nurse sees that in addition to estrogen therapy, the patient is also taking spironolac- tone. Which laboratory result will the nurse assess to ensure the patient does not suffer adverse effects due to the medication? a. Blood glucose level b. White blood cell count c. Serum potassium level d. Platelet count

C

The patient has an indwelling catheter in place following a transurethral resection of the pros- tate. What instructions will the nurse give to unlicensed assistive personnel regarding the catheter? a. Secure the catheter so there is no tension. b. Irrigate the catheter to prevent clotting. c. Maintain traction on the catheter. d. Defer catheter care until the patient is discharged.

C

What is the major advantage of neoadjuvant therapy? a. It is the newest chemotherapy for several different types of breast cancer. b. It has fewer and milder side effects than conventional chemotherapy. c. Shrinkage of tumor allows lumpectomy rather than mastectomy. d. Tumor frequently resolves spontaneously without surgical intervention.

C

What is the primary factor for the low sur- vival rates for patients who are diagnosed with ovarian cancer? a. Ovarian cancer develops in patients with underlying immunosuppression and poor health. b. Ovarian cancer does not respond well to conventional radiation and chemotherapy treatments. c. Symptoms are mild and vague; therefore, the cancer is often not detected until its late stage. d. There are no specific diagnostic tests that can confirm or rule out ovarian cancer.

C

What preprocedural instructions would the nurse give the patient about a mammogram? a. Do not eat or drink anything 6-7 hours before the test. b. Abstain from sexual relations prior to test to avoid pregnancy. c. Do not use lotions, creams, or powder on breasts before the study. d. Wear a supportive bra and bring a breast pad for use after testing.

C

Which factor makes the mammogram a more sensitive screening tool than other tests? A. Higher compliance rate because it is done annually. B. Less expensive than other tests that identify tumor markers. C. Able to reveal masses too small to be palpated manually. D. Able to differentiate between fluid and solid masses

C

A patient has just been diagnosed with advanced breast cancer. Which behavior by the patient is the strongest indicator of readiness for additional patient teaching and information? a. Cheerfully talking about her family and the vacation they will take to Europe b. Being hostile and angrily throwing her belongings into her suitcase c. Crying and being upset, asking the nurse to call a spiritual counselor d. Being quiet and thoughtfully fingering the lace on her new bra

D

A patient is receiving internal radiation therapy (brachytherapy) and has had a low-dose radiation seed implanted directly into the prostate gland. What nursing implication is related to this therapy? a. Ensure that any staff member or visitor who is pregnant is not exposed to the patient. b. Organize the nursing care so that exposure to the patient is limited to a few minutes. c. Instruct all staff that all urine specimens should be immediately discarded. d. Teach the patient that fatigue is common but should pass after several months.

D

A patient phones the clinic because of a one-time exposure to syphilis that occurred about 6 weeks ago. He reports being asymp- tomatic and abstinent since the incident. What is the nurse's best response? a. "The first sign is a chancre which will usually develop by the third week." b. "Continue abstinence for up to 90 days and observe the genitalia for painless sores." c. "Use a latex or polyurethane condom for genital and anal intercourse." d. "The chancre can appear and then disappear, so you should get tested."

D

A patient with a sexually transmitted disease has no reaction to a penicillin skin test, so the health care provider orders benzathine penicil- lin G intramuscularly. What does the nurse do immediately after the injection? a. Ask the patient to give contact informa- tion for all sexual partners. b. Instruct the patient to go home to rest for 2-4 hours. c. Observe the patient for 4 hours to detect any allergic reaction. d. Observe the patient for at least 30 minutes to detect any allergic reaction.

D

A patient with pelvic inflammatory disease is on bedrest with bathroom privileges. What position is best for the patient while on bedrest? a. Prone b. Supine c. Side-lying d. Semi-Fowler's

D

An obese 59-year-old patient describes exces- sive menstrual bleeding that occurs approxi- mately every 10 days. Which question should the nurse ask first? a. "Have you noticed fever or signs of infection?" b. "Did you ever use oral contraceptives?" c. "When was the last time you had intercourse?" d. "How many pads (or tampons) do you use each day?"

D

For a patient who is taking estrogen therapy, which vital sign is most important to assess for the detection of a health risk that can be caused by this therapy? a. Temperature b. Pulse c. Respiration d. Blood pressure

D

The nurse is caring for an older patient who had an indwelling urinary catheter inserted after a transurethral resection of the prostate. The patient is intermittently confused, and picks at the IV tubing and the catheter. What should the nurse try first? a. Obtain an order to restrain the patient's hands and forearms. b. Sedate the patient until the IV tube and catheter can be removed. c. Inform the family that a family member will have to sit by the patient. d. Give the patient a familiar object to hold, such as a family picture.

D

The nurse is examining the breasts of an older woman. What would be considered a normal finding? a. Gentle palpation elicits reports of pain or discomfort. B. Tissue is difficult to palpate because of fat deposits. C. Nipples are retracted and there is a brownish discharge. D. Breasts are atrophied, flattened, and elongated

D

The nurse is preparing to assess an obese patient who reports subjective symptoms and urinary patterns associated with benign pros- tatic hyperplasia. Which technique does the nurse use to perform the physical assessment? a. Instruct the patient to undress from the waist down, then inspect and palpate the bladder. B. Have the patient drink several large glasses of water and percuss the bladder. C. Apply gentle pressure to the bladder to elicit urgency, then instruct the patient to void. D. Instruct the patient to void and then use the bedside ultrasound bladder scanner.

D

The nurse is taking a history on a patient who has risk for problems related to reproductive health. Despite the nurse's best attempts to establish rapport and trust, the patient absolutely refuses to answer questions about sexual practices. What should the nurse do? a. Tell the patient that all information is confidential. b. Advise the patient that withholding information can be dangerous. c. Rephrase questions to make them less offensive. d. Respect the patient's choice to refuse to answer questions.

D

The nurse is taking a history on a patient with probable gynecologic cancer. Which clinical manifestation is a sign of metastasis? a. Watery vaginal discharge b. Constipation c. Dyspareunia d. Dysuria

D

The nurse is talking to a woman who was diagnosed with breast cancer at the age of 30. The woman has a 6-year-old daughter. What advice does the nurse give about breast cancer screenings for the daughter? a. Your pediatrician should begin annual screenings even at age 6. b. When your daughter starts menstruating, screenings should start. c. Screenings should begin when she turns 29 or 30 years old. d. Screenings should begin when she turns 20 years old.

D

The nurse is teaching a 24-year-old patient about breast self-examination (BSE). What does the nurse tell the patient about the best time to perform BSE? a. Day before her menstrual flow is due b. Third day after menstrual flow starts c. Whenever ovulation occurs d. One week after her menstrual period

D

The nurse is using the International Prostate Symptom Score to assess a patient. Which data does the nurse intend to obtain through the use of this assessment tool? a. Patient's attitudes and beliefs about prostate surgery b. Pattern of growth of prostate and correlation with symptoms c. Data in aggregate that can be used for prostate research d. Effect of urinary symptoms on the quality of life

D

Which diagnostic test is considered the most definitive for diagnosing breast cancer? a. Magnetic resonance imaging b. Mammography c. Breast tomosynthesis d. Breast biopsy

D

Which disorder is strongly associated with prolonged exposure to estrogen without the protective effects of progesterone? a. Endometriosis b. Uterine cancer c. Leiomyomas d. Endometrial cancer

D

Which intervention would be used for a patient after a modified radical mastectomy? a. Position the patient on the affected side to aid flow of drainage from the incision site. b. Place arm on the affected side in a dependent position postoperatively. c. Give pain medication so that arm exer- cises can begin as soon as possible. d. Teach signs and symptoms of infection and how to monitor for altered wound healing.

D

Which man has the highest risk for prostate cancer? a. A 65-year-old Caucasian American man who has two cousins with prostate cancer b. A 45-year-old Asian American man with a history of benign prostatic hyperplasia c. A 55-year-old Hispanic American man who has poor dietary practices d. A 75-year-old African American man whose brother had prostate cancer

D

Which patient is most likely to be accepting of surgery and demonstrate better coping behaviors? a. 62-year-old woman with an active social and work life has an abdominal hysterec- tomy for advanced ovarian cancer. b. 23-year-old woman with BRCA1 and BRCA2 genes elects to have a prophylactic bilateral salpingo-oophorectomy. c. 58-year-old woman with one supportive adult child has ovarian cancer which was treated with cytoreduction. d. 44-year-old woman with two children and supportive partner had an abdominal hysterectomy for uterine fibroids.

D

Which sign/symptom detected during clinical breast examination suggests advanced breast disease? a. Gynecomastia b. Oval-shaped, mobile, rubbery mass c. Thin, milky discharge from the nipple d. Skin change of peau d'orange

D

_____: A glycoprotein produced solely by the prostate. The normal blood level of PSA is less than 4 ng/mL; levels are higher in patients with increased prostatic tissue as a result of benign prostatic hyperplasia, prostatic infarction, prostatitis, and prostate cancer. Levels associated with prostate cancer are usually much higher than those occurring with other prostate tissue enlargement.

Prostate-specific antigen (PSA)

_____: The traditional "closed" surgical procedure for removal of the prostate. In this procedure, the surgeon inserts a resectoscope (an instrument similar to a cystoscope, but with a cutting and cauterizing loop) through the urethra. The enlarged portion of the prostate gland is then resected in small pieces.

Transurethral resection of the prostate (TURP)

A 23-year-old woman is diagnosed with a chlamydial infection but is reluctant to spend the money for treatment because she is asymptomatic and does not have a job or health insurance. The nurse advises her that chlamydial infections can result in which condition? a. Female infertility b. Male partner infertility c. Amenorrhea d. Teratogenic effects

A

A patient had loop electrosurgical excision procedure for treatment and diagnosis of cervical cancer. In the discharge instructions, what does the nurse tell the patient to expect after the procedure? a. Spotting b. Menses-like vaginal bleeding c. Cramps lasting 24 hours d. Watery discharge

A

A patient tells the nurse that he was diagnosed with benign prostatic hyperplasia. Based on this diagnosis, which symptom is the patient most likely to report? A. Pain in the scrotum b. Trouble passing urine c. Erectile dysfunction d. Constipation

B

The nurse writes a client problem of "anxiety related to potential sexual dysfunction" for a client diagnosed with cancer of the prostate. Which intervention should the nurse implement? 1. Tell the client to discuss his fears with the HCP. 2. Talk to the wife about the client's concerns. 3. Inform the client sexual functioning will not be altered. 4. Provide a private area for the client to discuss his concerns.

ANSWER: 4. 1. This is not an appropriate referral. The nurse should discuss the client's fears. 2. The wife may need to be encouraged to talk about her concerns, but this does not address the client's concerns. 3. The client's sexual functioning may be impaired depending on the treatment options chosen. 4. Because the client may be sensitive and embarrassed about discussing problems related to genitalia and sexual functioning, the nurse should provide a private area in which to discuss these concerns.

_____: Surgery for severe symptoms of cystocele in which the pelvic muscles are tightened for better bladder support.

Anterior colporrhaphy

A female patient is tested for and diagnosed with gonorrhea. The nurse advocates that the choice of drug therapy include medications that concurrently treat which condition? A. Genital herpes b. Chlamydia c. Syphilis d. Genital warts

B

_____: The absence of living sperm in the semen.

Azoospermia

A 36-year-old patient is diagnosed with dys- functional uterine bleeding. During the pelvic exam, the health care provider determines that the bleeding is acute. What is the nurse's priority action? a. Prepare the patient for transfer to the operating room. b. Prepare to assist with a dilation and curettage. c. Anticipate an order for oral contraceptive therapy. d. Obtain an order for injectable medroxyprogesterone acetate.

C

A female patient is diagnosed and treated for sexually transmitted disease. What does the nurse teach the patient about resuming sexual relations? a. Douche within 24 hours after vaginal intercourse. b. Sexual relations are prohibited for 3 months. c. Intercourse should be postponed until the treatment regimen is completed. d. Intercourse is permitted unless there is an increase in abdominal pain.

C

A nurse is working at an ambulatory clinic. Which patient is most likely to need a pelvic examination? a. 12-year-old whose mother desires a "virgin check" b. 25-year-old with a possible urinary tract infection c. 62-year-old who reports resumption of menses d. 53-year-old who reports decreased libido

C

A patient who has testicular cancer is likely to have which common problem? a. Priapism b. Erectile dysfunction c. Azoospermia d. Cryptorchidism

C

Disorders that alter a woman's metabolism or nutrition can result in which condition? a. Excessive bleeding b. Endometriosis c. Amenorrhea d. Pelvic inflammatory disease

C

_____: Herniation of the bladder into the vagina.

Cystocele

A 20-year-old woman is being evaluated for possible toxic shock syndrome. What question would the nurse ask? a. "How many pads do you use on heavy flow days?" b. "Have you ever used intravaginal estrogen therapy?" c. "Do you have a history of multiple sexual partners?" d. "Do you use internal contraceptives?"

D

A 60-year-old female patient informs the nurse that she has experienced some vaginal changes since menopause. What gynecologic change is the patient most likely to report? A. Thin, white vaginal drainage b. Vaginal odor c. Excessive vaginal bleeding d. Vaginal dryness

D

A 72-year-old patient admitted to the medical- surgical unit tells the nurse that he has benign prostatic hyperplasia. Which question will the nurse ask? a. "Have you had chemotherapy or radiation treatments?" b. "Were you recently diagnosed with benign prostatic hyperplasia?" c. "Would you like to review information about nutrition therapy?" d. "Are you having urinary incontinence or frequency at night?"

D

A female-to-male patient is seeking informa- tion about sex reassignment surgery. Which reconstructive surgery is the most difficult and the least likely to yield patient satisfaction? a. Bilateral mastectomy b. Hysterectomy and bilateral salpingo- oophorectomy c. Pectoral muscle implants d. Phalloplasty

D

A male patient had mumps as a child which caused orchitis. Which potential complication could result? a. Decreased libido b. Chronic urinary infection c. Enlarged prostate gland d. Testicular atrophy

D

_____: Breast imaging procedure that allows the radiologist to visualize through layers or "slices" of breast tissue, similar to a CT scan.

Digital 3D mamography

_____: The abnormal occurrence of endometrial tissue outside the uterine cavity.

Endometriosis

_____: Physiologic nodularity of the breast that is thought to be caused by an imbalance in the normal estrogen-to-progesterone ratio. It is the most common breast problem of women between 20 and 30 years of age.

Fibrocystic breast condition (FBC)

_____: An acute, recurring incurable viral disease of the genitalia caused by the herpes simplex virus and transmitted through contact with an infected person. An outbreak typically is preceded by a tingling sensation of the skin followed by the appearance of vesicles (blisters) on the penis, scrotum, vulva, perineum, vagina, cervix, or perianal region. The blisters rupture spontaneously, leaving painful erosions. After the lesions heal, the virus remains dormant, periodically reactivating with a recurrence of symptoms.

Genital herpes (GH)

_____: A person's genital anatomy present at birth. Also known as biological sex.

Natal sex

_____: The surgical removal of one or both testes.

Orchiectomy

_____: Surgical removal of the ovary.

Oophorectomy

_____: The involuntary loss of urine when the bladder is overdistended.

Overflow urinary incontinence

_____: Any infection of the pelvis involving the upper genital tract beyond the cervix in women. It occurs when organisms from the lower genital tract migrate from the endocervix upward through the uterine cavity into the fallopian tubes.

Pelvic inflammatory disease (PID)

_____: Condition in which the sling of muscles and tendons that support the pelvic organs becomes weak and is no longer able to hold them in place.

Pelvicorgan prolapse (POP)

_____: The surgical procedure to repair a rectocele by strengthening pelvic supports and reducing the bulging.

Posterior colporrhaphy

_____: A protrusion of the rectum through a weakened vaginal wall.

Rectocele

_____: Breast reduction surgery in which the surgeon removes excess breast tissue and then repositions the nipple and remaining skin flaps to produce an optimal cosmetic effect.

Reduction mammoplasty

_____: Interventions that reduce the risk of nonintact skin or mucous membranes coming in contact with infected body fluids and blood, such as using a condom.

Safer sex practices

_____: Infection of the fallopian tube.

Salpingitis

_____: Any of a group of diseases caused by infectious organisms that have been passed from one person to another through intimate contact. Some organisms that cause these diseases are transmitted only through sexual contact. Other organisms are transmitted by parenteral exposure to infected blood, fecal-oral transmission, intrauterine transmission to the fetus, and perinatal transmission from mother to neonate.

Sexually transmitted disease (STD) (Sexually transmitted infections, STIs)

_____: A severe illness caused by a toxin produced by certain strains of Staphylococcus aureus. It was first recognized in 1980 as related to menstruation and tampon use. It is characterized by abrupt onset of a high fever and headache, sore throat, vomiting, diarrhea, generalized rash, and hypotension. The most common manifestations are skin changes (initially a rash that resembles a severe sunburn and changes to a macular erythema similar to a drug-related rash).

Toxic shock syndrome (TSS)

_____: Procedure for treating benign prostatic hyperplasia using high temperatures to heat and destroy excess tissue.

Transurethral microwavetherapy (TUMT)

Cancer surveillance for high-risk women is used to detect cancer in its early stages and is referred to as what kind of prevention? a. Primary b. Secondary c. Tertiary d. Prophylactic

B

For a patient with a low testosterone level, what is the patient most likely to report? a. Increased weight and muscle mass b. Changes in urinary pattern c. Problems with sexual performance d. Testicular pain with nausea

C

A patient stopped having menses about a year 27. ago. When does the nurse advise the patient to perform breast self-examination (BSE)? a. The first day of every other month b. After menopause, BSE does not detect masses c. The last day of each month d. Any day of the month, but follow a consistent schedule

D

A male-to-female patient underwent a vaginoplasty with epidural anesthesia. What is an expected assessment finding during the first several hours after surgery? a. Patient is unable to move legs. b. The pedal pulses are diminished. c. Feet and ankles are cold to the touch. d. Patient has a decreased level of consciousness.

A

A patient calls to make an appointment for a routine pelvic exam which includes a Pap smear. What type of instructions does the nurse give the patient about preparing for the exam? a. "Do not have intercourse for at least 24 hours before the exam." b. "Do not eat or drink anything after midnight." c. "Clean the genitals with mild soap and water before the exam." d. "Do not wear a tampon if you are menstruating."

A

A patient had a mastectomy several years ago and calls the nurse to report sensations of heaviness, aching, fatigue, numbness, tingling, and swelling in the affected arm and upper chest. Which intervention would the nurse use? a. Arrange an immediate appointment with a lymphedema specialist. b. Tell patient to elevate the arm during sleep until symptoms resolve. c. Advise use of a supportive sling for at least 10-12 hours a day. d. Advise restarting postoperative exercises that were taught at the hospital.

A

A patient had a transrectal ultrasound with biopsy. After this procedure, what does the nurse instruct the patient to do? a. Report fever, chills, bloody urine, and any difficulty voiding. b. Limit fluid intake for several hours after the procedure. c. Expect decreased urine output for 24 hours after the procedure. d. Expect some mild perineal and abdominal pain

A

A patient had breast reconstruction surgery two days ago. A Jackson-Pratt drain was placed to collect serosanguineous fluid. The nurse notices at 7:00 am that the drainage container contains 150 mL. It was last emptied at 6:00 am. What is the priority nursing intervention? A. Notify the health care provider about amount and appearance of drainage for the past hour. B. Empty the drain every 2 hours and flush the tubing so the suction will be more effective. C. Assess for pain, or other symptoms; chart the type and amount of drainage and continue to monitor. D. Reinforce the drainage site with a sterile bulky dressing and recheck the site in one hour.

A

A patient has an enlarged prostate. Which procedure does the nurse anticipate that the health care provider will order to test for bladder outlet obstruction? a. Urodynamic pressure-flow study b. Bladder scan c. Transrectal ultrasound d. Computer tomography scan

A

A patient has had a posterior colporrhaphy. What is included in the nursing care of this patient? a. Give pain medication before a bowel movement. b. Obtain an order for prn laxatives. c. Resume regular activities after discharge. d. Promote a high-fiber diet.

A

A patient is admitted for emergency surgery after an accident. In addition, the patient has a pustular rash related to secondary syphilis. What instructions does the nurse give to unlicensed assistive personnel ? a. Gloves should always be worn when caring for or touching the patient. b. The lesions are highly contagious, so the patient should do own hygienic care. c. No instructions are given because patient confidentiality is essential. d. If the skin is open and draining pus or fluid, use gloves during patient care.

A

A patient is diagnosed with early latent syphilis. Which assessment will the nurse make related to the medication that is likely to be prescribed for the patient? a. Assess for allergies to penicillin. b. Check results of coagulation studies. c. Assess for history of noncompliance. d. Ask if the patient takes nitrates.

A

A young patient is diagnosed with testicular cancer. He and his wife have been trying to conceive a child for several months. What information does the nurse give the couple about sperm storage? a. Arrangements for sperm storage should be made as soon as possible after diagnosis. b. Sperm collection should be completed after radiation therapy or chemotherapy. c. Two or three samples should be collected 6 days apart. d. Saving sperm helps to alleviate fears related to erectile dysfunction.

A

The nurse is caring for a patient with a radioac- tive implant in the uterus. Which instruction will the nurse give to unlicensed assistive personnel? a. Patient is on bedrest and excessive move- ment is restricted. b. Assist the patient to ambulate in the hall at least three times per shift. c. Assist the patient to get up to the toilet or the commode chair. d. Linens and patient gown should be frequently changed for drainage.

A

The nurse is counseling a patient who is expe- riencing recurrent outbreaks of genital herpes. What suggestions for symptomatic treatment does the nurse include? a. Oral analgesics, sitz baths, and increased oral fluid intake b. Topical anesthetics, nutritional therapy, and warm compresses c. Abstinence, frequent bathing, and fluid restriction d. Bedrest and application of podofilox 0.5% solution

A

The nurse is helping a patient schedule an ap- pointment for a hysteroscopy. When should the procedure be done? a. 5 days after menses have ceased b. 5 days before the beginning of menses c. During the menstrual period d. Whenever she can take 3-4 days off of work

A

The patient tells the nurse that he had unpro- tected sexual intercourse 1 week ago and just found out that the person might have gonorrhea. The patient reports that he has been watching for symptoms but has not noticed anything, so he is hoping that he is okay. What is the nurse's best response? a. "Symptoms usually develop in 3-10 days, but you should be tested even if there are no symptoms." b. "Abstain from sex for 7 more days, then come in for testing. Meanwhile, watch for symptoms." C. "If you are not having any symptoms after 1 week, it is unlikely that you were infected." D. "We can give you a prescription for antibiotics, but you must avoid sex for 2-3 months."

A

What information would the nurse give to a sexually active 35-year-old woman about conventional Papanicolaou (Pap) smear and human papillomavirus (HPV) testing? a. Every 5 years is sufficient. b. Annual screening is recommended. c. Testing can stop after three normal Pap smears. D. If there are no risk factors, testing is not necessary.

A

What sexually transmitted organisms are most often responsible for pelvic inflammatory disease? a. Chlamydia trachomatis and Neisseria gonorrhoeae b. Herpes simplex virus and Escherichia coli c. Haemophilus influenzae and staphylococcus d. Treponema pallidum and Gardnerella vaginalis

A

The nurse is reviewing laboratory results for a patient with pelvic inflammatory disease. Which lab results does the nurse expect to see? Select all that apply. a. Elevation in white blood cell count b. Elevation in erythrocyte sedimentation rate c. Elevation of C-reactive protein d. Presence of human chorionic gonadotropin in urine e. Cervical infection with Neisseria gonorrhoeae or Chlamydia trachomatis f. Presence of virus in polymerase chain reaction

A,B,C,E

The nurse is taking a health history and inquir- ing about any interventions that the patient has had for gender reassignment. Which questions would the nurse include? Select all that apply. a. "Have you made any changes in gender expression, such as clothes or hairstyle?" b. "Have you ever had psychotherapy for body image or to strengthen coping mechanisms?" c. "Are you currently taking any hormonal therapy to feminize or masculinize your body?" d. "How have your parents and other people responded to changes in your appearance?" e. "Have you had or considered having surgery to change sexual characteristics?" f. "Have you tried attending a support group that would reaffirm your natal sex role?"

A,B,C,E

According to the World Professional Associa- tion for Transgender Health, which patient(s) would not meet the criteria for hormone therapy? Select all that apply. a. A 16-year-old who has had gender dysphoria since early childhood. b. A 35-year-old lesbian with history of suicide attempts. c. A 62-year-old with known gender dysphoria who has symptoms of dementia. d. A 30-year-old with gender dysphoria and no physical or mental health problems. e. A 22-year-old who has bisexual relation- ships but calls self queer/questioning. f. A 40-year-old who would like to tempo- rarily try out being the opposite gender.

A,B,C,E,F

A 37-year-old patient reports abnormal vaginal bleeding not related to her menstrual cycle. The nurse would ask the patient about which associ- ated symptoms? Select all that apply. a. Pain b. Change in bowel habits c. Breast mass d. Abdominal fullness e. Urinary difficulties f. Fluid intake

A,B,D,E

The nurse is teaching a patient about self-care following an open radical prostatectomy. What does the nurse include in the health teaching? Select all that apply. a. Teach how to care for the indwelling catheter and manifestations of infection. b. Instruct to walk short distances. c. Instruct to have prostate-specific antigen testing 12 weeks after surgery and then once a year. d. Advise to maintain an upright position and not to walk bent or flexed. e. Advise to shower rather than soak in a bathtub for the first 2-3 weeks. f. Teach to use enemas or laxatives as needed to prevent straining.

A,B,D,E

What features would be considered normal findings for the scrotum of a young white male? Select all that apply. a. Suspended below the pubic bone b. Tenderness with gentle palpation c. Contracts with exposure to cold d. Sparse hair follicles e. Skin of pouch is thin walled f. Warm compared to surrounding tissues

A,C,D,E

The nurse is interviewing a patient to determine the presence of lower urinary tract symptoms associated with benign prostatic hyperplasia. Which questions would the nurse ask? Select all that apply. A. "Do you have difficulty starting and continuing urination?" b. "Have you ever had a testicular infection?" c. "Do you have reduced force and size of the urinary stream?" d. "Have you noticed dribbling or leaking after urinating?" e. "How many times do you have to get up during the night to urinate?" f. "Have you noticed blood at the start or at the end of urinating?"

A,C,D,E,F

The nurse is teaching a patient who had an open retroperitoneal lymph node dissection. What instructions does the nurse give to the patient? Select all that apply. a. Do not lift anything over 15 lbs. b. Limit intake of fluids to 1000 mL per day. c. Do not drive a car for several weeks. d. Perform monthly testicular self- examination on the remaining testis. e. Have follow-up diagnostic testing for at least 3 years after the surgery. f. Report fever, drainage, or increasing tenderness or pain around the incision.

A,C,D,E,F

A client has a continuous bladder irrigation after surgery yesterday. The amount of bladder irrigating solution that has infused over the past 12 hours is 1050 mL. The amount of fluid in the urinary drainage bag is 1825 mL. The nurse records that the client had ____ mL urinary output in the past 12 hours.

ANS: 775. The fluid in the drainage bag consists of the irrigating solution (irrigant) and actual urine produced by the client (for a total of 1825 mL). Therefore, if the client had 1050 mL of irrigant infused, then the actual amount of urine output is 1825 mL - 1050 mL = 775 mL urine

A patient had a partial mastectomy. When teaching about care of the arm on the affected side, what does the nurse tell the patient? a. Do not start any arm or hand exercises until the drains are removed from the incision. b. Begin using your arm for normal activi- ties, such as eating or combing your hair. c. No one should take a blood pressure on the affected arm for 6 months after sur- gery. d. Do push-ups and arm circles on a routine basis for a full recovery.

B

A patient had an anterior colporrhaphy and is returning to the clinic for the follow-up appointment. Which patient statement indicates that the procedure has achieved the desired therapeutic outcome? a. "The abdominal pain is almost gone." b. "I have good control over my urination." c. "That constipated feeling has resolved." d. "My vaginal bleeding has resolved."

B

A patient has undergone a total hysterectomy with vaginal repair. Which over-the-counter product will the nurse recommend to decrease sexual discomfort related to intercourse? a. Hydrocortisone cream b. Water-based lubricants c. Petroleum jelly d. Vitamin A and D ointment

B

A patient reports a possible exposure to syphilis. Which screening tests are typically done first? a. Serology testing, such as glycoprotein G antibody-based, to identify either type 1 or 2 b. Venereal Disease Research Laboratory serum test and the more sensitive rapid plasma reagin c. Fluorescent treponemal antibody absorp- tion test or microhemagglutination assay for Treponema pallidum d. Viral cell culture or polymerase chain reaction assays of the lesions

B

A patient reports finding a mass in her breast 6 months ago. What question does the nurse ask related to possible metastases of a potential cancer? a. "Why did you wait 6 months before seeking medical attention?" b. "Have you noticed any joint or bone pain or other changes in your body?" c. "Have you ever had any exposure to radiation or toxic chemicals?" d. "Has your sister or mother ever been diagnosed with breast cancer?"

B

A patient reports having erectile dysfunction and is seeking a prescription for sildenafil. Because of the potential for dangerous drug-drug interac- tions, the nurse asks the patient specifically if he takes which type of medication? a. NSAIDs b. Nitrates c. Opioids d. Antilipemics

B

A patient with human papillomavirus will commonly develop which disease? a. Human immunodeficiency virus b. Genital warts c. Vulvovaginitis d. Chancroid

B

According to the American Cancer Society rec- ommendations, which healthy woman with no previous history of an abnormal Pap test should be advised to have a Pap test every 3 years? a. 18-year-old b. 21-year-old c. 35-year-old d. 70-year-old

B

If the prostate gland is not functioning correctly, which statement typifies what the patient is most likely to report? a. "Well, I am not as strong or as muscular as I was when I was younger." b. "My wife and I have been trying to conceive a child for several years." c. "Occasionally, if I am tired I have prob- lems getting an erection." d. "My libido is fine, but sometimes I have premature ejaculation."

B

The female-to-male patient reports that he has been taking testosterone therapy. What is an indication that the medication is having the desired effect? a. Patient reports a decreased libido. b. Nurse observes increased body hair. c. Nurse observes average male penis. d. Patient reports breast tenderness.

B

The health care provider has just informed the patient about the diagnosis and complications of salpingitis. Which intervention is the nurse most likely to use with this patient? a. Review nonpharmacologic methods to control chronic pain. b. Use empathetic listening for feelings related to possible infertility. c. Inform that annual Pap smears are recommended for salpingitis. d. Review expected changes that will occur with menstrual flow.

B

The nurse is talking to a 25-year-old African American woman whose health history and lifestyle do not suggest an increased risk for sexually transmitted disease (STD); however, which question will the nurse ask that addresses the concept of health care disparities? a. "Would you like a brochure about incidence of STDs among African Americans?" b. "Do you have a primary care provider who you see on a regular basis?" c. "Can you describe safer sex practices that you and your partner(s) use?" d. "Do you have a family history for any type of reproductive cancers?"

B

The nurse is talking to a 35-year-old African American man about prostate-specific antigen (PSA) testing. The patient tells the nurse that his father was diagnosed with prostate cancer in his 50s. What should the nurse tell the patient? a. "Although authorities do not always agree, PSA testing usually starts at age 50." b. "Your genetic and racial risk factors suggest testing should begin at age 45." c. "Because of your African American heritage, you should start testing now." d. "PSA testing can be started at any time for all males at any age."

B

The nurse is talking to a patient who is about to undergo a hysterosalpingogram. The patient discloses information that may not have been available to the health care provider when the test was initially scheduled. Which disclosure could cause the provider to cancel or reschedule the test? a. Took over-the-counter acetaminophen 1 hour ago. b. Has fever with malodorous vaginal discharge. c. Has a family history of uterine fibroids. d. Is on day 10 of the menstrual cycle.

B

What is the most common symptom that leads a patient with pelvic inflammatory disease to seek medical health care? a. Vaginal itching b. Lower abdominal pain c. Malaise with fever d. Abnormal menstrual flow

B

Women who have a personal history of breast cancer are at high risk for developing a recur- rence. Which factors contribute to recurrence? Select all that apply. A. BRCA3 genetic mutation b. Strong family history of breast cancer c. BRCA2 genetic mutation d. BRCA1 genetic mutation e. History of breast reduction f. History of fibrocystic breast condition

B,C,D

What are common sites of metastasis for prostate cancer? Select all that apply. a. Pancreas b. Bones of the pelvis c. Liver d. Lumbar spine e. Lungs f. Kidneys

B,C,D,E

The nurse is caring for a patient who had hysteroscopic surgery. The nurse is vigilant to assess for signs and symptoms of which potential complications? Select all that apply. a. Postembolectomy syndrome b. Fluid overload c. Embolism d. Perforation of uterus e. Hemorrhage f. Ureter injury

B,C,D,E,F

A patient reports an itching or tingling sensation felt in the skin 1-2 days followed by a blister on the penis which ruptured sponta- neously with painful erosion. These symptoms are consistent with which condition? a. Syphilis b. Genital warts c. Genital herpes d. Gonorrhea

C

A patient reports having uncomfortable and unsettling episodes of "hot flashes" after receiving hormonal therapy for a prostate tumor. To alleviate this symptom, the nurse would obtain an order for which medication? A. Bisphosphonate drug such as pamidronate b. Antiandrogen drug such as bicalutamide c. Hormonal inhibitor drug such as megestrol acetate d. Antimuscarinic agents such as tolterodine

C

A patient had a total abdominal hysterec- tomy. Which patient behavior is the best in- dicator that she is coping and adapting suc- cessfully? A. Refuses to look at the wound but encourages the nursing students to look. B. Sits quietly and passively while the nurse performs wound care. C. Asks questions about the wound care but seems reluctant to do self-care. D. Frequently stares at the wound site but refuses to touch the area.

C

A patient has just been informed by the health care provider that she has specific BRCA1 and BRCA2 gene mutations. Which brochure would the nurse prepare for the patient? a. "Role of Nutrition Therapy in Reproduc- tive Health" b. "Risk Factors and Treatments for Infertility" c. "Risk Factors and Treatments for Breast Cancer" d. "Colposcopy and Other Tests for Cervical Cancer"

C

A patient identifies self as genderqueer. How does the nurse interpret this? a. Patient's significant other will be the same gender. b. Patient prefers to have male sexual partners. c. Patient's gender identity does not conform to male or female. d. Patient identifies self as female, but natal sex is identified as male.

C

A patient is being discharged with a prescrip- tion for tamoxifen to decrease the chance of breast cancer recurrence. Because of the common side effect, what does the nurse suggest to the patient? a. Have soda crackers and ginger ale on hand. b. Install a handrail around the bathtub. c. Purchase a scale to monitor body weight. d. Buy a soft-bristle toothbrush.

C

A patient is considering estrogen therapy for male-to-female transition. What is an expected physical change that will occur with this therapy? a. Weight loss b. Thicker, longer hair c. Decreased testicular size d. Feminization of vocalization

C

A patient is diagnosed with Condylomata acuminata. What are the desired outcomes of management for this patient? a. Reduce pain and prevent recurrence. b. Prevent long-term complications to the cardiac system. c. Remove the warts and treat the symptoms. d. Prevent infertility and systemic infection.

C

A patient with a sexually transmitted disease freely admits to being a commercial sex worker. In talking to this patient, the nurse recognizes that she has not disclosed a true name, address, or partner contact information. What is the best treatment strategy to use with this patient? a. Reassure her that all health data are confi- dential and will be handled with discretion. b. Spend extra time with the patient to elicit trust so that she will give correct infor- mation. c. Administer a one-dose course of treatment and dispense a box of condoms. d. Give her all medications for a 7-day treat- ment and convey a nonjudgmental attitude.

C

The nurse is working in an inner-city clinic that serves a diverse population. While all patients are apt to ask for directions to restrooms, the staff is continuously debating about how to direct LGBTQ patients to public restrooms. What should the nursing staff do? a. Agree to give general directions to both male and female restrooms to everyone. b. Ask administration to post more signs so that directions to restrooms are explicit. c. Advocate for creation of designated unisex or single-stall restrooms. d. Directly ask patients if they would like directions to the male or female restrooms.

C

An older adult patient had a transurethral resection of the prostate at 8:00 am. At 3:00 pm, the nurse assesses the patient. Which finding does the nurse report to the health care provider? a. Patient reports a continuous urge to void. b. Patient keeps attempting to void around catheter. c. Patient wants to get out of bed d. Patient keeps moving and ketchup-like urine output is noted.

D

A patient had a partial mastectomy yesterday and the nurse notes that the patient is very anxious because of removal of breast tissue. What is the nurse's priority intervention? a. Use distraction until the patient improves and can think more clearly. b. Encourage the patient to have a positive attitude so she will heal faster. c. Ensure that the patient takes prn anxio- lytic medication every 4-6 hours. d. Encourage the patient to discuss her fears and ask questions about her concerns.

D

A patient had a transurethral resection of the prostate and has a three-way urinary catheter taped to the left thigh. What does the nurse instruct about the position of the left leg? A. Maintain slight abduction. B. Maintain slight flexion of the hip. C. Keep the leg elevated. D. Keep the leg straight

D

A patient has just been diagnosed with breast cancer and informed that surgery is likely to be needed. The patient seems anxious and upset. What is the priority nursing care for this patient? a. Provide patient education about treatment options. b. Assist the patient to make independent decisions. c. Provide reassurance about long-term outcomes. d. Allow patient to talk openly about feelings.

D

In males who have breast cancer, what is the most typical presenting sign/symptom? a. Nipple discharge with gross blood b. Localized red and painful lump c. Dimpling or orange peel appearance d. Hard, painless, subareolar mass

D

In recalling dietary intake for a recent 24-hour period, a female patient describes eating eggs, whole milk, and bacon for breakfast; fried chicken and French fries for lunch; three-cheese pizza and ice cream for dinner. This type of diet places her at increased risk for which disorder? a. Dysfunctional uterine bleeding b. Dyspareunia c. Early menopause d. Cancer of the ovaries

D

The advanced-practice nurse is preparing to examine a patient's prostate gland. Before the exam, what does the nurse tell the patient? a. He may feel the urge to defecate or faint as the prostate is palpated. b. He should lie supine with knees bent in a fully flexed position. c. The examination is very painful, but it only lasts a few seconds. d. The gland will be massaged to obtain a fluid sample for possible prostatitis.

D

The nurse is assisting a patient who needs a pelvic examination. Which action will the nurse perform? a. Clean perineum with antiseptic solution. b. Administer a mild analgesic. c. Assess for allergies to iodine. d. Instruct to empty the urinary bladder.

D

The nurse is counseling a woman who was re- cently diagnosed with breast cancer. Which factor has the most influence on the choice for treatment? a. Age at the time of diagnosis b. Overall health status c. Personal choice and self-care capacity d. Extent and location of metastases

D

The nurse sees a laboratory report that indi- cates that the patient's specimen is positive for Treponema pallidum. The nurse will call the health care provider to obtain an order for which medication? A. Valacyclovir b. Levofloxacin c. Azithromycin d. Benzathine penicillin G

D

The nurse sees that the patient was prescribed valacyclovir. Which patient statement indicates that the goal of therapy is being met? A. "The rash is still present, but I hardly notice it." B. "Pain in my joints is much less than before treatment." C. "Abdominal pain is no longer interfering with movement." D. "The sores are not as painful as they were before."

D

The nursing student is writing a report about caring for a 56-year-old patient with diabetes mellitus who identified himself as transgender. Which sentence reflects proper use of terminology? a. "Today I learned how to properly care for a transvestite who has diabetes." b. "I developed a therapeutic relationship with a 56-year-old tranny with diabetes." c. "A 56-year-old transgender had complica- tions related to his diabetes." d. "A 56-year-old transgender patient was admitted for complications due to diabetes."

D

When obtaining a complete obstetric- gynecologic history, the nurse also takes a sexual history from the patient. Which approach is the most therapeutic to elicit information from the patient? a. Use a checklist to ask "yes" and "no" questions. b. Ask the patient to detail her sexual history. c. Ask directly about episodes of sexually transmitted disease. d. Ask open-ended questions.

D

Which group has the greatest risk for contracting primary and secondary syphilis? a. Males aged 15-25 years b. Postmenopausal women c. Women who have sex with women d. Men who have sex with men

D


Related study sets

Econ 331: Money and Banking Test 2 (Chapters 9,12,14,15,16)

View Set

Domestic and Intl. Banking Chp 11

View Set

Midterm 1 Practice Questions [PRACTICE QUIZ CHAP. 6]

View Set

#10 Unit 2 Chapter 8 Preamble Constitutional principles

View Set

Physiology Topic 6.1 Quiz—Transport Part 1

View Set